ORTHOPEDIC MCQS ONLINE QUESTION BANK H2A

ORTHOPEDIC MCQS ONLINE QUESTION BANK H2D

A 4-year-old girl is brought in for examination by her mother because of a bump on the lateral side of her elbow. The girl is unable to extend her elbow. She falls as much as any child, but no particular injury to the elbow is recalled. Radiographs show a dislocated, enlarged radial head that is convex proximally. There is a proximal radioulnar synostosis. Recommended treatment includes:

 

1) Radial head excision

3) Open reduction of the radial head and ulnar lengthening osteotomy

2) Open reduction of the radial head and annular ligament reconstruction

5) No treatment

4) Silastic radial head arthroplasty

 

This child has a congenital dislocation of the radial head, and no treatment is indicated unless the forearm is fixed in a position of extreme malrotation.

 

 

Radial head excision should only be performed if there is pain and the child is skeletally mature. Silastic radial head arthroplasty has a significant risk of particulate synovitis.

Correct Answer: No treatment

 

 

1735. (512) Q5-712:

A 6-year-old child falls and suffers a fracture of the elbow. The tenderness is mostly lateral. A fracture line may be seen separating a fragment of the humeral metaphysis and physis 3 mm from the lateral portion of the elbow, but the distal extension of the fracture cannot be visualized because of lack of ossification at this age. Treatment should consist of:

 

1) No immobilization and early range of motion

3) Application of a long arm cast for 8 weeks

2) Application of a long arm cast for 4 weeks followed by early motion

5) Open reduction and plate fixation

4) Closed reduction and percutaneous pin fixation

 

Closed reduction and pin fixation should be attempted for all lateral condyle fractures that are displaced more than 2 mm. If closed reduction is not successful, open reduction should be performed.

 

 

This is a lateral condyle fracture; nonunion may result with no immobilization and early range of motion. With more than 2 mm of displacement, reduction and fixation should be carried out.

 

Plate fixation is not feasible because of the presence of the physis. Rigid fixation beyond the use of pins is not required.

Correct Answer: Closed reduction and percutaneous pin fixation

 

 

1736. (513) Q5-713:

The following is the most common complication of lateral condyle fractures:

 

1) Radial nerve palsy

3) Heterotopic ossification

2) Compartment syndrome

5) Posterior interosseous nerve injury

4) Nonunion

 

Nonunion occurs more commonly after lateral condyle fracture than following most childrenâs fractures because the fracture line is predominantly intra-articular and not always adequately reduced and immobilized.

 

Radial nerve injury, compartment syndrome, heterotopic ossification, and posterior interosseous nerve injury are uncommon with lateral condyle fractures.

Correct Answer: Nonunion

 

The most common nerve injury in a Monteggia fracture and the type of Monteggia fracture with which it is most commonly found is:

 

1) Ulnar nerve, type I

3) Posterior interosseous nerve, type III

2) Posterior interosseous nerve, type IV

5) Median nerve, type III

4) Median nerve, type I

 

The posterior interosseous branch of the radial nerve is injured in 10%-20% of Monteggia fractures. Most cases occur at the time of fracture, but some occur later. Spontaneous resolution is the usual outcome.

 

The ulnar nerve has only been reported injured in one case.

 

The posterior interosseous nerve is the most commonly injured, at 10-20%, but type I and type III are the most common types affected.

 

The median nerve is the second most commonly injured, after the radial nerve.

Correct Answer: Posterior interosseous nerve, type III

 

 

1738. (515) Q5-715:

A 7-year-old girl is seen because of a persistent anterior dislocation of the radial head that occurred 2 months ago with an ulna fracture. The ulna has healed but has 25° of angulation. Her family would like to have this fixed to remove the prominence in the hope of preventing future joint degeneration. The recommended treatment is:

 

1) Closed reduction of the radial head

3) Open reduction and annular ligament reconstruction

2) Open reduction and pin fixation of the radial head

5) Open reduction, radial shortening, and annular ligament reconstruction

4) Open reduction, annular ligament reconstruction, and ulnar osteotomy

 

The combination of open reduction, annular ligament reconstruction, and ulnar osteotomy should correct all of the components of the deformity.

 

Closed reduction is not successful beyond 1-2 weeks after injury.

 

 

Because of the 25° ulnar bow, recurrence is likely, and ulnar osteotomy is indicated. An ulnar osteotomy should be included to prevent recurrence.

Correct Answer: Open reduction, annular ligament reconstruction, and ulnar osteotomy

 

 

1739. (516) Q5-716:

A 6-year-old patient has an acute proximal ulnar fracture with an apex posteriorly, as well as a radial head dislocation. Treatment at this stage should consist of:

 

1) Closed reduction and immobilization in supination and flexion more than 90° in a long arm cast

3) Closed reduction and intramedullary rod fixation of the ulna

2) Closed reduction and immobilization in extension in a long arm cast

5) Open reduction of the radial head and annular ligament reconstruction

4) Open reduction of the radial head and plate fixation of the ulna

 

Extension will reduce the dislocation and the fracture.

 

 

This type II Monteggia proximal fracture is best immobilized in extension. An intramedullary rod is indicated only if closed treatment fails.

 

 

Open reduction of the radial head and plate fixation of the ulna are indicated only if closed treatment fails. Annular ligament reconstruction is indicated only if the dislocation is unreduced for more than 1 to 2 weeks.

Correct Answer: Closed reduction and immobilization in extension in a long arm cast

 

A 5-year-old patient sustains a fracture of the ulna with apex anteriorly, as well as an anterior dislocation of the radial head. The recommended treatment is:

 

1) Closed reduction with elbow flexed at least 90° and somewhat supinated

3) Closed reduction with the elbow flexed and the forearm in maximal pronation

2) Closed reduction with the elbow in extension

5) Open reduction of the radial head, annular ligament repair, and closed reduction of the ulna

4) Closed reduction of the radial head and intramedullary rod of the ulna

 

Flexion to at least 90° helps redirect the radial head, as does partial supination.

 

 

 

 

Extension of the elbow will increase the tendency to dislocate. Pronation of the forearm will increase the tendency to dislocate. An intramedullary rod is needed only if closed treatment fails. Open reduction is required only if closed treatment fails.

 

Correct Answer: Closed reduction with elbow flexed at least 90° and somewhat supinated

 

 

1741. (518) Q5-718:

A 2-year-old boy fell 4 feet from a countertop and landed on his outstretched hand. There is circumferential swelling and tenderness. Radiographs show no fracture, but the posterior fat pad elbow is elevated and the radius and ulna are translated slightly laterally on the anteroposterior view, and posteriorly on the lateral view. The most likely diagnosis is:

 

1) Undisplaced supracondylar fracture

3) Lateral condyle fracture

2) Transphyseal fracture of the distal humerus

5) Congenital elbow dislocation

4) Traumatic elbow dislocation

 

The separation occurs through cartilage, so it is not visible as a fracture on plain films. However, the translation indicates a problem and elbow dislocation does not occur with any frequency at this age.

 

 

An undisplaced supracondylar fracture would not produce translation of the radius and ulna. A lateral condyle fracture would not produce translation of the ulna.

 

Traumatic dislocation is extremely uncommon in this age group.

 

Congenital elbow subluxation would not account for the swelling and tenderness. This is an extremely rare condition.

Correct Answer: Transphyseal fracture of the distal humerus

 

 

1742. (519) Q5-719:

A physeal fracture-separation of the distal humerus is seen in an 18-month-old boy. When the parents ask about the prognosis after this injury, you tell them that the most common complication is:

 

1) Cubitus varus

3) Ulnar nerve injury

2) Median nerve palsy

5) Undergrowth of the humerus

4) Brachial artery injury

 

Cubitus varus occurred in seven out of twelve physeal fracture separations of the distal humerus in one series. This most likely occurred due to malalignment or avascular necrosis of the trochlea.

 

Median nerve injury, ulnar nerve injury, and brachial artery injury are extremely uncommon after this injury because the surfaces of the metaphysis and the physis are relatively smooth in comparison to supracondylar fractures. Also, these injuries are relatively low-energy.

 

Growth plate damage causing undergrowth is rare after this fracture.

Correct Answer: Cubitus varus

 

 

 

 

 

slide 1

A 12-year-old boy presents to the emergency department after being struck by a car (slide 1). His only complaint at the time of presentation is right ankle pain. After obtaining an excellent reduction and casting the leg, the risks of a future growth disturbance through the involved physis must be discussed with the family. What are the chances of a significant growth disturbance of his leg:

 

1) < 1%

3) 10% to 15%

2) 2% to 3%

5) 80% to 90%

4) 45% to 55%

 

This is a Salter-Harris type II fracture of the distal tibia. The distal tibia is at moderate risk for growth arrest after physeal injury. The average incidence of growth disturbance is 15% for all physeal injuries in this area. The marked displacement and mechanism of injury in this patient increase the risk of permanent physeal damage. The patientâs age and remaining growth also increase the likelihood of a growth arrest causing a significant deformity or leg length discrepancy.Correct Answer: 10% to 15%

 

 

 

1744. (521) Q5-721:

Three years after a Salter-Harris type I physeal fracture of the right distal femur, a 12-year-old boy presents with complaints of knee pain and a limp. On examination, the boy has a valgus alignment of his right knee and a 2-cm leg length discrepancy with the right leg shorter than the left. Plain radiographs and a scanogram showed 30% growth plate closure with a femoral-tibial angle of 12° of valgus and 2.5 cm of shortening of the right femur. What is the best treatment:

 

1) Lengthening procedure on the right leg

3) Physeal bar resection of the right distal femur and opening wedge lateral osteotomy

2) Varus osteotomy of the right femur and bilateral distal femur epiphyseodesis

5) Completion of distal femoral epiphyseal closure

4) Right medial hemiepiphyseal stapling

 

The bar resection has a reasonable chance of success. The angulation can be corrected by the osteotomy and the length would be partially corrected.

 

Correction of the length (2 cm) is less important than prevention of future angulation and shortening.

 

At 12 years old, if varus osteotomy of the right femur and bilateral distal femur epiphyseodesis were performed, the patient would be sacrificing 6 cm of leg length which most patients would consider unacceptable.

 

Right medial hemiepiphyseal stapling and completion of distal femoral epiphyseal closure would neither produce significant length equalization nor correct the angulation.

Correct Answer: Physeal bar resection of the right distal femur and opening wedge lateral osteotomy

 

Two years after a Salter-Harris type II fracture of the right distal femur, a 12-year-old girl presents to the clinic with knee pain. On examination, she is found to have a valgus alignment of the right knee and a 3-cm leg length discrepancy with the right leg shorter than the left leg. The scanogram confirms 3 cm of shortening in the right femur. The next step in the management of this patient is:

 

1) Obtain a magnetic resonance image of the right distal femoral physis

3) Obtain a bone scan

2) Obtain a computed tomography scan of the right distal femoral physis

5) Patientâs return in 9 months for a repeat scanogram

4) Obtain a single photon emission computed tomography scan

 

A magnetic resonance image of the right distal femoral physis provides the best detail of the physis if the correct region and sequences are ordered.

 

A computed tomography scan is not the preferred method to allow visualization of the physis.

 

 

A bone scan or photon emission computed tomography scan will not highlight a bar of the physis. Waiting 9 months would allow for worsening of the deformity.

Correct Answer: Obtain a magnetic resonance image of the right distal femoral physis

 

 

1746. (523) Q5-725:

The mutations underlying Stickler syndrome have been identified in which of the following molecules:

 

1) Type I collagen

3) Fibrillin

2) Type II collagen

5) Fibroblast growth factor receptor protein

4) Sulfate transport protein

 

Type II collagen is abnormal in the classic form of Stickler syndrome, but in a few patients, type XI collagen has been found to be the causative mutation.

 

Type 1 collagen is predominant in bone and is abnormal in osteogenesis imperfecta, for example. Stickler syndrome is characterized by a defect in type II collagen.

 

Fibrillin is abnormal in Marfan syndrome, but in Stickler syndrome, the causative abnormalities have been found in type II collagen.

 

Sulfate transport protein has been found abnormal in diastrophic dysplasia

 

Fibroblast growth factor proteins are abnormal in achondroplasia and hypochondroplasia, but the basic defect in Stickler syndrome is type II collagen.

Correct Answer: Type II collagen

 

A 15-month-old child has bowing of the legs. Examination reveals a 3 cm distance between the femoral condyles and a thigh-foot angle of 20° internal. Radiographs reveal 10° varus of the mechanical axis, no evidence of skeletal dysplasia, and a metaphyseal-diaphyseal angle of 9° on each side. Recommended treatment is:

 

1) Full-time bracing with a knee-ankle-foot orthosis

3) Bilateral hemiepiphyseal stapling of the lateral tibia

2) Bilateral oblique osteotomies of the proximal tibia

5) Reassurance to parents that the legs will develop normally; no further follow-up

4) Re-examination in 6 months

 

Although a diagnosis of Blount disease cannot be made at this time, the disease cannot be ruled out. The patient should be rechecked in 6 months by an experienced examiner. However, if the parents are reliable, they can be told that they do not need to return if the childâs legs develop a normal shape in 6 months.

 

 

 

The diagnosis of pathologic varus cannot be made; therefore, no treatment is indicated. The findings described are not outside of normal limits; surgery is not necessary.

 

No pathologic diagnosis has yet been made. In addition, stapling is not an indicated form of treatment for such a young child.

 

Although a pathologic condition has not been diagnosed, one cannot be ruled out either at this stage.

Correct Answer: Re-examination in 6 months

 

 

1748. (582) Q5-818:

An 11-year-old girl is observed for legs that have been bowed for the past 5 years. She has a mechanical axis that is in 26° of varus, a medial tibial plateau slope of 8°, a tibial joint angle of 15° varus, and a femoral joint angle of 10° varus. Her physis appears open. Recommended treatment is:

 

1) Tibial valgus osteotomy

3) Tibial and femoral valgus osteotomy

2) Femoral valgus osteotomy

5) Brace treatment

4) Medial tibial plateau elevation

 

Both a tibial and a femoral valgus osteotomy are necessary to correct significant deformities in these areas.

 

 

A tibial valgus osteotomy is necessary but not sufficient. The femoral joint angle is off by 13° from the normal angle. A femoral valgus osteotomy is necessary but not sufficient. The tibial joint angle is off by 15° of varus.

 

 

A tibial plateau elevation is not necessary for this mild degree of plateau depression. Brace treatment is not recommended for patients older than 3 years of age.

Correct Answer: Tibial and femoral valgus osteotomy

 

 

1749. (583) Q5-819:

The radiographic feature that is most characteristic of infantile Blount disease is:

 

1) Widening and irregularity of the entire proximal tibial physis

3) Focal bowing of the distal medial femur

2) Poor bone mineralization

5) External tibial torsion

4) Focal bowing of the proximal medial tibia

 

Bowing is focally located at the proximal medial tibia in infantile Blount disease.

 

 

The physis may be widened medially, but it is normal laterally. Bone mineralization is not impaired in Blount disease.

 

 

Bowing is focally located at the proximal tibia, not the distal femur. Blount disease is associated with internal, not external, tibial torsion.

Correct Answer: Focal bowing of the proximal medial tibia

 

Marfan syndrome is now recognized as a defect in the following molecule:

 

1) Fibrillin-1

3) Type II collagen

2) Type I collagen

5) Type IX collagen

4) Sulfate transport protein

 

Marfan syndrome is due to an abnormality in fibrillin-1, a component of elastic connective tissue.

 

 

Type I collagen is most commonly abnormal in osteogenesis imperfecta. Type I collagen is normal in Marfan syndrome. Type II collagen is a homotrimer (encoded by a single gene) and is mainly located in cartilaginous tissues. Type II collagen is abnormal in spondyloepiphyseal dysplasia congenita and Stickler syndrome.

 

The sulfate transport protein is important for cartilage formation. The sulfate transport protein is impaired in diastrophic dysplasia.

 

Type IX collagen is also found in cartilage. Type IX collagen is abnormal in some forms of multiple epiphyseal dysplasia.

Correct Answer: Fibrillin-1

 

 

1751. (585) Q5-821:

A 15-year-old basketball player has mild scoliosis, pes planus, pectus carinatum, and long slender fingers. In order to help determine if he has Marfan syndrome and should be allowed to continue playing basketball, it is most useful to order a:

 

1) Cardiac echo

3) Slit lamp examination

2) Magnetic resonance image of the lumbosacral spine

5) Dermal fibroblast assay

4) Chest radiograph

 

A cardiac echo will give information about enlargement of the ascending aorta, as well as the valves. This is the most important information for activity recommendations and patient prognosis.

 

A magnetic resonance image (MRI) of the lumbosacral spine will either rule in or out dural ectasia, but a MRI has no clinical importance for activity recommendations.

 

 

A slit lamp examination is not as important as a cardiac echo in making sport recommendations. A chest radiograph may be helpful, if the patient is suspected of having a pneumothorax.

 

A dermal fibroblast assay is useful in assessing collagen, such as in osteogenesis imperfecta, but it is not helpful in diagnosing Marfan syndrome.

Correct Answer: Cardiac echo

 

 

1752. (586) Q5-823:

The following skeletal feature helps to establish a diagnostic level of major skeletal involvement in Marfan syndrome:

 

1) Hyperextensible knees

3) Standing height over 2 m

2) Atlantoaxial subluxation

5) Pectus carinatum

4) Increased thoracic kyphosis

 

Pectus carinatum has a high diagnostic value for Marfan syndrome.

 

Hyperextension of the knees is not common in patients with Marfan syndrome. This skeletal feature is seen in many other conditions; therefore, it has low diagnostic specificity.

 

Atlantoaxial subluxation is rare in Marfan syndrome, but is common in some of the skeletal dysplasias.

 

 

Although patients with Marfan syndrome are generally tall, height is too nonspecific to be a helpful diagnostic criterion. Kyphosis is not a skeletal diagnostic criterion.

Correct Answer: Pectus carinatum

 

A 12-year-old girl has a scoliosis of 36° from T2-T7 and 15° from T7-L1. She is premenarchal. The following treatment is recommended:

 

1) Charleston bending brace

3) Milwaukee brace

2) Boston overlap brace

5) Posterior spinal fusion

4) Lateral electrical spinal stimulation

 

The Milwaukee brace offers the best chance of controlling this curve.

 

 

 

 

Charleston bending braces are not effective for curves larger than 35°. Boston overlap braces are not effective for curves with an apex above T8. Lateral electrical spinal stimulation has been proven ineffective for scoliosis. This curve is not large enough to pose a recommend fusion.

 

Correct Answer: Milwaukee brace

 

 

1754. (588) Q5-825:

All of these findings are features of patients with Scheuermann kyphosis, except:

 

1) Vertebral wedging

3) Schmorl nodes

2) Endplate irregularity

5) Back pain

4) Truncal obesity

 

Obesity is not any more common in patients with Scheuermann kyphosis than in the general population.

 

 

Vertebral wedging is a common feature of Scheuermann kyphosis. Endplate irregularity is a common feature of Scheuermann kyphosis.

 

 

Schmorl nodes are a manifestation of the disordered behavior of the vertebral endplates under load. Back pain is common in Scheuermann kyphosis.

Correct Answer: Truncal obesity

 

 

1755. (589) Q5-826:

An 18-year-old man is seen in the office because of back pain in the thoracic region. He has a kyphosis of 65°, a slight wedging in the midthoracic spine, and a Risser sign of 4. Recommended treatment includes:

 

1) A program of strengthening and stretching exercises

3) A Boston brace

2) A Milwaukee brace

5) An anterior and posterior spinal fusion

4) A posterior spinal fusion

 

Exercises must be the first treatment for this patient because he is too mature for brace treatment.

 

 

Brace treatment is not indicated for someone of this skeletal maturity. Surgery is not indicated unless the patient fails conservative treatment.

Correct Answer: A program of strengthening and stretching exercises

 

Prior to treatment, this pathologic finding characterizes clubfoot:

 

1) The talar head and neck are deviated medially.

3) The navicular is positioned more laterally than normal.

2) The dome of the talus is flattened.

5) The tendoachilles inserts in a more lateral position than normal.

4) The foot has a rocker deformity.

 

The talar head and neck are deviated medially. This deformity cannot be corrected surgically but may improve with growth.

 

 

The dome of the talus is not originally flattened, but it may become this way after repeated manipulation. The navicular is positioned more medially than normal and may touch the medial malleolus.

 

The foot has either a normal arch or a cavus deformity (the opposite of a rocker bottom). The rocker bottom foot may develop as a result of over-vigorous manipulation of a clubfoot against a tight heelcord.

 

The tendoachilles inserts are more medially positioned than normal on the calcaneus.

Correct Answer: The talar head and neck are deviated medially.

 

 

1757. (591) Q5-828:

In the surgical correction of a clubfoot, the following clinical or radiographic finding indicates that a child should have a plantar release:

 

1) Anterior extrusion of the talus

3) Medial subluxation of the calcaneocuboid joint

2) Inferior subluxation of the calcaneocuboid joint

5) Lack of ability to dorsiflex the ankle

4) Plantarflexion of the first ray

 

Plantarflexion of the first ray, especially if the first ray is not colinear with the talus on the lateral radiograph, should serve as an indication for a plantar release.

 

 

Anterior extrusion of the talus is common because of the tightness of the Achilles tendon and the posterior capsule. Inferior subluxation of the calcaneocuboid joint is not a feature of clubfoot.

 

Medial subluxation of the calcaneocuboid joint, in combination with the clinical appearance of a curved lateral border of the foot, should serve as an indication for a lateral release, not a plantar release.

 

Inability to dorsiflex the ankle is an indication for a posterior release.

Correct Answer: Plantarflexion of the first ray

 

 

1758. (592) Q5-830:

Hip subluxation is most likely to occur in patients with this type of cerebral palsy:

 

1) Athetoid

3) Hemiplegic

2) Diplegic

5) Monoplegic

4) Total involvement

 

Patients with total involvement have the highest degree of axial muscle imbalance; therefore, they have the highest risk of hip subluxation (greater than 50%).

 

Hip dysplasia is uncommon in athetoid cerebral palsy (CP).

 

 

Diplegic CP has the second highest incidence of neuromuscular hip dysplasia, but total involvement has the highest rate. Hip subluxation is not common in hemiplegic CP.

 

Monoplegia is a rare form of CP with a low rate of subluxation.

Correct Answer: Total involvement

 

A 13-year-old boy sustains a Salter II fracture of the proximal humeral epiphysis. On radiograph, there is a 40° varus angulation and a 30° apex anterior angulation. Recommended treatment includes:

 

1) Closed reduction and abduction cast

3) Closed reduction and percutaneous pin fixation

2) Closed reduction and sling

5) Application of a sling and swathe

4) Open reduction and percutaneous pin fixation

 

Spontaneous partial reduction will occur when the patient becomes upright, and there is good remodeling potential due to growth.

 

Closed reduction and abduction cast is not necessary because of the patientâs age, remodeling potential, and range of motion available in the joint.

 

A formal attempt at closed reduction is not necessary because there is no way of holding it without internal fixation. This process will occur naturally to a large degree when the patient becomes upright.

 

Open reduction and percutaneous pin fixation poses a risk of pin tract infection because of the large amount of muscle traversed. This procedure is not necessary because of the good results with conservative treatment.

Correct Answer: Application of a sling and swathe

 

 

1760. (594) Q5-832:

A 12-year-old girl sustains a closed type III Monteggia fracture. One week after closed reduction, the radial head resubluxates and the ulna bows. The next step of treatment is:

 

1) Repeat closed reduction and a long arm cast

3) Reduce and internally fix the ulna and close-reduce the radial head

2) Reconstruct the annular ligament using a strip of triceps fascia

5) Reduce and pin the radial head

4) Reduce and internally fix the ulna and open-reduce the radial head

 

Once the ulna is predictably reduced, the radial head has a good chance of staying reduced.

 

Repeat closed reduction and a long arm cast has a significant chance of redisplacement. If further time passes, radial head reduction will be difficult.

 

Annular ligament reconstruction is only indicated if there is late subluxation (after several weeks) that does not reduce with closed means.

 

Once the ulna is realigned and fixed, closed reduction of the radius must be tried next.

 

Pinning of the radial head should be a last resort because of the risk of breakage and stiffness.

Correct Answer: Reduce and internally fix the ulna and close-reduce the radial head

 

 

1761. (595) Q5-833:

The following parameter is the most useful in predicting the need for surgical correction of developmental coxa vara:

 

1) The range of active abduction

3) The acetabular index

2) The epiphyseal extrusion index

5) Presence of a Trendelenburg gait

4) The Hilgenreiner-epiphyseal angle

 

The Hilgenreiner-epiphyseal angle is a measure of the shear stress on the femoral neck. A value greater than 60° is an indication for surgery.

 

 

The range of abduction does not directly reflect the mechanical stresses on the femoral neck that may need correction. The epiphyseal extrusion index is a parameter used in Perthes disease.

 

The acetabular index is used in developmental hip dysplasia. It is usually essentially within normal limits, even in severe coxa vara.

Correct Answer: The Hilgenreiner-epiphyseal angle

 

A 7-month-old girl is newly seen for a dislocation of the left hip. The newborn exam was unremarkable; there was no history of trauma or evidence of spasticity. Recommended treatment includes:

 

1) Exam, arthrogram and attempted closed reduction under anesthesia to guide treatment

3) Open reduction through a medial approach

2) Pavlik harness

5) A Salter osteotomy

4) Open reduction through a lateral approach

 

An exam under anesthesia and arthrogram should be performed. Closed reduction must be performed if the hip reduces with a medial dye pool less than 4 mm and a safe zone greater than 20°.

 

 

The Pavlik harness is not strong enough to hold an infant older than 4 to 6 months. Open reduction is reserved for hips that do not reduce by closed means.

 

A Salter osteotomy is only indicated if the hip is still unstable after an open reduction.

Correct Answer: Exam, arthrogram and attempted closed reduction under anesthesia to guide treatment

 

 

1763. (597) Q5-835:

A 4-year-old girl has unilateral idiopathic avascular necrosis involving the entire femoral head. Sixty percent of the height of the lateral column is resorbed. There is no lateral subluxation or physeal disturbance. The range of abduction is 30°.

Recommended treatment consists of:

 

1) Observation

3) Femoral osteotomy

2) Atlanta Scottish Rite orthosis

5) Petrie cast

4) Iliac osteotomy

 

The young age is the overwhelmingly positive factor in this patient. None of the findings mandate a more vigorous approach than observation.

 

 

The Atlanta Scottish Rite orthosis has not been proven to have any effect on the disorder. Femoral osteotomy would only be possibly indicated, if there were subluxation greater than 25%.

 

 

Iliac osteotomy would only be indicated in this age group, if there were subluxation and/or other poor prognostic factors. Petrie cast is indicated only if the patient has subluxation.

Correct Answer: Observation

 

 

1764. (598) Q5-836:

A 13-year-old girl has hip pain and the inability to bear weight. On anteroposterior and lateral hip radiographs the femoral head is displaced inferiorly and posteriorly by 50% of its diameter. Recommended treatment includes:

 

1) Closed reduction and immobilization in a spica cast

3) Open reduction and pin fixation

2) Traction followed by internal fixation

5) In situ fixation with screw(s)

4) Metaphyseal osteotomy and internal fixation

 

In situ fixation has results superior to any of the other methods.

 

 

Besides being extremely cumbersome, spica cast treatment carries significant risk of redisplacement and chondrolysis. There is no indication for traction in this situation.

 

The amount of displacement can be accepted as long as it is stabilized.

 

Metaphyseal osteotomy carries a risk of avascular necrosis. It should only be undertaken in grade III slips.

Correct Answer: In situ fixation with screw(s)

 

A 12-year-old boy is brought to the emergency department with posterior left pelvic pain, a limp, a temperature of 101° F, and a white blood count of 15,000. C-reactive protein is 55. Magnetic resonance imaging shows increased signal within the posterior ilium on T2 weighted images, but no fluid is outside the bone. Recommended treatment includes:

 

1) Obtaining a percutaneous bone biopsy

3) Oral dicloxacillin

2) Irrigation and debridement

5) Intravenous oxacillin and gentamicin

4) Intravenous oxacillin

 

Staphylococcal osteomyelitis is the presumptive diagnosis; therefore, intravenous oxacillin should be administered.

 

Infection is much more likely than tumor. The predicted organisms should be sensitive to oxacillin. A bone biopsy is not mandatory unless the patient fails to respond to intravenous antibiotics.

 

Debridement is not likely to be necessary given no evidence of a sequestrum.

 

Although acute hematogenous osteomyelitis is the likely diagnosis, it is standard practice to begin treatment with intravenous antibiotics and then switch to oral antibiotics when a clinical response is evident.

Gentamicin is not needed in this setting.

Correct Answer: Intravenous oxacillin

 

 

1766. (600) Q5-838:

A newborn infant in the nursery must be seen because of his foot. The dorsum of the foot rests against the tibia. The heel moves up when the forefoot moves down. Power is present in all muscles. The foot has an arch and the leg lengths are equal. The diagnosis is:

 

1) Fibular hemimelia

3) Calcaneovalgus foot

2) Vertical talus

5) Lipomeningocele

4) Tethered cord

 

Calcaneovalgus foot has all of these findings and resolves spontaneously.

 

 

 

Fibular hemimelia typically has less calcaneus attitude and more valgus and shortening. Vertical talus entails loss of an arch and loss of cohesive movement of the foot as a whole. There is no evidence of muscle weakness.

 

There is no evidence of a neuropathic component.

Correct Answer: Calcaneovalgus foot

 

A newborn girl is noted to have decreased movement in the right upper extremity. She was large (10 lbs) at birth and was delivered vaginally with shoulder dystocia. She does not have elbow flexion, external shoulder rotation, or abduction. She has had weak finger flexion for 3 months. At 4-months-old, she regains the ability to flex her elbow. Recommended treatment includes:

 

1) Magnetic resonance imaging of the shoulder

3) Physical therapy

2) Tendon transfers of the teres major and latissimus

5) Open reduction of the glenohumeral joint

4) Microvascular repair of the brachial plexus

 

Conservative therapy is predicted to bring a good result because biceps are returning at four months of age. However, stretching of the shoulder is indicated to maintain a range of external rotation and abduction.

 

Magnetic resonance imaging is only indicated if there is a need to consider microvascular repair.

 

Tendon transfers are performed later (at several years of age), if shoulder abduction and external rotation are significantly limited.

 

Microvascular repair is mainly considered in patients who do not have return of biceps function by five months.

 

Open reduction is indicated later (if the shoulder joint is subluxated or severely contracted) after motor recovery has reached a plateau.

Correct Answer: Physical therapy

 

 

1768. (602) Q5-840:

A 7-year-old boy with diplegic cerebral palsy has had lengthening of his hamstrings and heelcords. He is examined 2 years later. He walks with the knees nearly straight throughout the gait cycle and circumducts each extremity during swing. This pattern is most likely due to:

 

1) Recurrent hamstring tightness

3) Spasticity of the rectus femoris

2) Overlengthening of the heelcords

5) Spasticity of the psoas muscle

4) Contracture of the tensor fascia lata

 

 

The straight legs and circumduction suggest spasticity of the rectus femoris as the most likely cause. Recurrent hamstring tightness (by itself) would cause the opposite combination of problems.

 

Overlengthening of the heelcords usually causes increased dorsiflexion at the ankles and would help to increase the flexion moment rather than the extension.

 

 

Contracture of the tensor fascia lata is extremely rare in cerebral palsy. Spasticity in the psoas muscle would not lead to a stiff knee gait.

Correct Answer: Spasticity of the rectus femoris

 

 

1769. (603) Q5-841:

A 3-year-old girl is brought in for evaluation of leg alignment. She has bilateral foot progression angles of 35° internal. Her thigh-foot angles are 40° internal. Her hip rotation in the prone position is 50° external and 30° internal. The metaphyseal-diaphyseal angle is 2° on each side. Recommended treatment includes:

 

1) Denis Browne bar with feet 45° outward

3) Femoral derotation osteotomy

2) Bilateral double-upright knee-ankle-foot orthoses

5) Observation

4) Tibial osteotomy

 

The tibial torsion described has an excellent chance of resolution over time. Observation is indicated.

 

 

This child has tibial torsion. The Denis Browne bar has not been proven to affect the natural history of tibial torsion. The knee-ankle-foot orthoses are used for genu varum, which is not the primary problem in this case.

 

 

A femoral osteotomy is rarely used to correct femoral anteversion in older children. A tibial osteotomy is rarely used to correct tibial torsion in older children.

Correct Answer: Observation

 

1770. (604) Q5-842:

A newborn baby has a foot that is dorsiflexed and in valgus. The differential diagnosis includes all of the following conditions except:

 

1) Calcaneovalgus foot

3) Muscle imbalance from an L5 myelomeningocele

2) Vertical talus

5) Tibial hemimelia

4) Posteromedial bow of the tibia

 

The foot in a patient with tibial hemimelia does not resemble the other four conditions described; the foot is in equinus and varus.

 

 

Calcaneovalgus foot is dorsiflexed and everted through the axis of the ankle joint. A vertical talus has excessive forefoot dorsiflexion and valgus.

 

A patient with an L5 myelomeningocele may have this appearance due to activity of the dorsiflexors and evertors, with absent power in the plantarflexors and invertors.

 

Due to the posteromedial bow in the tibia, the foot may appear dorsiflexed and in valgus.

Correct Answer: Tibial hemimelia

 

 

1771. (605) Q5-843:

A 6-year-old girl with osteogenesis imperfecta has severe bowing of both femurs. The family is interested in surgery to correct the condition. This method will give her the longest interval between procedures in the future:

 

1) Closed osteoclasis and cast application

3) Multiple osteotomies and realignment over parallel rods

2) Multiple osteotomies and realignment over a single smooth rod

5) Excision of the abnormal bone and replacement with allograft

4) Multiple osteotomies and realignment over telescoping rods

 

Telescoping or Bailey-Dubow rods are able to grow with the patient and maintain stabilization for a mean of 4 years between operations in growing children.

 

Without the addition of internal fixation, bowing is likely to resume.

 

Although a single rod is likely to provide current stability, bowing is likely to occur as the patient grows and the bone becomes longer than the rod.

 

Parallel rods do not interlock and are likely to splay and lose fixation.

 

There are no reports of large segment allograft replacement for diaphyses in osteogenesis imperfecta. In the absence of internal fixation, bowing is likely to resume as growth occurs.

Correct Answer: Multiple osteotomies and realignment over telescoping rods

 

 

1772. (606) Q5-844:

An infant is seen in the office for the first time. She has slender, stiff fingers with few creases, elbow range of motion 20° to 45°, and internally rotated arms. Her legs are flexed and externally rotated at the hip. Knee range of motion is 15° to 40° of flexion, and she has bilateral clubfeet that are stiff. The most likely diagnosis is:

 

1) Larsen syndrome

3) Ehlers-Danlos syndrome

2) Cerebral palsy

5) Arthrogryposis

4) Down syndrome

 

The patient has enough joint contractures to make arthrogryposis a likely diagnosis.

 

Larsen syndrome is characterized by dislocation of multiple joints.

 

 

Cerebral palsy is characterized by spasticity and flexors overpowering extensors. Ehlers-Danlos is characterized by hyperlaxity of the large joints.

 

Down syndrome does not present with stiffness.

Correct Answer: Arthrogryposis

 

Which of the following statements best characterizes the natural history of metatarsus adductus in a newborn:

 

1) Metatarsus adductus is likely to become fixed if not treated with casts.

3) Metatarsus adductus is likely to become fixed if not surgically corrected.

2) Metatarsus adductus is likely to become fixed if not treated by 6 months.

5) Most infants will improve spontaneously.

4) Metatarsus adductus is likely to later develop hindfoot equinus.

 

Virtually all patients with metatarsus adductus will improve with time in the absence of active treatment.

 

 

Casts are not needed for the majority of cases because spontaneous improvement is by far the most common outcome. Reverse last shoes are not needed in the majority of patients with metatarsus adductus.

 

Most patients will not need surgery.

 

Equinus of the hindfoot is not part of the pathology in metatarsus adductus.

Correct Answer: Most infants will improve spontaneously.

 

 

1774. (608) Q5-846:

A 2½-year-old boy has severe spastic diplegia and hips that are subluxating. His abduction is 20° on each side. The migration index is 35% on each side. Recommended treatment includes:

 

1) Traction followed by bilateral Salter osteotomy

3) Nighttime abduction splinting

2) Trochanteric transfer

5) Physical therapy each day to stretch abductors along with night

4) Bilateral adductor lengthening and abduction bracing

 

If followed by postoperative bracing, lengthening of the adductors is usually successful in children younger than 4 to 6 years of age.

 

Traction would not be appropriate because the muscles are spastic. The surgical plan does not include the most important component â weakening the overactive adductors.

 

Trochanteric transfer is not advisable in patients younger than 8 years old. Trochanteric transfer would not help decrease the overpull of the adductors which is the cause of the subluxation.

 

Nighttime abduction splinting should be a component of the care after surgery. Bracing alone would not be an adequate treatment.

 

Physical therapy in the absence of surgery does not seem to be enough to overcome the pull of the adductors.

Correct Answer: Bilateral adductor lengthening and abduction bracing

 

 

1775. (609) Q5-847:

A 15-year-old boy has a Salter type-2 fracture of the distal tibia. His foot is in valgus and external rotation. Recommended treatment is:

 

1) Obtaining computed tomograms of the fracture

3) Closed reduction and immobilization in a long leg cast

2) Obtaining a magnetic resonance imaging study of the ankle

5) Open reduction and internal fixation

4) Closed reduction and percutaneous fixation of the fracture

 

There is a good chance that a satisfactory closed reduction may be achieved by correcting the valgus and external rotation.

 

The fracture should first be reduced. There is a good chance of a satisfactory reduction. The patient is old enough that a significant physeal growth disorder is not likely.

 

Magnetic resonance imaging is not indicated unless there is a question of physeal disorder after healing or an entrapped fragment.

Correct Answer: Closed reduction and immobilization in a long leg cast

 

Adolescent girls with multiple radiographs for idiopathic scoliosis are statistically at increased risk for which of the following problems later in life:

 

1) Lung cancer

3) Lymphoma

2) Breast cancer

5) Squamous carcinoma

4) Leukemia

 

In a historical cohort study, the risk was increased to 1.7 times the expected rate of breast cancer. The radiation dose is currently lower. The exposure to the breast may be lowered by taking posteroanterior rather than anteroposterior films, and eliminating lateral films in routine situations.

 

Correct Answer: Breast cancer

 

 

1777. (611) Q5-849:

The mean amount of growth height occurring in each vertbra per year during later childhood and early adolescence is:

 

1) 0.7 mm

3) 2.9 mm

2) 2 mm

5) 6 mm

4) 4 mm

 

The mean growth is 0.7 mm per vertebra per year. This measurement is averaged across all of the thoracic and lumbar vertebrae.

Correct Answer: 0.7 mm

 

 

1778. (612) Q5-850:

Which of the following features is true of congenital scoliosis but not infantile idiopathic scoliosis:

 

1) Bracing has been shown to decrease progression.

3) The thoracic curve may be convex to either the left side or the right in either curve type.

2) The age of onset is before 3 years old.

5) Vertebrae are abnormally formed from birth.

4) The rib-vertebral angle difference predicts the risk of worsening.

 

In congenital scoliosis, the vertebrae are abnormally formed from birth. The vertebrae are normal at birth in infantile idiopathic scoliosis.

 

Age of onset is before age 3 in both types of scoliosis.

 

The thoracic curve may be convex to the left slide or the right side in either curve type. In infantile idiopathic scoliosis, it is most commonly convex to the left.

 

The rib-vertebral angle difference (angle between the apical vertebral endplate and the rib on the convexity minus the rib on the concavity) greater than 20° predicts an increased risk of worsening in infantile idiopathic scoliosis but not in congenital scoliosis.

 

Bracing has not been shown to affect infantile idiopathic scoliosis.

Correct Answer: Vertebrae are abnormally formed from birth.

 

A 12-year-old child with sickle cell anemia has had pain in the distal femur for 1 day, a temperature of 101.5° F, and a white blood count of 14,000/mm3 . Plain films are unremarkable. Recommended treatment includes:

 

1) Magnetic resonance imaging of the area

3) Needle biopsy of the distal femur

2) Bone scan

5) Intravenous hydration and analgesia

4) Indium labeled white cell scan

 

Because the odds are greatly in favor of a noninfectious process, it is appropriate to treat empirically with rehydration and analgesia.

 

 

Magnetic resonance imaging would not distinguish between infection and vaso-occlusive crisis. Bone scan would not distinguish between infection and vaso-occlusive crisis.

 

 

Needle biopsy is not necessary at this stage since the odds are high (greater than 95%) that the process is not infection. White blood cell scan is not able to distinguish between infection and vaso-occlusive crisis.

Correct Answer: Intravenous hydration and analgesia

 

 

1780. (614) Q5-852:

A 6-year-old girl is wearing a seatbelt but no shoulder harness when the car she is occupying strikes another car. She suffers an abdominal contusion as well as a spine injury. The facets of L1 and L2 are spread apart as is the disk between them. The angle between the two vertebrae is 35°. The neurologic exam is within normal limits. Recommended treatment includes:

 

1) Reduction and immobilization in a hyperextension cast

3) Traction for 3 weeks followed by an orthosis

2) Reduction and immobilization in a plastic orthosis

5) Open reduction, in situ fusion, and cast immobilization

4) Open reduction, instrumentation, and fusion L1-L2

 

Open reduction and 2-level fusion is the simplest way of handling this injury.

 

 

Immobilization in a hyperextension cast is likely to exacerbate the abdominal injury. An orthosis is not likely to reduce the deformity.

 

 

Traction would not be a good mechanism to reduce the deformity. In situ fusion would be insufficient without instrumentation.

Correct Answer: Open reduction, instrumentation, and fusion L1-L2

 

 

1781. (650) Q5-893:

Secondary ossification of the elbow

 

1) Occurs over a two-year period

3) Proceeds in a predictable fashion through skeletal maturity

2) Occurs more rapidly in males

5) Is complete by age 10

4) Is not important for the orthopedist to understand

 

Secondary ossification is very imprtant in managing fractures. It begins at age 1-2 and is complete by 14 years girls and 16 years in boys.Correct Answer: Proceeds in a predictable fashion through skeletal maturity

 

 

1782. (651) Q5-894:

Definitive diagnosis of septic arthritis is made by:

 

1) Magnetic resonance imaging

3) Needle aspiration

2) Plain radiographs

5) Serologic testing

4) Open biopsy

 

Definitive diagnoisis of septic arthritis is made by needle asperation.Correct Answer: Needle aspiration

 

 

1783. (652) Q5-895:

Osteomyelitis in the child

 

1) requires operative debridement in the majority of cases.

3) never causes growth disturbance of the involved bone.

2) requires antibiotic therapy for 3 to 6 weeks.

5) Always crosses the physis in children

4) occurs in the diaphysis in most cases.

 

Osteomyelitis in the child requires 3-6 weeks of antiobiotics which may be administered parenterally or internally.Correct Answer: requires antibiotic therapy for 3 to 6 weeks.

 

 

1784. (4039) Q5-896:

All of the following characterize lateral patellar compression syndrome except:

 

1) Anterior knee pain localized to the patellofemoral joint

3) Subluxation of the patella

2) A tight lateral retinaculum

5) May be unilateral or bilateral

4) A lateral patella tilt with narrowing of the lateral patella and femoral articular surfaces on the sunrise radiograph

 

By definition, patients with a lateral patellar compression syndrome do not have a subluxation or dislocation of the patella.Correct Answer: Subluxation of the patella

 

 

1785. (653) Q5-898:

Osteochondral defects occur bilaterally in the distal femur in approximately:

 

1) 10% to 20% of patients.

3) 30% to 40% of patients.

2) 20% to 30% of patients.

5) 80% to 90% of patients

4) 60% to 70% of patients.

 

Osteochondral defects occur bilaterally in the distal femur for approximately 20% to 30% of patients. The fact that 20% to 30% of patients with an osteochondral lesion in the distal femur have bilateral involvement suggests that there is a predisposition to the development of a lesion at this location, either genetic or secondary to repetitive microtrauma.Correct Answer: 20% to 30% of patients.

 

 

1786. (654) Q5-899:

The magnetic resonance imaging signs that suggest instability of an osteochondral dissecans lesion include all the following except:

 

1) The presence of a low signal intensity line at the interface between the lesion and the underlying bone.

3) Focal cartilaginous defects.

2) Cartilaginous fractures.

5) Fluid signal between lesion and bone

4) Cysts underlying the osteochondritis dissecans lesion.

 

The presence of a high signal intensity line at the interface between the lesion and the underlying bone suggests instability of an osteochondritis dissecans lesion.Correct Answer: The presence of a low signal intensity line at the interface between the lesion and the underlying bone.

 

 

 

1) Has no primary ossification centers visible at birth.

3) Ossifies the carpus from the margins, centrally.

2) Demonstrates secondary ossification centers from proximal to distal.

4) Is proportionately different from the mature hand.

 

The neonatal hand is proportionally different from the mature hand.Correct Answer: Is proportionately different from the mature hand.

 

 

1788. (656) Q5-904:

Pediatric bone:

 

1) Has a higher modulus of elasticity than adult bone.

3) Has a short plastic phase on the load-deformation curve.

2) Has lower bending strength than adult bone.

5) Is more mineralized than adult bone

4) Has periosteum, which is more prone to tear than adult periosteum.

 

Pediatric bone has less mineral and more vascular channels than adult bone. This gives it a lower bending strength and lower modules of elasticity than adult bone.Correct Answer: Has lower bending strength than adult bone.

 

 

1789. (657) Q5-905:

In congenital lesions characterized by failure of formation of parts, the most functional, without treatment, is/are:

 

1) Transverse arrest at the wrist.

3) VATER syndrome.

2) Central deficiencies.

5) Radial deficiencies

4) Ulnar deficiencies.

 

Central deficiencies allow a wide grasp, good release and pinch. These are also termed "cleft hand". The other conditions produce greater impairment.Correct Answer: Central deficiencies.

 

 

1790. (658) Q5-906:

Which category of failure of formation anomalies is most often associated with systemic anomalies?

 

1) Radial deficiencies.

3) Central deficiencies.

2) Ulnar deficiencies.

5) Tibial deficiencies

4) Transverse deficiencies.

 

Radial deficiencies are often seen in the later association. which may include cardiac,renal,anorectal and tracheoesphogeal abnormalities.Correct Answer: Radial deficiencies.

 

 

1791. (659) Q5-910:

Arthrogryposis multiplex congenita:

 

1) Primarily affects joints, and secondarily the muscles which move them.

3) Is typically asymmetrical.

2) Is more often neuropathic than myopathic.

5) Has a natural history of increasing joint range of motion with time

  1. Does not respond to passive joint mobilization.

     

    Arthrogryposis multiplex congenita is an idiopathic disorder that may be due to a primary deficiency of anterior horn cells. Arthrogryposis multiplex congenita results in lack of muscle development; the joint stiffness is secondary to this. It is usually reasonably symmetrical.Correct Answer: Is more often neuropathic than myopathic.

     

     

     

    1) 80%

    3) 50%

    2) 66%

  2. 5%

4) 20%

 

The percentage of the genome that represents the sequence of our genes is approximately 5%. The rest of the genome codes are for initiator and termination sequences, maintenance functions, and unknown functions.

Correct Answer: 5%

 

 

1793. (678) Q5-937:

In studying a newly recognized disorder using a large population of affected individuals, geneticists discover that although the disorder often affects siblings, it was rarely, if ever, detected in their ancestors. This disorder most closely follows which pattern of inheritance:

 

  1. Autosomal dominant

3) Sex-linked

2) Autosomal recessive

5) Anticipation

4) Multifactorial

 

Autosomal recessive conditions classically show âhorizontalâ inheritance. Ancestors do not display the gene because they would likely have only one copy of the mutant allele. Only when two carriers reproduce is the phenotype manifest in approximately one-fourth of their offspring.

 

 

Autosomal dominant inheritance is characterized by vertical transmission. Many generations manifest the trait because it takes only a single copy of a mutant allele to display the phenotype.

 

 

Sex-linked conditions are often traced back in a family. Normally the males are affected and the females are carriers. Multifactorial conditions are thought to result from the combination of different genes. Although the risk of recurrence in kindred is somewhat greater than the population as a whole, it is still quite low (only a few percent). It is rare for siblings to be affected.

 

Anticipation refers to the phenomenon in which successive generations are likely to display more severe forms of a given disorder. Myotonic dystrophy is a classic example of this phenomenon.

Correct Answer: Autosomal recessive

 

 

1794. (679) Q5-938:

Diseases caused by enzyme deficiency are commonly inherited by which of the following patterns:

 

1) Autosomal dominant

3) X-linked dominant

2) Autosomal recessive

5) Non-mendelian

4) Multifactorial

 

Two copies of a mutant allele are required to reduce enzyme function to levels that cause clinical impairment.

 

Enzyme defects are rarely inherited by an autosomal dominant pattern because even half of the normal activity of most enzymes is adequate to maintain normal function.

 

Enzyme defects are rarely inherited in an X-linked dominant pattern because one copy of a mutant allele is usually sufficient.

 

Multifactorial inheritance refers to the interaction of multiple, or different genes, to produce a disorder. Enzyme deficiencies are typically the result of a defect in a single gene.

 

Because enzymes are typically coded by a single gene, they follow mendelian patterns.

Correct Answer: Autosomal recessive

 

 

 

1) Alpha-L-iduronidase

3) Beta-glucuronidase

2) Galactose-6-sulfatase

5) Sulfate transport protein

4) Fibroblast growth factor receptor protein

 

Morquio syndrome is a member of the family of mucopolysaccharidoses. Morquio syndrome is a deficiency in the enzyme galactose-6-sulfatase. A deficiency in galactose-6-sulfatase results in increased urinary excretion of keratosulfate.

 

Alpha-L-iduronidase is deficient in Hurler syndrome.

 

 

 

Beta-glucuronidase is deficient in some rare mucopolysaccharidoses. Fibroblast growth factor receptor protein is deficient in achondroplasia. Sulfate transport protein is deficient in diastrophic dysplasia.

 

Correct Answer: Galactose-6-sulfatase

 

 

1796. (681) Q5-940:

Polymerase chain reaction (PCR) is best characterized by which of the following descriptions:

 

1) Use of enzymes to link chains of deoxyribonucleic acid (DNA) together

3) Denaturing and reannealing DNA multiple times with known primers

2) Use of viral vectors to insert new DNA into a cell

5) The process by which a cell-surface receptor turns on the transcription process

4) Use of high temperatures to create ultra-high molecular weight polyethylene

 

Polymerase chain reaction refers to denaturing DNA, isolating a segment of interest with known primers, and reannealing the strands multiple times to produce exponential copies of a segment.

Correct Answer: Denaturing and reannealing DNA multiple times with known primers

 

 

1797. (682) Q5-941:

Pleiotropy is demonstrated by which of the following examples:

 

1) Patients with osteogenesis imperfecta differ in the number of fractures they have received.

3) Hurler syndrome is usually not present in prior generations of an affected patient.

2) Patients with hemophilia A have different target joints.

5) Patients with Ollier disease often have more involvement on one side of the body.

4) Some patients with Marfan syndrome have scoliosis or pectus carinatum, while other patients with Marfan syndrome do not.

 

The term pleiotropy refers to a disease taking different shapes in various patients.

 

 

Variation in the severity of a given problem is better termed "variable expressivity." Target joints are not genetically determined.

 

Hurler syndrome usually not being present in prior generations of an affected patient is an example of autosomal recessive inheritance.

 

The term pleiotropy refers to a disease taking different shapes in different subjects, whereas the cause of patients with Ollier disease having more involvement on one side of the body is unknown.

Correct Answer: Some patients with Marfan syndrome have scoliosis or pectus carinatum, while other patients with Marfan syndrome do not.

 

 

 

1) Basilar invagination

3) Subaxial subluxation

2) Rotatory subluxation of C 1-C 2

5) Cervical stenosis

4) Small, stiff jaw

 

Stiffness and mandibular hypoplasia are fairly common in juvenile rheumatoid arthritis (JRA) due to inflammation of the temporomandibular joint that affects the growth plates of the mandibles.

 

Basilar invagination is rare in JRA.

 

 

Rotatory subluxation of C 1-C 2 is rare in JRA. Subaxial subluxation is rare in JRA.

 

 

Cervical stenosis is not a clinical problem in JRA. Correct Answer: Small, stiff jaw

 

1799. (684) Q5-943:

A 4-year-old boy is brought to a clinic because he has been fussy, febrile, and unable to bend over for the past 4 days. In the office, his temperature is 38.2° C and his neurologic examination is normal. His lumbar lordosis is flattened and he resists flexion or extension. He has normal range of hip motion. Plain films of the lumbar spine are normal. The next imaging study should be:

 

1) Magnetic resonance imaging of the spine

3) Spinal ultrasound

2) Hip arthrogram

5) Indium labeled white blood cell scan

4) Computed tomograms of the lumbar spine

 

Magnetic resonance imaging should be the next step to rule out pyogenic spondylitis.

 

Ultrasound has not been proven effective in evaluation of anterior spinal pathology.

 

Computed tomograms do not have a greater sensitivity than plain films in early diagnosis of infection.

 

An indium labeled scan may yield diagnostic information but would not be the preferred test because of the time needed and inability to provide other diagnostic information.

 

Hip arthrogram would not be the next step because the hip range of motion is normal. Even if hip pathology were suspected, the next step would be a plain film and an ultrasound.

Correct Answer: Magnetic resonance imaging of the spine

 

 

1800. (685) Q5-944:

A 6-year-old child suffers a displaced fracture of the distal humerus in the supracondylar region. Neurologic and vascular exams are normal. The surgeon decides to reduce and pin the fracture. Which of the following risks increases if the procedure is delayed more than 8 hours?

 

1) Brachial artery damage

3) Radial nerve palsy

2) Median nerve palsy

5) No risks increase

4) Need for an open reduction

 

 

A retrospective comparison study has shown no increase of risks in delayed treatment of supracondylar fractures. Correct Answer: No risks increase

 

 

1) The distal fibula grows more than the distal tibia.

3) The anterolateral portion of the tibial physis ceases growing first.

2) The distal tibia grows more than the distal fibula.

5) The two growth plates are part of a common physis.

4) The two physes should be at an even level.

 

The distal tibia grows more than the distal fibula.

 

 

The anterolateral portion of the tibial physis ceases growing last, thus explaining the phenomenon of the Tillaux fracture. The physis of the distal fibula is always located more distally than the distal tibia.

 

The two physes are not conjoined.

Correct Answer: The distal tibia grows more than the distal fibula.

 

 

1802. (687) Q5-946:

Which of the following is the most common final attribution of back pain in children and adolescents after all appropriate diagnostic studies are performed:

 

1) Spondylolysis

3) Infection

2) Osteoid osteoma

5) No identifiable cause

4) Herniated nucleus pulposus

 

The majority of children and adolescents do not have an identifiable cause of back pain after all appropriate tests are performed. Of the smaller percent of patients with an actual diagnosis, spondylolysis followed by herniated nucleus pulposus are most common.

 

Correct Answer: No identifiable cause

 

 

1803. (688) Q5-947:

Which of the following is the most definitive means of making a diagnosis of active skeletal tuberculosis:

 

1) Positive tuberculin tine test

3) Positive culture and histological exam

2) Negative tuberculin tine test

5) Enzyme linked immunosorbent assay (ELISA) test

4) Magnetic resonance imaging

 

The lower thoracic-upper lumbar spine is most commonly affected by tuberculosis. The most definitive diagnosis is by culture and histologic examination.

Correct Answer: Positive culture and histological exam

 

 

 

1) Disc space is narrowed before significant bony changes occur.

3) Bony erosions seen on computerized tomography are usually small and focal.

2) Involvement of multiple contiguous levels is uncommon.

5) Magnetic resonance imaging rarely shows significant soft tissue swelling.

4) Vertebral destruction exceeds disc destruction.

 

Vertebral destruction exceeds disc destruction in tuberculosis.

 

Bony changes occur earlier in tuberculosis than in pyogenic spondylitis.

 

Involvement of multiple contiguous levels is more common in tuberculosis than pyogenic spondylitis.

 

Bony erosions seen on computerized tomography are large in tuberclosis and small in pyogenic spondylitis. Magnetic resonance imaging often shows significant soft tissue involvement in both disorders.

Correct Answer: Vertebral destruction exceeds disc destruction.

 

 

1805. (690) Q5-949:

A 5-year-old girl comes into the clinic with back pain. Her family has just moved to the United States from southeastern Asia. A lateral radiograph shows destruction of T11, T12, and L1. Magnetic resonance imaging shows a moderate posterior soft tissue mass. A neurological exam is normal. Biopsy confirms tuberculosis. For treatment of the girlâs spinal problem, recommended treatment includes:

 

1) A two-drug therapy for at least 6 months

3) A two-drug therapy and posterior spinal fusion to prevent deformity

2) A two-drug therapy for at least 6 months along with a body cast

5) Anterior debridement, strut graft, and posterior fusion with instrumentation

4) Anterior spinal debridement and a rib strut graft

 

Anterior debridement, strut graft, and posterior fusion with instrumentation provide the patient with the best chance of a positive result. This procedure minimizes graft dislodgement and posterior overgrowth.

 

A two-drug therapy for at least 6 months leaves the patient at a significant risk of progressive kyphosis and neurologic deficit.

 

A two-drug therapy for at least 6 months along with a body cast also leaves the patient with significant risk of progressive kyphosis and neurologic deficit.

 

The lack of anterior support from a two-drug therapy and posterior spinal fusion to prevent deformity leaves the patient with significant risk of kyphosis.

 

Even with an anterior spinal debridement and a rib strut graft, there is a risk of graft dislodgment over this large defect and of posterior growth into kyphosis.

Correct Answer: Anterior debridement, strut graft, and posterior fusion with instrumentation

 

congenital vertical talus:

 

1) The dorsal approach requires a more extensive dissection.

3) The dorsal approach requires plication of the talonavicular capsule.

2) The dorsal approach has a lower risk of avascular necrosis of the talus.

5) The dorsal approach requires a longer tourniquet time.

4) The dorsal approach has a higher risk of redislocation.

 

The dorsal approach has not shown evidence of avascular necrosis, whereas the posteromedial approach has shown such changes at follow-up in as many as 40% of cases.

 

The dorsal approach requires a less extensive dissection than the posteromedial approach.

 

The dorsal approach does not require or permit plication of the talonavicular capsule, whereas the posteromedial approach does.

 

The dorsal approach does not appear to have a higher rate of redislocation of the talonavicular joint than the posteromedial approach.

 

The dorsal approach requires a shorter tourniquet time than the posteromedial approach.

Correct Answer: The dorsal approach has a lower risk of avascular necrosis of the talus.

 

 

1807. (692) Q5-951:

Which of the following conditions is not associated with an increased risk of congenital vertical talus?

 

1) Sacral agenesis

3) Myelomeningocele

2) Cerebral palsy

5) Nail patella syndrome

4) ArthrogryposisV

 

Patients with cerebral palsy do not have an increased risk of congenital vertical talus, but they may develop an acquired neuromuscular vertical talus.

 

Patients with myelomeningocele have approximately a 5% to 10% risk of vertical talus, far above that of the general population.

 

Arthrogryposis is associated with an increased risk of vertical talus.

 

 

Nail patella syndrome is associated with an increased risk of vertical talus. Sacral agenesis is associated with an increased risk of vertical talus.

Correct Answer: Cerebral palsy

 

 

1808. (693) Q5-953:

Scoliosis in Marfan syndrome, as compared to idiopathic scoliosis, is characterized by which of the following:

 

1) Scoliosis curves are more likely to begin in the juvenile period.

3) Brace treatment is more likely to be successful because of the flexibility.

2) There is an increased likelihood of left thoracic curves.

5) Curves are more likely to be stable in adulthood.

4) Patients are less likely to have back pain.

 

Scoliosis curves are much more likely to begin in the juvenile period than idiopathic scoliosis.

 

 

 

There is no significant difference in the likelihood of left thoracic curves in Marfan syndrome. Brace treatment is less likely to be successful in Marfan syndrome than in idiopathic scoliosis. Marfan patients with scoliosis are more likely to have back pain.

 

Marfan curves are more likely to progress in adulthood.

Correct Answer: Scoliosis curves are more likely to begin in the juvenile period.

 

 

 

1) Demineralized bone matrix is weakly osteoinductive.

3) Demineralized bone matrix is not osteoconductive.

2) Demineralized bone matrix is consistent between forms and different sterilization methods.

5) Demineralized bone matrix is strongly osteoinductive.

4) Demineralized bone matrix is osteogenic.

 

Demineralized bone matrix is weakly osteoinductive.

 

The term osteogenic refers to direct transmittal of cells capable of making bone. Demineralized bone matrix is not osteogenic.

 

 

 

Demineralized bone matrix varies in efficacy between different forms and different methods of sterilization. The term osteoconduction refers to provision of a favorable scaffold and environment for bone formation. Demineralized bone matrix is osteoconductive.

 

Correct Answer: Demineralized bone matrix is weakly osteoinductive.

 

 

1810. (695) Q5-955:

A 16-year-old boy with type I Ehlers-Danlos syndrome has a spinal curvature that has progressed 18° in the past year. The curve is a double major type with a Cobb angle of 60° in each curve. There is no associated kyphosis. The following treatment is recommend:

 

1) Observation

3) Anterior fusion and instrumentation

2) Bracing

5) Anterior and posterior fusion with instrumentation

4) Posterior fusion and instrumentation

 

Posterior fusion and instrumentation is the best-documented treatment. Although this form of treatment is followed by an increased incidence of wound healing problems, the problems can be treated.

 

Observation is not recommended because the curve is highly likely to increase and cause a decrease in pulmonary function.

 

Bracing has no role in large curves, and it is not known if bracing is successful at all in Ehlers-Danlos syndrome.

 

Anterior fusion with instrumentation would be difficult with a double curve. Anterior fusion carries an increased risk due to vascular fragility. It is not necessary because there is no increased risk of crankshaft or pseudarthrosis.

 

There is no particular reason for adding an anterior procedure in this situation in view of the vascular risk.

Correct Answer: Posterior fusion and instrumentation

 

 

1811. (696) Q5-956:

Scoliosis in osteogenesis imperfecta is characterized by which of the following:

 

1) Scoliosis which is due primarily to vertebral fractures.

3) Scoliosis is due primarily to associated neurologic problems.

2) Scoliosis is due primarily to ligamentous laxity.

5) Scoliosis rarely impairs quality of life.

4) Scoliosis usually responds to brace treatment.

 

Scoliosis in osteogenesis imperfecta (OI) is due primarily to ligamentous laxity.

 

 

Scoliosis in OI is due primarily to ligamentous laxity, not bony fractures. There is no association between brainstem impression and scoliosis.

 

Scoliosis in OI rarely responds to brace treatment.

 

Scoliosis, when present in OI, is a major impairment of quality of life.

Correct Answer: Scoliosis is due primarily to ligamentous laxity.

 

 

 

1) Penciling of the ribs

3) Widening of the foramen

2) Scalloping of the vertebrae

5) Vertebral rotation

4) Thinning of the transverse processes

 

Vertebral rotation is not a specific characteristic of dystrophic curves. Rotation is more pronounced in dystrophic curves than in nondystrophic curves, but it is commonly present in both types of curves.

 

Penciling of the ribs is one of the features specific for dystrophic curves in neurofibromatosis 1.

 

 

Scalloping of the vertebrae anteriorly and posteriorly is characteristic of dystrophic curves in neurofibromatosis 1. Widening of the neural foramen is specific for dystrophic curves in neurofibromatosis 1. Widening of the neural foramen is due to tumorous masses passing through the foramen.

 

Thinning of the transverse process is a characteristic of dystrophic curves in neurofibromatosis 1.

Correct Answer: Vertebral rotation

 

 

1813. (698) Q5-958:

The spine in familial dysautonomia is characterized by which of the following:

 

1) Rare scoliosis

3) Dense bone

2) Flexible scoliosis

5) Spinal stenosis

4) Increased risk of loss of fixation after surgery

 

There is an increased risk of loss of fixation in familial dysautonomia curves due to decreased bone density and curve rigidity.

 

 

Scoliosis is common in patients with familial dysautonomia and affects up to one-half of patients with the disorder. The curves in familial dysautonomia are rigid, leading to limited correction.

 

The bone density in familial dysautonomia is decreased.

 

Spinal stenosis is not reported in patients with familial dysautonomia.

Correct Answer: Increased risk of loss of fixation after surgery

 

 

1814. (699) Q5-959:

Which of the following problems is most common in achondroplasia:

 

1) Atlantoaxial instability

3) Symptomatic kyphosis

2) Basilar invagination

5) Spondylolisthesis

4) Thoracolumbar stenosis

 

Symptomatic stenosis of the thoracic and lumbar spine is seen in almost half of all achondroplastic patients, although not all patients require surgery.

 

 

Atlantoaxial instability is rare in achondroplasia, although it is not uncommon in other dysplasias. Basilar invagination is not present in achondroplasia.

 

 

Kyphosis is often transient in achondroplasia and rarely persists beyond the second year. Kyphosis is rarely symptomatic. Spondylolisthesis is rare in achondroplasia.

Correct Answer: Thoracolumbar stenosis

 

shows a kyphosis of 35° from C 3 to C 6. A neurologic exam is normal, although she does have stiff joints. The patient is not yet walking. For management of this kyphosis, recommended treatment includes:

 

1) Observation

3) Halo traction

2) Cervicothoracic orthosis

5) Anterior and posterior fusion

4) Posterior fusion

 

Many of these kyphoses will correct spontaneously if the curve does not exceed 50°. In this patient, the inability to walk is most likely due to other skeletal factors.

 

There is no evidence that orthosis will change the natural history of the disorder.

 

There is no need for traction given the high chance of spontaneous resolution and the dangers of traction.

 

 

Posterior fusion is only indicated if the kyphosis is continually progressive, or if neurologic signs or symptoms develop. Anterior and posterior surgery is only indicated in cases with severe pre-existing neurologic deficit.

Correct Answer: Observation

 

 

1816. (701) Q5-962:

A baby born with diastrophic dysplasia today may eventually require all of the following orthopedic procedures during childhood or adulthood except:

 

1) Correction of equinus or varus feet

3) Posterior spinal fusion for scoliosis

2) Arthrodesis from the occiput to the atlas or axis

5) Arthroplasty of the knees

4) Arthroplasty of the hips

 

Patients with diastrophic dysplasia rarely have instability of the upper cervical spine.

 

Babies with diastrophic dysplasia often have rigid equinovarus feet that require surgery to become plantigrade and wear normal shoes.

 

 

A number of patients with diastrophic dysplasia develop progressive scoliosis that requires surgical treatment. Degenerative disease of the hips is common and often requires arthroplasty in early adulthood.

 

Degenerative disease of the knees is common and often requires arthroplasty in early adulthood.

Correct Answer: Arthrodesis from the occiput to the atlas or axis

 

 

1817. (702) Q5-963:

A patient with spondyloepiphyseal dysplasia congenita reaches the age of 5 without being able to walk with a walker. She has five beats of clonus in both ankles. Her reflexes are brisk and her toes are upgoing. The most likely problem that accounts for these conditions is:

 

1) Severe scoliosis of the thoracic spine

3) Lumbar stenosis

2) Foramen magnum stenosis

5) Atlantoaxial instability

4) Thoracolumbar kyphosis

 

Atlantoaxial instability, sometimes combined with stenosis of the atlas, is a frequent cause of myelopathy in spondyloepiphyseal dysplasia congenita.

 

Scoliosis does not account for developmental delay or myelopathy.

 

Foramen magnum stenosis is rare in spondyloepiphyseal dysplasia congenita.

 

 

Lumbar stenosis is rare with spondyloepiphyseal dysplasia congenita and would not account for myelopathy. Thoracolumbar kyphosis severe enough to cause myelopathy is rare in spondyloepiphyseal dysplasia congenita condition.

Correct Answer: Atlantoaxial instability

 

 

 

1) Syringomyelia

3) Spondylolisthesis

2) Atlantoaxial instability

5) Spinal decompensation

4) Spinal stenosis

 

Scoliosis with cleidocranial dysplasia (CCD) is frequently associated with syringomyelia.

 

 

 

Cervical instability is rare in CCD. Spondylolisthesis is rare in CCD. Spinal stenosis is rare in CCD.

 

 

Spinal decompensation is rare in CCD. Correct Answer: Syringomyelia

 

1819. (704) Q5-965:

A 3-year-old girl with Larsen syndrome is brought into the office for examination. A spinal radiograph demonstrates a 50° kyphosis of the cervical spine. Her neurologic examination is normal. Recommended treatment includes:

 

1) Observation

3) Cervical-thoracic orthosis

2) Halo traction

5) Anterior and posterior fusion

4) Posterior cervical fusion

 

Posterior cervical fusion has been proven effective in curves of 60°or less, in preventing progression, and allowing correction with anterior growth.

 

 

This curve is much more likely to worsen than to spontaneously improve, so preventive surgery is indicated. There is no role for halo traction in this situation.

 

Bracing has not been proven effective in helping patients with Larsen syndrome.

 

Addition of anterior fusion is not needed for this degree of curve in a neurologically normal child.

Correct Answer: Posterior cervical fusion

 

 

1820. (705) Q5-966:

A 10-year-old patient with Hurler syndrome has undergone a bone marrow transplant and is currently medically stable. He has developed a painful thoracolumbar kyphosis that measures 50° with 25% subluxation T12 on L1. Recommended treatment includes which of the following:

 

1) Exercise program for the trunk extensor muscles

3) Halo traction followed by orthosis

2) Thoracolumbar orthosis

5) Anterior and posterior spinal fusion

4) Posterior spinal fusion

 

Anterior and posterior fusion will correct the translation, instability, and ensure a solid fusion.

 

 

Exercises will not correct the subluxation, which is the cause of the pain. A thoracolumbar orthosis is not corrective or well tolerated.

 

There is no need for halo traction.

 

Posterior fusion alone is not enough to control this focal instability if the patient is well enough to tolerate a more involved procedure.

Correct Answer: Anterior and posterior spinal fusion

 

 

 

1) Cervical

3) Lower thoracic

2) Upper thoracic

5) Sacral

4) Lumbar

 

Aneurysmal bone cyst of the spine is most common in the lumbar spine, followed by the cervical spine. Aneurysmal bone cyst of the spine is treated with embolization and/or surgical resection and reconstruction.

Correct Answer: Lumbar

 

 

1822. (707) Q5-968:

Aneurysmal bone cyst of the spine is most likely in this age group:

 

1) First decade

3) Third decade

2) Second decade

5) Fifth decade

4) Fourth decade

 

 

The most common age is the second decade; the mean age is 13 years old. Correct Answer: Second decade

 

1823. (708) Q5-969:

A 14-year-old girl is examined because of a pain in her left flank. The radiographs of the lumbar spine show loss of the pedicle with expansion of the lateral wall of the third lumbar vertebral body. Magnetic resonance imaging shows multiple fluid levels in the vertebral body with no additional areas of involvement. She is neurologically normal. The least invasive, effective treatment is which?

 

1) Observation

3) Selective arterial embolization

2) Radiation therapy

5) Curettage plus radiation therapy

4) Radical en bloc resection

 

This patient has an aneurysmal bone cyst of the vertebra. Selective arterial embolization is a minimally invasive treatment that often succeeds in arresting the lesions. Many times it is the only treatment needed. Selective arterial embolization can also be used as part of a strategy to be followed by curettage and reconstruction to decrease operative bleeding.

 

 

 

 

This lesion will continue to expand and might cause neurologic compromise or mechanical instability. Radiation therapy poses risks of later malignant degeneration. There are other ways of treating this lesion. Radical en bloc resection may unnecessarily injure neurologic structures.

 

While curettage is often necessary, there is no reason to introduce the risk of radiation therapy.

Correct Answer: Selective arterial embolization

 

necrosis. Recommended treatment is a hip arthrodesis. In response to questions about late effects, after surgery the patient should be told that she is most likely to experience:

 

1) Low back pain

3) Pain in the contralateral hip

2) Marked limitation of activity

5) Significant continued limp

4) Continued severe pain in the ipsilateral hip

 

Low back pain, followed closely by ipsilateral knee pain, is the most common late effect of hip arthrodesis in young patients. The tolerable pain usually occurs much later but may be treated by conversion to arthroplasty, if needed.

 

Ipsilateral hip pain should be minimal or absent if the fusion is successful.

 

 

Activity following arthrodesis is not significantly limited. Sports and heavy physical activities are feasible. The limp is usually minimal because the loss of hip motion is masked by lumbar motion.

 

Pain in the contralateral hip is rare and is often minimal after hip arthrodesis.

Correct Answer: Low back pain

 

 

1825. (710) Q5-971:

Which of the following is an appropriate position for arthrodesis of the hip in a young person:

 

1) Flexion of 45°

3) Adduction of 0°

2) Abduction of 15° if there is shortening

5) Shortening of at least 3 cm

4) External rotation of 25°

 

 

Neutral abduction is important in preventing back pain. The flexion should be between 25° and 35°.

 

 

Any abduction beyond neutral poses increased risk of back pain. External rotation beyond approximately 5° is not needed.

 

Arthrodesis often produces some shortening; therefore, intentional shortening is not needed.

Correct Answer: Adduction of 0°

 

 

1826. (711) Q5-972:

A 12-year-old girl is brought into the office for an examination because of hip pain. She is able to bear weight on the involved limb while using crutches for stability. Radiographs reveal a grade III slip of the capital femoral epiphysis. Recommended treatment for this patient is:

 

1) Skeletal femoral traction in order to improve the position

3) Osteotomy of the femoral neck to improve the alignment

2) Manipulate the hip under anesthesia in order to improve the position of the head

5) In situ fixation

4) Application of a hip spica

 

 

In situ fixation provides the best results no matter what the grade of slip. Correct Answer: In situ fixation

A 9-year-old boy with cerebral palsy has trouble sitting. His mother states that whenever his diapers are changed or his hips are moved, he begins to cry. Radiographs demonstrate high dislocations of both femoral heads. The femoral heads have an ovoid shape and superolateral flattening. Recommended treatment is:

 

1) Botulinum toxin injected into the adductors

3) Bilateral femoral osteotomies with acetabuloplasty

2) Bilateral open adductor tenotomy

5) Bilateral Colonna arthroplasty

4) Bilateral proximal femoral resection

 

Bilateral proximal femoral resection is the recommended treatment.

 

 

Femoral head dislocations may become painful in cerebral palsy at a much earlier age than in nonspastic individuals. Botulinum toxin or adductor tenotomy will not solve the problem.

 

Replacing the deformed femoral heads into the acetabulum will not achieve the long-term goal of good hip range of motion.

Correct Answer: Bilateral proximal femoral resection

 

 

1828. (713) Q5-974:

A 9-year-old boy is examined due to a closed distal forearm fracture. The radius and ulna are both fractured and translated 100%. After manipulation twice with sedation, the translation cannot be reduced. There is 10-mm shortening of the radius and 5-mm shortening of the ulna. The distal radial angulation on the anteroposterior view is 5° less than normal. The least invasive treatment which would produce acceptable results is:

 

1) Closed reduction in the operating room under general anesthesia

3) Open reduction and percutaneous pin fixation

2) Open reduction and cast application

5) Acceptance of the reduction and maintenance with a cast

4) Open reduction and plate fixation

 

The translation and shortening are not problems and the amount of angulation will easily remodel with this fracture. There is nothing to be gained from operative reduction.

Correct Answer: Acceptance of the reduction and maintenance with a cast

 

 

1829. (714) Q5-975:

When applying a halo for postoperative immobilization in a skeletally mature teenager, which of the following is the proper torque for the pins:

 

1) 2 inch-pounds

3) 8 inch-pounds

2) 4 inch-pounds

5) 12 inch-pounds

4) 10 inch-pounds

 

This patient should be treated like an adult. Eight inch-pounds is the currently recommended torque to provide optimal biomechanical fixation while minimizing pin penetration.

Correct Answer: 8 inch-pounds

 

A posterior spine fusion with segmental hook fixation from T4-L4 is performed for idiopathic scoliosis in a 15-year-old girl. Somatosensory evoked potential monitoring is normal throughout the procedure. The patient awakens and is unable to move either lower extremity, but she does have some sensation in the lower extremities. Recommended treatment includes:

 

1) Removal of instrumentation

3) Laminectomy above the conus medullaris

2) Myelogram

5) Full heparinization of the patient

4) Administration of corticosteroids and observation for 6 hours

 

Spinal cord injury occurs in approximately 1% of patients operated upon for idiopathic scoliosis. In some cases, sensory spinal cord monitoring may be unchanged, especially if the injury preserves the dorsal columns. The instrumentation should be removed as soon as possible in case spinal traction or derotation or implant protrusion is producing effects on the cord or its blood supply.

 

 

 

Corticosteroids should be administered at spinal cord injury doses, but this should not be the only measure. Obtaining a myelogram may delay the removal of instrumentation and should not be the first step.

 

 

Heparinization has no proven effect. Correct Answer: Removal of instrumentation

 

1831. (716) Q5-977:

A 12-year-old boy with achondroplasia has a gradual 40° thoracolumbar kyphosis. He is unable to walk more than two blocks. Magnetic resonance imaging reveals spinal stenosis, and the patient is scheduled to undergo posterior decompression from T12-S1. In addition to this procedure, you recommend:

 

1) Observation with serial radiographs every 4 months

3) In situ fusion with bone graft

2) Postoperative brace for 6 months

5) Anterior corpectomy and fusion of T12

4) Posterior fusion across the kyphosis with instrumentation

 

Extensive posterior decompression poses a high risk of postoperative increase in kyphosis because of both the patientâs age and pre-existing kyphosis.

 

Observation would not be a good idea because the risk is already known to be high.

 

 

Neither a brace nor an uninstrumented fusion would prevent the deformity from developing. Corpectomy is not indicated because the kyphosis is not focal.

 

Posterior instrumented fusion at the time of decompression is indicated.

Correct Answer: Posterior fusion across the kyphosis with instrumentation

 

 

1832. (717) Q5-978:

Which of the following is true regarding brace treatment for Scheuermann kyphosis:

 

1) The Milwaukee brace is not indicated.

3) Bracing is effective in curves over 75°.

2) Permanent improvement is usually obtainable if compliant.

5) The brace should be worn for 1 year after starting brace treatment.

4) Bracing is ineffective in curves having an apex at or above T8.

 

Brace treatment is effective for Scheuermann kyphosis. Unlike idiopathic scoliosis, permanent improvement of the deformity is the goal.

 

The Milwaukee brace is often indicated.

 

 

Brace treatment is ineffective for curves over 74°. The brace should be worn until skeletal maturity.

Correct Answer: Permanent improvement is usually obtainable if compliant.

 

Which of the following statements is true about bone marrow transplantation in mucopolysaccharidoses:

 

1) Bone marrow transplantation is contraindicated.

3) Bone marrow transplantation reverses the orthopedic manifestations.

2) Bone marrow transplantation does not affect the orthopedic problems.

5) Bone marrow transplantation should be deferred until skeletal maturity.

4) Graft-versus-host disease is rare.

 

Bone marrow transplantation is effective in minimizing the deposition of mucopolysaccharides in solid organs. Transplantation should be done early to prevent organ damage. Because the lysosomal enzyme does not cross the cell membrane of osteocartilaginous cells, it does not affect the orthopedic aspects. The risk of graft-versus-host disease is high but may be treated in most cases. Survival rate is 61% at 2 years for Hurler disease, which is otherwise fatal before maturity.

 

Correct Answer: Bone marrow transplantation does not affect the orthopedic problems.

 

 

1834. (796) Q5-1057:

A 7-year-old boy is brought in for an examination due to back pain. He has limited forward bending. Neurologic examination is normal. Radiographs reveal a uniform flattening of the third lumbar vertebra to 10% of its normal height. His temperature is

37.1° C and his white blood count is 11,000. The erythrocyte sedimentation rate is 18. The most likely diagnosis is:

 

1) Osteogenesis imperfecta

3) Tuberculosis

2) Eosinophilic granuloma

5) Compression fracture

4) Bacterial infection

 

Eosinophilic granuloma often produces complete flattening of a single vertebral body in the absence of trauma or neurologic deficit.

 

Osteogenesis imperfecta produces a more uniform flattening of the vertebrae but not by this degree.

 

Tuberculosis rarely produces this much flattening of a single vertebra. The erythrocyte sedimentation rate is elevated in this condition.

 

Bacterial infection rarely produces this much flattening of a single vertebral body without associated disk changes. The lab studies should suggest an inflammatory process.

 

Compression fracture produces less complete flattening of the vertebral bodies.

Correct Answer: Eosinophilic granuloma

 

 

1835. (797) Q5-1058:

A 12-year-old patient with osteogenic sarcoma metastatic to the spine is noted to have new onset of weakness of both lower extremities. Magnetic resonance imaging shows a mass expanding posteriorly and encroaching on the spinal cord. The recommended initial step is:

 

1) Radiation therapy and steroids

3) Surgical resection

2) Increasing the dose of chemotherapy

5) Observation only

4) Steroids and observation alone

 

Radiation therapy combined with steroids should be tried first to try to halt progression of the tumor. Unfortunately, the prognosis for this child is extremely poor.

 

Increasing the dose of chemotherapy is not likely to work because the metastasis has already progressed despite initial treatment.

 

 

Surgical resection must be tried if radiation does not produce improvement. Steroids are an adjunct to treatment but not sufficient alone.

 

The patient is likely to have progressive paraparesis and loss of bowel function. In order to improve the quality of life remaining, surgical resection should be offered to the patient.

Correct Answer: Radiation therapy and steroids

 

The primary purpose of osteotomy in the closure of classic exstrophy of the bladder is to:

 

1) Decrease the tension on the closure of the abdominal wall and bladder

3) Prevent degenerative disease of the hip

2) Decrease the strain on the sacroiliac joints

5) Allow reconstruction of a normal symphysis pubis

4) Normalize the gait

 

The primary purpose of osteotomy is to improve the chance of a successful urologic reconstruction. This is achieved by decreasing the tension on the closure of the abdominal wall and bladder.

 

The strain on the sacroiliac joints has not been measured with or without closure; this is not a primary purpose of the osteotomy.

 

There is no conclusive evidence that the hips are at increased risk of degenerative disease in patients with exstrophy, or that osteotomy will alter the condition.

 

 

In patients with exstrophy, the gait progressively normalizes over time. It is not possible to reconstruct a normal symphysis pubis in exstrophy.

Correct Answer: Decrease the tension on the closure of the abdominal wall and bladder

 

 

1837. (799) Q5-1060:

Which of the following is not a common finding in cloacal exstrophy:

 

1) Omphalocele

3) Hydrocephalus

2) Spinal dysrhaphism

5) Dislocation of the hip(s)

  1. Dysplasia of the sacroiliac joints

     

    Hydrocephalus is rare because most patients have lipomeningocele, not myelomeningocele.

     

    Omphalocele is common in cloacal exstrophy.

     

     

    Most patients with cloacal exstrophy have a lipomeningocele that is a form of spinal dysrhaphism. Many patients have malformations of the sacroiliac joints.

     

    Approximately 25% of patients have dislocations of at least one hip.

    Correct Answer: Hydrocephalus

     

     

    1838. (800) Q5-1061:

    The thickness of a flexible intramedullary nail used in pediatric femur fractures should be which of the following percentages of the diameter of the femoral isthmus:

     

    1) 10%

    3) 40%

    2) 25%

  2. 75%

4) 50%

 

 

It is recommended that the intramedullary nail be 40% of the diameter of the femoral isthmus. Correct Answer: 40%

An infant is born with fibular hemimelia and has 20% shortening of the involved below-knee segment. Four rays are present on the foot, and the ankle is in slight valgus. Limb lengthening is likely to be superior to Syme disarticulation in which of the following parameters:

 

  1. Pain

3) Psychological acceptance

2) Function

5) Prosthetic costs

4) Number of procedures

 

The cost of prosthetics is greater in the disarticulation group.

 

Pain is greater in the lengthened group.

 

 

Function is equal to or better in the group who had disarticulation. Psychological acceptance is greater in the disarticulation group.

 

The lengthened group requires more than twice the number of procedures.

Correct Answer: Prosthetic costs

 

 

1840. (802) Q5-1063:

Which of the following is the best discriminator for risk of nonaccidental (child abuse) injury in young children with femoral shaft fractures:

 

1) Pattern of the fracture

3) Socioeconomic class

2) Level of the fracture on the femur

5) Coexisting disability in the child

4) Ability to walk

 

Nonaccidental injury was a factor in 29% of patients who were unable to walk vs 3% for patients who were able to walk.

 

The fracture pattern and level of fracture do not help determine nonaccidental injury.

 

 

Socioeconomic class is not the best discriminator and, generally, should not be factored into the decision process. Coexisting disability is not a significant discriminator.

Correct Answer: Ability to walk

 

 

1841. (803) Q5-1064:

A 12-year-old boy with hemophilia A has a painless mass in his thigh. The femur is eroded anterolaterally and there is a large overlying soft tissue mass. Magnetic resonance imaging shows a 5 cm x 7 cm mass arising from the bone. The most likely diagnosis is:

 

1) Telangiectatic osteosarcoma

3) Infection

2) Aneurysmal bone cyst

5) Lymphangioma

  1. Pseudotumor

     

    A pseudotumor is a hemophilic subperiosteal hematoma. The pseudotumor expands by repeated bleeds and increasing osmotic pressure.

     

     

    There was no periosteal reaction or intralesional calcification. The bone wall itself is not expanded as in aneurysmal bone cyst.

     

    There is nothing in the physical examination or patient history to point to infection.

    Correct Answer: Pseudotumor

     

    In classic hemophilia, a natural factor-VIII level of less than what percentage will lead to severe bleeding and complications:

     

    1) 50%

    3) 15%

    2) 25%

  2. 5%

4) 10%

 

A surprisingly small amount of circulating factor-VIII (approximately 5%) is necessary to protect a patient from severe bleeding complications.

Correct Answer: 5%

 

 

1843. (805) Q5-1066:

A 1-week-old female infant with arthrogryposis multiplex congenital has hips that are stiff in flexion and abduction, and her knees have a range of flexion from 20° to 40°. In addition, her right thigh has become swollen and tender. The most likely cause of this latter problem is:

 

  1. Osteomyelitis of the femur

3) Dislocation of the hip

2) Septic arthritis of the hip

5) Fracture of the femur

4) Deep vein thrombosis

 

Fracture is common in this condition because of osteopenia and the stress concentration due to joint stiffness.

 

 

Osteomyelitis is uncommon in the diaphysis and much less common in this scenario than fracture. Septic arthritis of the hip is uncommon in this disease.

 

 

Dislocation of the hip would not cause pain and swelling in this setting. Deep vein thrombosis is uncommon at this age.

Correct Answer: Fracture of the femur

 

 

1844. (806) Q5-1067:

Which of the following is a true statement regarding the results of surgery for a contracted joint in arthrogryposis:

 

1) The joint range of motion can easily be doubled.

3) The beginning and end of the range may change, but the total amount of motion remains about the same.

2) The joint cannot be changed.

5) There is not an indication for such surgery.

4) The joint usually becomes stiffer.

 

The beginning and end of the range may change, but the total amount of motion remains about the same.

 

 

 

The amount of the range cannot be significantly increased. The endpoint can change, but not the amount of the range. The joint does not usually become stiffer.

 

There may be an indication for surgery to put the joints in a functional position.

Correct Answer: The beginning and end of the range may change, but the total amount of motion remains about the same.

 

Which of the following statements is true regarding scoliosis in cerebral palsy (CP):

 

1) Scoliosis is most common in hemiplegic CP because of muscle imbalance.

3) Scoliotic curves over 50° are likely to worsen even if the children are mature.

2) A thoracolumbosacral orthosis is usually successful in halting curve progression.

5) The surgical complication rate is lower in CP than idiopathic scoliosis.

4) Surgery for scoliosis will prolong life expectancy.

 

Curves greater than 50° usually progress.

 

 

Scoliosis is most common in totally involved CP patients. Scoliosis is rare in patients with hemiplegia. Braces rarely halt curves in CP.

 

 

Surgery has no proven effect on prolonging life expectancy. The complication rate is higher in CP.

Correct Answer: Scoliotic curves over 50° are likely to worsen even if the children are mature.

 

 

1846. (808) Q5-1069:

A 12-year-old girl is referred because of a positive school scoliosis screen. She has a curve of 16° from T5 to T12, convex to the right. She incidentally also complains of mild back pain over the region of the curve several times per month. Neurologic examination is normal. Recommended treatment includes:

 

1) Magnetic resonance imaging

3) Treatment with a thoracolumbosacral orthosis

2) Technetium bone scintigraphy with SPECT

5) Home exercises and re-examination in follow-up

4) Computed tomography of the thoracic spine

 

Home exercises and re-examination in follow-up is the most appropriate treatment in view of lack of any worrisome features. If this child had severe pain or significant night pain, then further imaging studies would be warranted.

 

 

The magnetic resonance imaging is not indicated in this situation. The bone scan has a low likelihood of being positive.

 

Bracing is not indicated for the curve or the pain.

 

Computer tomography is unlikely to demonstrate any pathology.

Correct Answer: Home exercises and re-examination in follow-up

 

 

1847. (809) Q5-1070:

A 10-year-old boy undergoes biopsy of a spinal cord tumor through a laminectomy of C 7-T2. The most likely complication of this procedure is:

 

1) Progressive cervicothoracic kyphosis

3) Progressive scoliosis

2) Progressive cervicothoracic lordosis

5) Progressive C 7 radiculopathy

4) Degenerative disk disease

 

 

The removal of posterior restraints in the young and growing flexible spine usually leads to cervicothoracic kyphosis. Correct Answer: Progressive cervicothoracic kyphosis

A patient with neurofibromatosis and a 55° scoliosis may be treated with a posterior fusion and instrumentation alone in which of the following situations:

 

1) He has a kyphosis of 75°.

3) He has a prior pseudarthrosis.

2) He is also undergoing multilevel laminectomy for tumor.

5) He has a bone age of 9.

4) He has a kyphosis of 35°.

 

He has a kyphosis of 35°.

 

 

This degree of kyphosis increases the risk of pseudarthrosis with posterior fusion alone. The laminectomy increases the risk of pseudarthrosis.

 

Anterior fusion should be added when there is a history of pseudarthrosis.

 

A 9-year-old boy has a high risk of crankshift phenomenon with posterior fusion alone.

Correct Answer: He has a kyphosis of 35°.

 

 

1849. (811) Q5-1072:

Which of the following statements is true regarding school screening for scoliosis:

 

1) The American Academy of Orthopaedic Surgeons (AAOS) no longer recommends it.

3) The AAOS recommends screening boys and girls at age 9.

2) The AAOS recommends screening each year.

5) The AAOS recommends screening only boys at age 16.

4) The AAOS recommends screening boys and girls at age 11.

 

All children should be screened at age 11.

 

The AAOS still recommends school screening for scoliosis.

 

 

The AAOS recognizes that yearly screening is counterproductive. Screening at age 9 is too early..

 

Screening at age 16 is too late.

Correct Answer: The AAOS recommends screening boys and girls at age 11.

 

 

1850. (812) Q5-1073:

Treatment of a patient with lumbar level myelomeningocele who has a vertical talus should consist of:

 

1) Observation only

3) Achilles tenotomy

2) Talectomy

5) Triple arthrodesis in a reduced position

  1. Open reduction of the vertical talus

     

    Open reduction of the vertical talus will most likely prevent problems.

     

     

    With observation only, the patient is likely to stand or walk and develop pressure problems. Talectomy will not produce the most usable foot.

     

     

    Achilles tenotomy will not produce significant correction by itself. Triple arthrodesis will concentrate stress and lead to ulcers.

    Correct Answer: Open reduction of the vertical talus

     

    What percentage of the human genome represents the actual genes:

     

    1) 80%

    3) 50%

    2) 66%

  2. 5%

4) 20%

 

The percentage of the genome that represents the sequence of our genes is approximately 5%. The rest of the genome codes are for initiator and termination sequences, maintenance functions, and unknown functions.

Correct Answer: 5%

 

 

1852. (814) Q5-1075:

In studying a newly recognized disorder using a large population of affected individuals, geneticists discover that although the disorder often affects siblings, it is rarely found in any of their ancestors. This disorder most closely follows which pattern of inheritance:

 

  1. Autosomal dominant

3) Sex-linked

2) Autosomal recessive

5) Anticipation

4) Multifactorial

 

Autosomal recessive conditions classically show âhorizontalâ inheritance. Ancestors do not display the gene because they would likely have only one copy of the mutant allele. Only when two carriers reproduce is the phenotype manifest in approximately one-fourth of their offspring.

 

 

Autosomal dominant inheritance is characterized by vertical transmission. Many generations manifest the trait because it takes only a single copy of a mutant allele to display the phenotype.

 

 

Sex-linked conditions are often traced back in a family. Normally the males are affected and the females are carriers. Multifactorial conditions are thought to result from the combination of different genes. Although the risk of recurrence in kindred is somewhat greater than the population as a whole, it is still quite low (only a few percent). It is rare for siblings to be affected.

 

Anticipation refers to the phenomenon in which successive generations are likely to display more severe forms of a given disorder. Myotonic dystrophy is a classic example of this phenomenon.

Correct Answer: Autosomal recessive

 

 

1853. (815) Q5-1076:

Diseases caused by enzyme deficiency are commonly inherited by which of the following patterns:

 

1) Autosomal dominant

3) X-linked dominant

2) Autosomal recessive

5) Non-mendelian

4) Multifactorial

 

Two copies of a mutant allele are required to reduce enzyme function to levels that cause clinical impairment.

 

Enzyme defects are rarely inherited by an autosomal dominant pattern because even half of the normal activity of most enzymes is adequate to maintain normal function.

 

Enzyme defects are rarely inherited in an X-linked dominant pattern because one copy of a mutant allele is usually sufficient.

 

Multifactorial inheritance refers to the interaction of multiple, or different genes, to produce a disorder. Enzyme deficiencies are typically the result of a defect in a single gene.

 

Because enzymes are typically coded by a single gene, they follow mendelian patterns.

Correct Answer: Autosomal recessive

 

Morquio syndrome is caused by a deficiency in:

 

1) Alpha-L-iduronidase

3) Beta-glucuronidase

2) Galactose-6-sulfatase

5) Sulfate transport protein

4) Fibroblast growth factor receptor protein

 

Morquio syndrome is a member of the family of mucopolysaccharidoses. Morquio syndrome is a deficiency in the enzyme galactose-6-sulfatase. A deficiency in galactose-6-sulfatase results in increased urinary excretion of keratosulfate.

 

Alpha-L-iduronidase is deficient in Hurler syndrome.

 

 

 

Beta-glucuronidase is deficient in some rare mucopolysaccharidoses. Fibroblast growth factor receptor protein is deficient in achondroplasia. Sulfate transport protein is deficient in diastrophic dysplasia.

 

Correct Answer: Galactose-6-sulfatase

 

 

1855. (817) Q5-1078:

Polymerase chain reaction (PCR) is best characterized by which of the following descriptions:

 

1) Use of enzymes to link chains of deoxyribonucleic acid (DNA) together

3) Denaturing and reannealing DNA multiple times with known primers

2) Use of viral vectors to insert new DNA into a cell

5) The process by which a cell-surface receptor turns on the transcription process

4) Use of high temperatures to create ultra-high molecular weight polyethylene

 

Polymerase chain reaction refers to denaturing DNA, isolating a segment of interest with known primers, and reannealing the strands multiple times to produce exponential copies of a segment.

Correct Answer: Denaturing and reannealing DNA multiple times with known primers

 

 

1856. (818) Q5-1079:

Pleiotropy is demonstrated by which of the following examples:

 

1) Patients with osteogenesis imperfecta differ in the number of fractures they have received.

3) Hurler syndrome is usually not present in prior generations of an affected patient.

2) Patients with hemophilia A have different target joints.

5) Patients with Ollier disease often have more involvement on one side of the body.

4) Some patients with Marfan syndrome have scoliosis or pectus carinatum, while other patients with Marfan syndrome do not.

 

The term pleiotropy refers to a disease taking different shapes in various patients.

 

 

Variation in the severity of a given problem is better termed "variable expressivity." Target joints are not genetically determined.

 

Hurler syndrome usually not being present in prior generations of an affected patient is an example of autosomal recessive inheritance.

 

The term pleiotropy refers to a disease taking different shapes in different subjects, whereas the cause of patients with Ollier disease having more involvement on one side of the body is unknown.

Correct Answer: Some patients with Marfan syndrome have scoliosis or pectus carinatum, while other patients with Marfan syndrome do not.

 

 

 

1) Basilar invagination

3) Subaxial subluxation

2) Rotatory subluxation of C 1-C 2

5) Cervical stenosis

4) Small, stiff jaw

 

Stiffness and mandibular hypoplasia are fairly common in juvenile rheumatoid arthritis (JRA) due to inflammation of the temporomandibular joint that affects the growth plates of the mandibles.

 

Basilar invagination is rare in JRA.

 

 

Rotatory subluxation of C 1-C 2 is rare in JRA. Subaxial subluxation is rare in JRA.

 

 

Cervical stenosis is not a clinical problem in JRA. Correct Answer: Small, stiff jaw

 

1858. (820) Q5-1081:

A 4-year-old boy is brought to a clinic because he has been fussy, febrile, and unable to bend over for the past 4 days. In the office, his temperature is 38.2° C and his neurologic examination is normal. His lumbar lordosis is flattened and he resists flexion or extension. He has normal range of hip motion. Plain films of the lumbar spine are normal. The next imaging study should be:

 

1) Magnetic resonance imaging of the spine

3) Spinal ultrasound

2) Hip arthrogram

5) Indium labeled white blood cell scan

4) Computed tomograms of the lumbar spine

 

Magnetic resonance imaging should be the next step to rule out pyogenic spondylitis.

 

Ultrasound has not been proven effective in evaluation of anterior spinal pathology.

 

Computed tomograms do not have a greater sensitivity than plain films in early diagnosis of infection.

 

An indium labeled scan may yield diagnostic information but would not be the preferred test because of the time needed and inability to provide other diagnostic information.

 

Hip arthrogram would not be the next step because the hip range of motion is normal. Even if hip pathology were suspected, the next step would be a plain film and an ultrasound.

Correct Answer: Magnetic resonance imaging of the spine

 

 

1859. (821) Q5-1082:

A 6-year-old child suffers a displaced fracture of the distal humerus in the supracondylar region. The surgeon decides to reduce and pin the fracture. Which of the following risks increases if the procedure is delayed more than 8 hours?

 

1) Brachial artery damage

3) Radial nerve palsy

2) Median nerve palsy

5) No risks increase

4) Need for an open reduction

 

A retrospective comparison study has shown no increase of risks in delayed treatment of supracondylar fractures as long as the neurovascular examination is within normal limits.

Correct Answer: No risks increase

 

 

 

1) The distal fibula grows more than the distal tibia.

3) The anterolateral portion of the tibial physis ceases growing first.

2) The distal tibia grows more than the distal fibula.

5) The two growth plates are part of a common physis.

4) The two physes should be at an even level.

 

The distal tibia grows more than the distal fibula.

 

 

The anterolateral portion of the tibial physis ceases growing last, thus explaining the phenomenon of the Tillaux fracture. The physis of the distal fibula is always located more distally than the distal tibia.

 

The two physes are not conjoined.

Correct Answer: The distal tibia grows more than the distal fibula.

 

 

1861. (823) Q5-1084:

Which of the following is the most common final attribution of back pain in children and adolescents after all appropriate diagnostic studies are performed:

 

1) Spondylolysis

3) Infection

2) Osteoid osteoma

5) No identifiable cause

4) Herniated nucleus pulposus

 

The majority of children and adolescents do not have an identifiable cause of back pain after all appropriate tests are performed. Many times the diagnosis is "musculo-ligamentous strain." the most common identified cause is spondylolysis.

Correct Answer: No identifiable cause

 

 

1862. (824) Q5-1085:

Which of the following is the most definitive means of making a diagnosis of active skeletal tuberculosis:

 

1) Positive tuberculin tine test

3) Positive culture and histological exam

2) Negative tuberculin tine test

5) Enzyme linked immunosorbent assay (ELISA) test

4) Magnetic resonance imaging

 

An early histology confirmed later by a culture is considered the definitive means of diagnosis for active skeletal tuberculsosis.

 

 

The tuberculin tine tests do not indicate active disease, only exposure. Magnetic resonance imaging is not specific for a particular infectious organism.

 

The enzyme linked immunosorbent assay (ELISA) test is used to diagnose Lyme disease.

Correct Answer: Positive culture and histological exam

 

 

 

1) Cervical

3) Lower thoracic-upper lumbar

2) Upper thoracic

5) Cacral

4) Lower lumbar

 

The lower thoracic-upper lumbar spine is most commonly affected by tuberculosis. Multiple vertebrae are often involved in tuberculosis spondylitis.

Correct Answer: Lower thoracic-upper lumbar

 

 

1864. (826) Q5-1087:

Which of the following descriptions is more characteristic of tuberculosis than pyogenic spondylitis:

 

1) Disc space is narrowed before significant bony changes occur.

3) Bony erosions seen on computerized tomography are usually small and focal.

2) Involvement of multiple contiguous levels is uncommon.

5) Magnetic resonance imaging rarely shows significant soft tissue swelling.

4) Vertebral destruction exceeds disc destruction.

 

Vertebral destruction exceeds disc destruction in tuberculosis.

 

Bony changes occur earlier in tuberculosis than in pyogenic spondylitis.

 

Involvement of multiple contiguous levels is more common in tuberculosis than pyogenic spondylitis.

 

 

Bony erosions seen on computerized tomography are large in tuberclosis and small in pyogenic spondylitis. Magnetic resonance imaging often shows significant soft tissue involvement in both disorders.

Correct Answer: Vertebral destruction exceeds disc destruction.

 

 

1865. (827) Q5-1088:

A 5-year-old girl comes into the clinic with back pain. Her family has just moved to the United States from southeastern Asia. A lateral radiograph shows destruction of T11, T12, and L1. Magnetic resonance imaging shows a moderate posterior soft tissue mass. A neurological exam is normal. Biopsy confirms tuberculosis. For the girlâs spinal problem, recommended treatment includes:

 

1) A two-drug therapy for at least 6 months

3) A two-drug therapy and posterior spinal fusion to prevent deformity

2) A two-drug therapy for at least 6 months along with a body cast

5) Anterior debridement, strut graft, and posterior fusion with instrumentation

4) Anterior spinal debridement and a rib strut graft

 

Anterior debridement, strut graft, and posterior fusion with instrumentation provide the patient with the best chance of a positive result. This procedure minimizes graft dislodgement and posterior overgrowth.

 

A two-drug therapy alone for at least 6 months leaves the patient at a significant risk of progressive kyphosis and neurologic deficit.

 

A two-drug therapy for at least 6 months along with a body cast also leaves the patient with significant risk of progressive kyphosis and neurologic deficit.

 

The lack of anterior support from a two-drug therapy and posterior spinal fusion to prevent deformity leaves the patient with significant risk of kyphosis.

 

Even with an anterior spinal debridement and a rib strut graft, there is a risk of graft dislodgment over this large defect and of posterior growth into kyphosis.

Correct Answer: Anterior debridement, strut graft, and posterior fusion with instrumentation

 

congenital vertical talus:

 

1) The dorsal approach requires a more extensive dissection.

3) The dorsal approach requires plication of the talonavicular capsule.

2) The dorsal approach has a lower risk of avascular necrosis of the talus.

5) The dorsal approach requires a longer tourniquet time.

4) The dorsal approach has a higher risk of redislocation.

 

The dorsal approach has not shown evidence of avascular necrosis, whereas the posteromedial approach has shown such changes at follow-up in as many as 40% of cases.

 

The dorsal approach requires a less extensive dissection than the posteromedial approach.

 

The dorsal approach does not require or permit plication of the talonavicular capsule, whereas the posteromedial approach does.

 

The dorsal approach does not appear to have a higher rate of redislocation of the talonavicular joint than the posteromedial approach.

 

The dorsal approach requires a shorter tourniquet time than the posteromedial approach.

Correct Answer: The dorsal approach has a lower risk of avascular necrosis of the talus.

 

 

1867. (829) Q5-1090:

Which of the following conditions is not associated with an increased incidence of congenital vertical talus:

 

1) Cerebral palsy

3) Arthrogryposis

2) Myelomeningocele

5) Larsen syndrome

4) Nail patella syndrome

 

Patients with cerebral palsy do not have an increased risk of congenital vertical talus, but they may develop an acquired neuromuscular vertical talus.

 

Patients with myelomeningocele have approximately a 5% to 10% risk of vertical talus, far above that of the general population.

 

Arthrogryposis is associated with an increased risk of vertical talus.

 

Nail patella syndrome and Larsen syndrome are associated with an increased risk of vertical talus.

Correct Answer: Cerebral palsy

 

 

1868. (830) Q5-1092:

Scoliosis in Marfan syndrome is characterized by which of the following:

 

1) Scoliosis curves are more likely to begin in the juvenile period.

3) Brace treatment is more likely to be successful because of the flexibility.

2) There is an increased likelihood of left thoracic curves.

5) Curves are more likely to be stable in adulthood.

4) Patients are less likely to have back pain.

 

Scoliosis curves are much more likely to begin in the juvenile period than idiopathic scoliosis.

 

 

 

There is no significant difference in the likelihood of left thoracic curves in Marfan syndrome. Brace treatment is less likely to be successful in Marfan syndrome than in idiopathic scoliosis. Marfan patients with scoliosis are more likely to have back pain.

 

Marfan curves are more likely to progress in adulthood.

Correct Answer: Scoliosis curves are more likely to begin in the juvenile period.

 

 

 

1) Demineralized bone matrix is osteogenic.

3) Demineralized bone matrix is weakly osteoinductive.

2) Demineralized bone matrix is strongly osteoinductive.

5) Demineralized bone matrix is not osteoconductive.

4) Demineralized bone matrix is consistent between forms and different sterilization methods.

 

Demineralized bone matrix is weakly osteoinductive.

 

The term osteogenic refers to direct transmittal of cells capable of making bone. Demineralized bone matrix is not osteogenic.

 

 

 

Demineralized bone matrix varies in efficacy between different forms and different methods of sterilization. The term osteoconduction refers to provision of a favorable scaffold and environment for bone formation. Demineralized bone matrix is osteoconductive.

 

Correct Answer: Demineralized bone matrix is weakly osteoinductive.

 

 

1870. (832) Q5-1094:

A 16-year-old boy with type I Ehlers-Danlos syndrome has a spinal curvature that has progressed 18° in the past year. The curve is a double major type with a Cobb angle of 60° in each curve. There is no associated kyphosis. The following treatment is recommend:

 

1) Observation

3) Anterior fusion and instrumentation

2) Bracing

5) Anterior and posterior fusion with instrumentation

4) Posterior fusion and instrumentation

 

Posterior fusion and instrumentation is the best-documented treatment. Although this form of treatment is followed by an increased incidence of wound healing problems, the problems can be treated.

 

Observation is not recommended because the curve is highly likely to increase and cause a decrease in pulmonary function.

 

Bracing has no role in large curves, and it is not known if bracing is successful in Ehlers-Danlos syndrome.

 

Anterior fusion with instrumentation would be difficult with a double curve. Anterior fusion carries an increased risk due to vascular fragility. It is not necessary because there is no increased risk of crankshaft or pseudarthrosis.

 

There is no particular reason for adding an anterior procedure in this situation in view of the vascular risk.

Correct Answer: Posterior fusion and instrumentation

 

 

1871. (833) Q5-1095:

Scoliosis in osteogenesis imperfecta is characterized by which of the following:

 

1) Scoliosis is due primarily to vertebral fractures.

3) Scoliosis is due primarily to associated neurologic problems.

2) Scoliosis is due primarily to ligamentous laxity.

5) Scoliosis rarely impairs quality of life.

4) Scoliosis usually responds to brace treatment.

 

Scoliosis in osteogenesis imperfecta (OI) is due primarily to ligamentous laxity.

 

 

Scoliosis in OI is due primarily to ligamentous laxity, not bony fractures. There is no association between brainstem impression and scoliosis.

 

Scoliosis in OI rarely responds to brace treatment.

 

Scoliosis, when present in OI, is a major impairment of quality of life.

Correct Answer: Scoliosis is due primarily to ligamentous laxity.

 

 

 

1) Penciling of the ribs

3) Widening of the foramen

2) Scalloping of the vertebrae

5) Right convex apex

4) Thinning of the transverse processes

 

Right convex apex is not a specific characteristic of dystrophic curves. It is more common in dystrophic curves than in nondystrophic curves, but it is commonly present in both types of curves.

 

Penciling of the ribs is one of the features specific for dystrophic curves in neurofibromatosis 1.

 

 

Scalloping of the vertebrae anteriorly and posteriorly is characteristic of dystrophic curves in neurofibromatosis 1. Widening of the neural foramen is specific for dystrophic curves in neurofibromatosis 1. Widening of the neural foramen is due to tumorous masses passing through the foramen.

 

Thinning of the transverse process is a characteristic of dystrophic curves in neurofibromatosis 1.

Correct Answer: Right convex apex

 

 

1873. (835) Q5-1097:

The spine in familial dysautonomia is characterized by which of the following:

 

1) Rare scoliosis

3) Dense bone

2) Flexible scoliosis

5) Spinal stenosis

4) Increased risk of loss of fixation after surgery

 

There is an increased risk of loss of fixation in familial dysautonomia curves due to decreased bone density and curve rigidity.

 

 

Scoliosis is common in patients with familial dysautonomia and affects up to one-half of patients with the disorder. The curves in familial dysautonomia are rigid, leading to limited correction.

 

The bone density in familial dysautonomia is decreased.

 

Spinal stenosis is not reported in patients with familial dysautonomia.

Correct Answer: Increased risk of loss of fixation after surgery

 

 

1874. (836) Q5-1098:

Which of the following spinal problems is most common in achondroplasia:

 

1) Atlantoaxial instability

3) Symptomatic kyphosis

2) Basilar invagination

5) Spondylolisthesis

4) Thoracolumbar stenosis

 

Symptomatic stenosis of the thoracic and lumbar spine is seen in almost half of all achondroplastic patients, although not all patients require surgery.

 

 

Atlantoaxial instability is rare in achondroplasia, although it is not uncommon in other dysplasias. Basilar invagination is not present in achondroplasia.

 

 

Kyphosis is often transient in achondroplasia and rarely persists beyond the second year. Kyphosis is rarely symptomatic. Spondylolisthesis is rare in achondroplasia.

Correct Answer: Thoracolumbar stenosis

 

shows a kyphosis of 35° from C 3 to C 6. A neurologic exam is normal, although she does have stiff joints. The patient is not yet walking. For management of this kyphosis, recommended treatment includes:

 

1) Observation

3) Halo traction

2) Cervicothoracic orthosis

5) Anterior and posterior fusion

4) Posterior fusion

 

Many diastrophic cervical kyphoses will correct spontaneously if the curve does not exceed 50°. In this patient, the lack of walking is most likely due to other skeletal factors.

 

There is no evidence that orthosis will change the natural history of the disorder.

 

There is no need for traction given the high chance of spontaneous resolution and the dangers of traction.

 

 

Posterior fusion is only indicated if the kyphosis is continually progressive, or if neurologic signs or symptoms develop. Anterior and posterior surgery is only indicated in cases with severe pre-existing neurologic deficit.

Correct Answer: Observation

 

 

1876. (838) Q5-1100:

A baby born with diastrophic dysplasia today may eventually require all of the following orthopedic procedures during childhood or adulthood except:

 

1) Correction of equinus or varus feet

3) Posterior spinal fusion for scoliosis

2) Arthrodesis from the occiput to the atlas or axis

5) Arthroplasty of the knees

4) Arthroplasty of the hips

 

Patients with diastrophic dysplasia rarely have instability of the upper cervical spine.

 

Babies with diastrophic dysplasia often have rigid equinovarus feet that require surgery to become plantigrade and wear normal shoes.

 

 

A number of patients with diastrophic dysplasia develop progressive scoliosis that requires surgical treatment. Degenerative disease of the hips is common and often requires arthroplasty in early adulthood.

 

Degenerative disease of the knees is common and often requires arthroplasty in early adulthood.

Correct Answer: Arthrodesis from the occiput to the atlas or axis

 

 

1877. (839) Q5-1101:

A patient with spondyloepiphyseal dysplasia congenita reaches the age of 5 without being able to walk with a walker. She has five beats of clonus in both ankles. Her reflexes are brisk and her toes are upgoing. The most likely problem that accounts for these findings is:

 

1) Severe scoliosis of the thoracic spine

3) Lumbar stenosis

2) Foramen magnum stenosis

5) Atlantoaxial instability

4) Thoracolumbar kyphosis

 

Atlantoaxial instability, sometimes combined with stenosis of the atlas, is a frequent cause of myelopathy in spondyloepiphyseal dysplasia congenita.

 

Scoliosis does not account for developmental delay or myelopathy.

 

Foramen magnum stenosis is rare in spondyloepiphyseal dysplasia congenita.

 

 

Lumbar stenosis is rare with spondyloepiphyseal dysplasia congenita and would not account for myelopathy. Thoracolumbar kyphosis severe enough to cause myelopathy is rare in spondyloepiphyseal dysplasia congenita condition.

Correct Answer: Atlantoaxial instability

 

 

 

1) Syringomyelia

3) Spondylolisthesis

2) Atlantoaxial instability

5) Spinal decompensation

4) Spinal stenosis

 

Scoliosis with cleidocranial dysplasia (CCD) is frequently associated with syringomyelia.

 

 

 

Cervical instability is rare in CCD. Spondylolisthesis is rare in CCD. Spinal stenosis is rare in CCD.

 

 

Spinal decompensation is rare in CCD. Correct Answer: Syringomyelia

 

1879. (841) Q5-1103:

A 3-year-old girl with Larsen syndrome is brought into the office for examination. A spinal radiograph demonstrates a 50° kyphosis of the cervical spine. Her neurologic examination is normal. Recommended treatment includes:

 

1) Observation

3) Cervical-thoracic orthosis

2) Halo traction

5) Anterior and posterior fusion

4) Posterior cervical fusion

 

Posterior cervical fusion has been proven effective in curves of 60°or less in Larsen syndrome, in preventing progression, and allowing correction with anterior growth.

 

 

This curve is much more likely to worsen than to spontaneously improve, so preventive surgery is indicated. There is no role for halo traction in this situation.

 

Bracing has not been proven effective in helping patients with Larsen syndrome.

 

Addition of anterior fusion is not needed for this degree of curve in a neurologically normal child.

Correct Answer: Posterior cervical fusion

 

 

1880. (842) Q5-1104:

A 10-year-old patient with Hurler syndrome has undergone a bone marrow transplant and is currently medically stable. He has developed a painful thoracolumbar kyphosis that measures 50° with 25% subluxation T12 on L1. Recommended treatment includes which of the following:

 

1) Exercise program for the trunk extensor muscles

3) Halo traction followed by orthosis

2) Thoracolumbar orthosis

5) Anterior and posterior spinal fusion

4) Anterior spinal fusion

 

Anterior and posterior fusion will correct the translation, instability, and ensure a solid fusion. In some cases, posterior fusion alone will suffice if pedicle fixation is good.

 

 

Exercises will not correct the subluxation, which is the cause of the pain. A thoracolumbar orthosis is not corrective or well tolerated.

 

There is no need for halo traction.

 

Anterior fusion alone is not enough to control this focal instability if the patient is well enough to tolerate a more involved procedure.

Correct Answer: Anterior and posterior spinal fusion

 

 

 

1) Cervical

3) Lower thoracic

2) Upper thoracic

5) Sacral

4) Lumbar

 

Aneurysmal bone cyst of the spine is most common in the lumbar spine, followed by the cervical spine. Aneurysmal bone cyst most commonly involves the anterior elements but later may expand into the posterior elements.

Correct Answer: Lumbar

 

 

1882. (844) Q5-1106:

Aneurysmal bone cyst of the spine is most likely in this age group:

 

1) First decade

3) Third decade

2) Second decade

5) Fifth decade

4) Fourth decade

 

 

The most common age is the second decade; the mean age is 13 years old for patients with this disorder. Correct Answer: Second decade

 

1883. (845) Q5-1107:

A 14-year-old girl is examined because of a pain in her left flank. The radiographs of the lumbar spine show loss of the pedicle with expansion of the lateral wall of the third lumbar vertebral body. Magnetic resonance imaging shows multiple fluid levels with no additional areas of involvement. She is neurologically normal. Recommended treatment includes:

 

1) Observation

3) Selective arterial embolization

2) Radiation therapy

5) Curettage plus radiation therapy

4) Radical en bloc resection

 

This patient has an aneurysmal bone cyst. Selective arterial embolization is a minimally invasive treatment that often succeeds in arresting the lesions. Many times it is the only treatment needed. Selective arterial embolization can also be used as part of a strategy to be followed by curettage and reconstruction to decrease operative bleeding.

 

 

 

 

This lesion will continue to expand and might cause neurologic compromise or mechanical instability. Radiation therapy poses risks of later malignant degeneration. There are other ways of treating this lesion. Radical en bloc resection may unnecessarily injure neurologic structures.

 

While curettage is often necessary, there is no reason to introduce the risk of radiation therapy.

Correct Answer: Selective arterial embolization

 

Recommended treatment is a hip arthrodesis. In response to questions about late effects, after surgery the patient should be told that she is most likely to experience:

 

1) Low back pain

3) Marked limitation of activity

2) Continued severe pain in the ipsilateral hip

5) Pain in the contralateral hip

4) Significant continued limp

 

Low back pain, followed closely by ipsilateral knee pain, is the most common late effect of hip arthrodesis in young patients. The tolerable pain usually occurs much later but may be treated by conversion to arthroplasty, if needed.

 

Ipsilateral hip pain should be minimal or absent if the fusion is successful.

 

 

Activity following arthrodesis is not significantly limited. Sports and heavy physical activities are feasible. The limp is usually minimal because the loss of hip motion is masked by lumbar motion.

 

Pain in the contralateral hip is rare and is often minimal after hip arthrodesis.

Correct Answer: Low back pain

 

 

1885. (847) Q5-1109:

Which of the following is a parameter that should be used in positioning arthrodesis of the hip in a young person:

 

1) Flexion of 45°

3) Adduction of 0°

2) Abduction of 15° if there is shortening

5) Shortening of at least 3 cm

4) External rotation of 25°

 

 

Neutral abduction is important in preventing back pain. The flexion should be between 25° and 35°.

 

 

Any abduction beyond neutral poses increases risk of back pain. External rotation beyond approximately 5° is not needed.

 

Arthrodesis often produces some mandatory shortening; therefore, intentional shortening is not needed.

Correct Answer: Adduction of 0°

 

 

1886. (848) Q5-1111:

A 9-year-old boy with cerebral palsy has trouble sitting. His mother states that whenever his diapers are changed or his hips are moved, he begins to cry. Radiographs demonstrate high dislocations of both femoral heads. The femoral heads have an ovoid shape and superolateral flattening. Recommended treatment includes:

 

1) Botulinum toxin injected into the adductors

3) Bilateral femoral osteotomies with acetabuloplasty

2) Bilateral open adductor tenotomy

5) Bilateral Colonna arthroplasty

4) Bilateral proximal femoral resection

 

Bilateral proximal femoral resection is the recommended treatment.

 

 

Femoral head dislocations may become painful in cerebral palsy at a much earlier age than in nonspastic individuals. Botulinum toxin or adductor tenotomy will not solve the problem.

 

Replacing the deformed femoral heads into the acetabulum will not achieve the long-term goal of good hip range of motion.

Correct Answer: Bilateral proximal femoral resection

 

A 9-year-old boy is examined due to a closed distal forearm fracture. The radius and ulna are both fractured and translated 100%. After manipulation twice with sedation, the translation cannot be reduced. There is 10-mm shortening of the radius and 5-mm shortening of the ulna. The distal radial angulation on the anteroposterior view is 5° less than normal. The next step in treatment should include:

 

1) Closed reduction in the operating room under general anesthesia

3) Open reduction and percutaneous pin fixation

2) Open reduction and cast application

5) Acceptance of the reduction and maintenance with a cast

4) Open reduction and plate fixation

 

The translation and shortening are not problems and the amount of angulation will easily remodel with this fracture. There is nothing to be gained from operative reduction.

Correct Answer: Acceptance of the reduction and maintenance with a cast

 

 

1888. (850) Q5-1113:

When applying a halo for postoperative immobilization in a skeletally mature teenager, which of the following is the proper torque for the pins:

 

1) 2 inch-pounds

3) 8 inch-pounds

2) 4 inch-pounds

5) 12 inch-pounds

4) 10 inch-pounds

 

Eight inch-pounds is the currently recommended torque to provide optimal biomechanical fixation while minimizing pin penetration. This is also recommended for adults. In young children, 4- to 6-inch-pounds are preferred.

Correct Answer: 8 inch-pounds

 

 

1889. (851) Q5-1114:

A posterior spine fusion with segmental hook fixation from T4-L4 is performed for idiopathic scoliosis in a 15-year-old girl. Somatosensory evoked potential monitoring is normal throughout the procedure. The patient awakens and is unable to move either lower extremity, but she does have some sensation in the lower extremities. Recommended treatment includes:

 

1) Removal of instrumentation

3) Laminectomy above the conus medullaris

2) Myelogram

5) Full heparinization of the patient

4) Administration of corticosteroids and observation for 6 hours

 

Spinal cord injury occurs in approximately 0.1% of patients operated upon for idiopathic scoliosis. In some cases, sensory spinal cord monitoring may be unchanged, especially if the injury preserves the dorsal columns. The instrumentation should be removed as soon as possible in case spinal traction or derotation or implant protrusion is producing effects on the cord or its blood supply.

 

 

 

Corticosteroids should be administered at spinal cord injury doses, but this should not be the only measure. Obtaining a myelogram may delay the removal of instrumentation and should not be the first step.

 

 

Heparinization has no proven effect. Correct Answer: Removal of instrumentation

A 12-year-old boy with achondroplasia has a gradual 40° thoracolumbar kyphosis. He is unable to walk more than two blocks. Magnetic resonance imaging reveals spinal stenosis, and the patient is scheduled to undergo posterior decompression from T12-S1. In addition to this procedure, you recommend:

 

1) Observation with serial radiographs every 4 months

3) In situ fusion with bone graft

2) Postoperative brace for 6 months

5) Anterior corpectomy and fusion of T12

4) Posterior fusion across the kyphosis with instrumentation

 

Extensive posterior decompression poses a high risk of postoperative increase in kyphosis because of both the patientâs age and pre-existing kyphosis.

 

Observation would not be a good idea because the risk is already known to be high.

 

 

Neither a brace nor an uninstrumented fusion would prevent the deformity from developing. Corpectomy is not indicated because the kyphosis is not focal.

 

Posterior instrumented fusion at the time of decompression is indicated.

Correct Answer: Posterior fusion across the kyphosis with instrumentation

 

 

1891. (853) Q5-1116:

Which of the following is true regarding brace treatment for Scheuermann kyphosis:

 

1) The Milwaukee brace is not indicated.

3) Bracing is effective in curves over 75°.

2) Permanent correction is usually obtainable.

5) The brace should be worn for 1 year after starting brace treatment.

4) Bracing is ineffective in curves having an apex at or above T8.

 

Brace treatment is effective for Scheuermann kyphosis. Unlike idiopathic scoliosis, permanent correction of the deformity is the goal.

 

The Milwaukee brace is often indicated.

 

 

Brace treatment is ineffective for curves over 74°. The brace should be worn until skeletal maturity.

Correct Answer: Permanent correction is usually obtainable.

 

 

1892. (854) Q5-1117:

Which of the following statements is true about bone marrow transplantation in mucopolysaccharidoses:

 

1) Bone marrow transplantation is contraindicated.

3) Bone marrow transplantation reverses the orthopedic manifestations.

2) Bone marrow transplantation does not affect the orthopedic problems.

5) Bone marrow transplantation should be deferred until skeletal maturity.

4) Graft-versus-host disease is rare.

 

Bone marrow transplantation is effective in preventing the deposition of mucopolysaccharides in solid organs. Transplantation should be done early to prevent organ damage. Because the lysosomal enzyme does not cross the cell membrane of osteocartilaginous cells, it does not affect the orthopedic aspects. The risk of graft-versus-host disease is high but may be treated in most cases. Survival rate is 61% at 2 years for Hurler disease, which is otherwise fatal before maturity.

 

Correct Answer: Bone marrow transplantation does not affect the orthopedic problems.

 

A 3-year-old girl with spina bifida is able to flex and adduct her hips, but she cannot extend her knees. The hips are both dislocated. The patientâs parents ask how the hips must be treated to optimize function. The recommended treatment is:

 

1) Observation

3) Bilateral open reduction

2) Bilateral closed reduction

5) Bilateral open reduction with femoral osteotomies and muscle transfers

4) Bilateral open reduction and femoral osteotomies

 

Nonoperative treatment will allow the patient to retain maximum mobility of the hips.

Operative treatment carries substantial risk of avascular necrosis, stiffness, and redislocation. These risks are not worth taking in view of the patientâs limited muscle strength about the hips. In addition, the motor level alluded to (L2-L3) means that she will not have sensation of pain from the hips.

 

Correct Answer: Observation

 

 

1894. (856) Q5-1119:

A patient with spina bifida and L5 motor level undergoes tendon transfers about the ankle. After cast removal, he is lost to follow-up for 3 years. Upon re-examination, the patient has no motor power on either side below the knee. The most likely explanation is:

 

1) The tendon transfers have all pulled out.

3) His spinal cord has become tethered.

2) His ventriculoperitoneal shunt has malfunctioned.

5) He has sustained a silent compartment syndrome because of lack of sensation.

4) His muscles have fatigued.

 

This scenario is common. The most likely explanation of the patientâs loss of motor power on either side below the knee is a result of a tethered spinal cord.

 

It is unlikely that all transferred tendons have pulled out and that he has lost function in all of the other L5 muscles that should be active.

 

Shunt malfunction is a common occurrence in patients with spina bifida, but shunt malfunction does not present with a focal deficit at a distal level.

 

Muscle fatigue is not a recognized phenomenon in spina bifida.

Correct Answer: His spinal cord has become tethered.

 

 

1895. (857) Q5-1120:

A 6-year-old boy with spina bifida and L3 motor level presents to the clinic with bilateral swollen legs (below the knees). His legs have been swollen for the past 3 days and his knees are warm. The patient has an oral temperature of 38.1°C. He denies any pain. There is no history of systemic infection or of any other trauma. The problem may be most likely diagnosed by ordering:

 

1) A venogram of both lower extremities

3) Cell count, gram stain, and culture of synovial fluid of both knees

2) A duplex ultrasound of both lower extremities and the pelvic veins

5) Plain and stress varus-valgus radiographs of both knees

4) Aspirate of both distal femoral metaphyses

 

Low-energy fractures are the most likely cause of the patientâs bilateral swollen legs. The fractures may occur with everyday activities, and they do not cause pain because of the patientâs high neurologic level. If plain radiographs do not give the diagnosis, stress radiographs should be obtained to detect undisplaced physeal fractures.

 

 

Deep vein thrombosis is rare in this age group, especially bilaterally.

 

Spontaneous joint infection and spontaneous osteomyelitis are not any more likely in patients with spina bifida than in the general population.

Correct Answer: Plain and stress varus-valgus radiographs of both knees

 

A patient with spina bifida has significant symmetrical calcaneus positioning of both ankles. This is most likely due to:

 

1) Lack of ambulatory experience

3) Motor level at L3

2) Braces that are set in too much dorsiflexion

5) Motor level at S1

4) Motor level at L5

 

At motor level L5, dorsiflexors overpower plantarflexors to cause a calcaneus position.

 

This condition occurs from muscle imbalance. Even with significant ambulation, it will persist. Brace properties do not cause fixed deformity.

At L3, neither dorsiflexors nor plantarflexors of the ankle work. At S1, both dorsiflexors and plantarflexors are well innervated.

Correct Answer: Motor level at L5

 

1897. (859) Q5-1122:

Correction of the congenital gibbus in spina bifida must follow which of these surgical principles:

 

1) Long instrumentation

3) Avoidance of instrumentation

2) Short instrumentation

5) Anterior approach to deformity

4) Avoidance of bony resection

 

The leverage provided by long instrumentation prevents loss of correction and junctional deformity.

 

Short instrumentation poses a risk of junctional kyphosis or loss of fixation.

 

 

Because of the severe angular deformity, fusion in situ without correction will be followed by increasing deformity. Resection of one to three of the vertebrae on the lower limb of the kyphosis is essential to allow safe correction without excessive tension on vessels and viscera.

 

The anterior approach to the gibbus is deep and impractical. This approach does not allow mechanically efficient instrumentation.

Correct Answer: Long instrumentation

 

 

1898. (860) Q5-1123:

A patient with L4 level myelomeningocele has developed a full-thickness pressure sore on the heel that has a central necrosis and is draining. While undergoing debridement in the local emergency department, the patient develops labored respiration and a nondetectable blood pressure. The most likely cause is:

 

1) Latex allergy

3) Septic shock

2) Aortic dissection

5) Shunt failure

4) Spontaneous tension pneumothorax

 

Latex sensitivity is common in patients with spina bifida because of frequent exposure through catherizations and procedures. Latex avoidance is becoming the standard in institutions that commonly treat patients with spina bifida, but community hospitals that rarely see such patients may not always be aware of this problem.

 

 

 

Dissection is not likely unless the patient has a connective tissue disorder. Septic shock is not likely to develop from a freely draining peripheral ulcer.

 

Tension pneumothorax is not any more likely in patients with spina bifida than in the general population.

 

Although it is important to be aware of shunt failure, it is not likely in this procedure which is done without anesthetic.

Correct Answer: Latex allergy

 

The most common benign tumor of vertebral bodies is:

 

1) Osteoid osteoma

3) Osteochondrom

2) Osteoblastoma

5) Aneurysmal bone cyst

4) Giant cell tumor

 

Aneurysmal bone cyst is the most common benign bone tumor of the vertebral body. Aneurysmal bone cysts sometimes extend into the posterior elements.

 

 

Osteoid osteoma, osteoblastoma, and osteochondroma are primarily found in the posterior elements of the vertebrae. Giant cell tumor is rare in the spine.

Correct Answer: Aneurysmal bone cyst

 

 

1900. (885) Q5-1152:

A magnetic resonance image of a 7-year-old girl shows a line of high-signal intensity within the cord on T2 sequences that parallels the ventral surface of the cord and appears as a syrinx. However, on the axial images and on the T1 sequences, this finding is not evident. The most likely diagnosis is:

 

1) Collapsing syrinx

3) Motion artifact

2) Gibbs artifact

5) Astrocytoma

4) Ependymoma

 

A Gibbs artifact is a linear focus in the cord on T2-weighted images that parallels the ventral aspect of the cord and mimics a syrinx. The Gibbs artifact is due to the linear interface between two tissues of differing signal intensity. It is not seen on axial images or T1-weighted images.

 

 

 

Motion artifact is a blurring of the image due to patient motion, respiration, or cerebral spinal fluid pulsation. Ependymoma and astrocytoma are seen on both the axial and the sagittal images.

Correct Answer: Gibbs artifact

 

 

1901. (886) Q5-1154:

Which of the following statements correctly represents the definitions of Chiari I and Chiari II malformations:

 

1) A Chiari I malformation is displacement of the brainstem through the foramen magnum; a Chiari II malformation also includes the cerebellum.

3) A Chiari I malformation is a cyst in the pons; a Chiari II malformation is a cyst including the cerebellum.

2) A Chiari I malformation is displacement of the cerebellum through the foramen magnum; a Chiari II malformation also includes the brainstem.

5) A Chiari I malformation is a split in the spinal cord; a Chiari II malformation also includes a tight filum terminale.

4) A Chiari I malformation is an absence of the fourth ventricle; a Chiari II malformation also includes the presence of hydrocephalus.

 

A Chiari I malformation involves displacement of the cerebellum through the foramen magnum. A Chiari II malformation involves similar displacement of the brainstem, as well. In a Chiari II malformation, there is almost always a myelomeningocele in the lower cord. Sometimes a syrinx is also present.

 

Correct Answer: A Chiari I malformation is displacement of the cerebellum through the foramen magnum; a Chiari II malformation also includes the brainstem.

 

The conus medullaris is always termed abnormal if it terminates below this level:

 

1) L1

3) L2

2) L1-2 disk

5) L3

4) L2-3 disk

 

The vertebral bodies grow faster than the spinal cord. A more caudal placement of the conus medullaris is considered normal in an older patient. However, a conus medullaris below L3 at any age is abnormal and may represent a tethered and/or dysplastic cord.

 

Correct Answer: L3

 

 

1903. (888) Q5-1156:

Regardless of age, the tonsils of the cerebellum should not extend more than this distance below the foramen magnum:

 

1) 1 mm

3) 3 mm

2) 2 mm

5) Any protrusion is abnormal

4) 5 mm

 

Displacement of the cerebellar tonsils below the foramen magnum is termed a Chiari malfomation. A Chiari I malformation is an isolated malformation; a Chiari II malformation is associated with a syrinx. Symptoms may include headache, vomiting, and nystagmus. Although the normal protrusion allowed decreases with age, 5 mm is abnormal at any age.

 

Correct Answer: 5 mm

 

 

1904. (889) Q5-1157:

When applying a halo, the use of six pins instead of four has which of the following effects:

 

1) Increase in the infection rate

3) Increased rate of dural puncture

2) Increased rate of pin loosening

5) Increase in risk of injury of the supraorbital nerve

4) Increase in load to failure by 50%

 

With an increase from four to six pins, the load to failure is increased by more than 50%. Additional pins decrease many of the complications, such as loosening and pin tract infection. The risk of dural puncture is not measurably increased because it is a rare complication.

 

Correct Answer: Increase in load to failure by 50%

 

 

1905. (890) Q5-1158:

Which of the following findings is an indication for the Bernese (Ganz) osteotomy:

 

1) Acetabular dysplasia in a 10-year-old boy.

3) A 19-year-old man with osteonecrosis (Stulberg-Stage 4) secondary to Perthes disease.

2) A 20-year-old woman with a complete developmental dislocation of the hip.

5) A 35-year-old woman with an AP center-edge angle of 0° and a joint space of 2 mm

4) A 30-year-old woman with an anteroposterior (AP) center-edge angle of 10° and a joint space of 1 mm.

 

The Bernese (Ganz) osteotomy is contraindicated in patients with open triradiate cartilages because it crosses the cartilage. The procedure is also contraindicated in complete dislocations, incongruous hips, and advanced osteoarthritis with a joint space of 1 mm or less.

 

Correct Answer: A 35-year-old woman with an AP center-edge angle of 0° and a joint space of 2 mm

 

1906. (891) Q5-1159:

The false profile radiographic view of the hip is taken with the patient in which of the following positions:

 

1) The effected hip is against the film and the pelvis is rotated 65° away from the plane of the film.

3) The patient is supine and the beam is angled 45° cephalad.

2) The uneffected hip is against the film and the pelvis is rotated 65° away from the plane of the film.

5) The patient is supine and the affected hip is maximally internally rotated.

4) The patient is supine and the beam is angled 45° caudally.

 

The false profile radiographic view assesses anterior coverage and is obtained with the affected hip against the film and the pelvis rotated 65° away from the plane of the film.

Correct Answer: The effected hip is against the film and the pelvis is rotated 65° away from the plane of the film.

 

 

1907. (892) Q5-1160:

A 15-year-old boy has a history of excessive bleeding when he is cut. His maternal uncle passed away during a tonsillectomy. The boy presents with severe hip pain (worse in extension than in flexion) and some weakness in his knee extension. The most likely cause of the patientâs symptoms is:

 

1) Septic arthritis of the hip

3) Lumbar epidural hematoma

2) Psoas abscess

5) Hip hemarthrosis

4) Iliopsoas hematoma

 

The patient has features suggesting mild hemophilia A or B including x-linked inheritance and occasional bleeding episodes. Iliopsoas hematoma is a complication of hemophilia. Bleed into the hip or the lumbar epidural space is less common and there are no signs to suggest infection.

 

Correct Answer: Iliopsoas hematoma

 

 

1908. (893) Q5-1161:

A patient with hemophilia A has a hematoma of the iliopsoas. He has a partial femoral nerve palsy. Treatment involves continuous factor replacement and:

 

1) Open drainage

3) Percutaneous insertion of a drainage tube

2) Decompression of the fascia over the femoral nerve

5) Observation

4) Embolization of feeder vessels by interventional radiologist

 

The standard treatment of a psoas abscess is continuous factor replacement. Surgery is usually unnecessary, but it may be considered in cases of acute palsy with severe pain unresponsive to medical therapy. A percutaneous drainage tube is not recommended because the hematoma may be difficult to locate or drain.

 

Correct Answer: Observation

 

Which of the following best predicts the risk of nonaccidental injury in young children who sustain a fracture of the femur:

 

1) The fracture pattern

3) The socioeconomic status of the family

2) The location of the fracture

5) Coexistence of other disabilities

4) Whether the child was previously able to walk

 

Although a spiral fracture is classically consistent with nonaccidental injury, this is not always true. A spiral fracture is often seen in accidental injuries. The location of the fracture in the femur, the socioeconomic status of the family, and the coexistence of other disabilities are all poor predictors. The ability of the child to walk has predictive ability. In one study, nonaccidental injury accounted for 42% of all femur fractures in children before walking age vs. only 2% of fractures after walking age.

 

Correct Answer: Whether the child was previously able to walk

 

 

1910. (895) Q5-1163:

Which of the following is known about the genetics of multiple hereditary exostosis (MHE):

 

1) There is no genetic pattern for this condition.

3) MHE follows an autosomal recessive pattern.

2) MHE follows an x-linked inheritance pattern.

5) MHE involves a defect in fibroblast growth factor.

4) MHE encodes glycosyltransferases needed for biosynthesis of heparan sulfate.

 

Multiple hereditary exostosis (MHE) is inherited as an autosomal dominant condition. There are three genes known to be involved: EXT 1, 2, and 3. EXT 1 and 2 encode glycosyltransferases needed for biosynthesis of heparan sulfate. Fibroblast growth factor receptor is abnormal in achondroplasia.

 

Correct Answer: MHE encodes glycosyltransferases needed for biosynthesis of heparan sulfate.

 

 

1911. (896) Q5-1164:

A 10-year-old patient has a painful lytic lesion replacing 75% of the distal femoral metaphysis. The lesion has indistinct margins, but there is not any periosteal reaction. Biopsy reveals a benign histiocytic tumor with multiple eosinophils. Physical exam and skeletal survey reveal no other abnormalities. Which of the following would be the most appropriate treatment method:

 

1) Radiation therapy

3) Oral prednisone treatment for one month

2) Methotrexate, ifosfamide and adriamycin followed by resection

5) Curettage and bone graft

4) Resection and reconstruction

 

This patient has an eosinophilic granuloma, which is an isolated form of Langerhans cell histiocytosis. Patients with isolated involvement may be treated with observation, curettage, bone grafting, or steroid injection. For a lesion of this size, the risk of fracture is high and curettage with bone grafting is recommended. Chemotherapy and/or systemic steroids are reserved for patients with systemic disease.

 

Correct Answer: Curettage and bone graft

 

Which of the following methods has the highest sensitivity in detecting skeletal lesions in patients with Langerhans cell histiocytosis:

 

1) Physical examination

3) Ultrasound

2) Skeletal survey

5) Indium labeled white blood cell study

4) Bone scan

 

Skeletal survey is the most sensitive means of detecting lesions of eosinophilic granuloma (Langerhans cell histiocytosis). Bone scan detects most, but not all, of the lesions. Some lesions lack enough osteoblastic activity to appear on bone scan. Lesions are not detected on physical exam unless they are large enough to cause pain or tenderness. Indium labeled white cell studies are not used in this condition.

 

Correct Answer: Skeletal survey

 

 

1913. (898) Q5-1166:

A 15-year-old girl presents with pain and a 17°-scoliosis curve. The film suggests an enlargement and sclerosis of the transverse process of L2. The best study to further evaluate the nature and anatomic extent of the lesion is:

 

1) Coned radiographs

3) Bone scan with SPECT (single photon emission computed tomography)

2) Bone scintigraphy

5) Computed tomography

4) Magnetic resonance imaging

 

This lesion is most likely to be an osteoid osteoma or an osteoblastoma. The best study to further evaluate the nature and anatomic extent is with computed tomography.

Magnetic resonance imaging may overestimate the lesion because of sensitivity to edema in the marrow and surrounding soft tissue. Bone scintigraphy with or without SPECT does not provide enough anatomic detail. Coned radiographs do not provide the needed multidimensional detail for this condition, as well as many other spinal problems.

 

Correct Answer: Computed tomography

 

 

1914. (899) Q5-1167:

Fibrous dysplasia has been determined to be the result of a mutation in which gene:

 

1) GNAS1

3) FBN1

2) FGFR3

5) COL1A1

4) CFBA1

 

Fibrous dysplasia has been determined to be the result of a postzygotic mutation in the GNAS1 (stimulatory G protein of adenylyl cyclase) gene. All of the cells that are descended from the cell with the mutation manifest features of the disease.

 

 

FGFR3 (fibroblast growth factor receptor-3 protein) is abnormal in achondroplasia. FBN1 (fibrillin 1) is abnormal in Marfan syndrome.

 

CFBA1 is abnormal in cleidocranial dysplasia.

 

COL1A1 (collagen, type 1, alpha 1) is abnormal in osteogenesis imperfecta.

Correct Answer: GNAS1

 

Which of the following conditions is not associated with congenital contractures:

 

1) Amyoplasia

3) Popliteal pterygium syndrome

2) Diastrophic dysplasia

5) Down syndrome

4) Larsen syndrome

 

Down syndrome is the only condition listed that does not involve congenital contractures.

 

 

 

Amyoplasia (or classic arthrogryposis) involves multiple contractures with normal intelligence and loss of flexion creases. Diastrophic dysplasia is a disorder of cartilage that includes contractures of many joints, as well as epiphyseal irregularity. Popliteal pterygium syndrome includes contractures with webbing across the flexor surfaces of the joints.

 

Larsen syndrome involves multiple joint dislocations, as well as contractures.

Correct Answer: Down syndrome

 

 

1916. (901) Q5-1169:

Diastematomyelia is best described as:

 

1) Protrusion of neural elements dorsally from the back.

3) Two hemivertebrae at adjacent levels.

2) A duplicated spinal column.

5) A cyst within the spinal cord.

4) An osseous or fibrocartilaginous septum in the spinal canal.

 

A diastematomyelia is an osseous or fibrocartilaginous septum within the spinal canal that invaginates or splits the neural tissue. A diastematomyelia may or may not be associated with neurologic deficit. Surgical excision is indicated for the presence of symptoms or at the time of spinal deformity correction.

 

Correct Answer: An osseous or fibrocartilaginous septum in the spinal canal.

 

 

1917. (902) Q5-1170:

Diastematomyelia is most common in which region of the spine:

 

1) Cervical

3) Lower thoracic

2) Upper thoracic

5) Sacral

4) Lumbar

 

Several series of patients with diastematomyelia have shown the following anatomic distribution: 64% occur in the lumbar spine, 27% occur in the lower thoracic spine, 9% in the upper thoracic spine, and almost none occur in the cervical or sacral regions.

Correct Answer: Lumbar

 

 

 

1) Cutaneous lesions over the spine

3) Foot deformity

2) Anisomelia

5) Widened interpedicular distance

4) Neurologic deficit

 

Widened interpedicular distance is the most common finding (94% or greater) in patients with diastematomyelia.

 

Neurologic deficit is present in 60% to 88% of patients with diastematomyelia.

 

 

A cutaneous lesion, such as a dimple or hairy patch, is present in 55% to 79% of patients with diastematomyelia. Anisomelia is present in 53% to 58% of patients with diastematomyelia.

 

Foot deformities are present in 32% to 52% of patients with diastematomyelia.

Correct Answer: Widened interpedicular distance

 

 

1919. (904) Q5-1172:

In which of the following clinical situations would it be most appropriate to recommend an anterior and posterior hemiepiphyseodesis of the convex side of a congenital scoliosis:

 

1) A 12-year-old girl with a hemivertebra of L3.

3) A 5-year-old boy with a congenital bar of T3-4.

2) A 10-year-old boy with a hemivertebra of L3.

5) A 5-year-old boy with a hemivertebra of L5.

4) A 5-year-old boy with a hemivertebra of T5.

 

Hemiepiphyseodesis is most likely to produce progressive curve correction in patients under age 6. Hemiepiphyseodesis is not likely to result in improvement when there is no concave growth potential, as in the case of a congenital bar. In hemivertebra of the lumbosacral region, improvement is most likely when a hemivertebra resection is performed.

 

Correct Answer: A 5-year-old boy with a hemivertebra of T5.

 

 

1920. (905) Q5-1173:

Which of the following is the seating arrangement recommended for a 5-year-old in a family automobile:

 

1) Lap belt in the middle of the back seat

3) Lap and shoulder belt and booster seat in back

2) Lap and shoulder belt in the back seat

5) Lap and shoulder belt and booster seat in front

4) Rear-facing child seat in back

 

Children ages 4 to 8 (40 lbs to 60 lbs) are at risk for airbag injuries and should not be in the front seat. In addition, they require booster seats to allow proper fitting of the shoulder harness on the upper torso. Rear-facing seats are only appropriate for infants. Children should not be in the front seat until after age 12 and over 100 lbs.

 

Correct Answer: Lap and shoulder belt and booster seat in back

 

benefits:

 

1) Increased survival rate

3) Decreased incidence of deep venous thrombosis

2) Decreased incidence of fat emboli

5) Greater ease of management

4) Improved recovery of central nervous system function after head injury

 

Acute internal fixation in children has not been shown to provide the same life-saving and morbidity-reducing benefits as in adults. However, it will ease the management of the polytraumatized patient.

Correct Answer: Greater ease of management

 

 

1922. (907) Q5-1175:

Which of the following statements is true about pin fixation of supracondylar humerus fractures:

 

1) Fixation with two lateral pins is biomechanically stronger than crossed pins.

3) Pin fixation is associated with a higher risk of compartment syndrome than cast treatment alone.

2) Fixation with two lateral pins has a higher risk of iatrogenic nerve injury than medial and lateral pins.

5) Use of two medial pins is the most biomechanically stable construct.

4) Fixation with medial and lateral pins carries a higher risk of iatrogenic ulnar nerve injury than two lateral pins.

 

Pin fixation of supracondylar humerus fractures in children decreases the risk of compartment syndrome as well as malreduction. Use of medial and lateral pins has been shown to be the most stable construct in biomechanical studies. The use of medial and lateral pins carries a risk of iatrogenic injury to the ulnar nerve. No nerve injuries have been demonstrated after the use of two lateral pins.

 

Correct Answer: Fixation with medial and lateral pins carries a higher risk of iatrogenic ulnar nerve injury than two lateral pins.

 

 

1923. (908) Q5-1176:

A 6-year-old boy who is treated for a type III supracondylar fracture has no neurovascular deficit on initial examination. His fracture is treated by anatomic closed reduction and percutaneous fixation with medial and lateral pins. A partial deficit in ulnar motor and sensory function is noted after surgery. The next step in treatment should be:

 

1) Remove the lateral pin.

3) Explore the ulnar nerve.

2) Remove both pins and place the patient in traction.

5) Make sure the elbow is flexed to no more than 90° and observe.

4) Transpose the ulnar nerve anteriorly.

 

The use of medial pins in treatment of supracondylar fractures of the humerus has been associated with a risk of ulnar nerve injury. However, in the largest reported series, 16 of 17 palsies resolved spontaneously and completely with observation after a mean of 18 weeks. There is no evidence that pin removal, exploration, or transposition is necessary at this stage. Hyperflexion in a splint may cause the nerve to be apposed to the pin and can be decreased.

 

Correct Answer: Make sure the elbow is flexed to no more than 90° and observe.

 

 

 

1) Increase in the size of the curve

3) Increased level of training in deformity surgery

2) Decrease in the size of the curve

5) Use of the Ferguson technique instead of the Cobb technique

4) Film coned and centered on the deformity

 

Measurement of congenital scoliosis has an intraobserver variation of 8°. Measurement accuracy decreases with decreased level of training, increased curve size, and decreased clarity of the endpoints. A coned and centered film would give better detail.

Correct Answer: Increase in the size of the curve

 

 

1925. (910) Q5-1178:

Which of the following structures enables the growth plate to grow in diameter as the skeleton gets larger:

 

1) The chondroepiphysis

3) The groove of Ranvier

2) The apophysis

5) The secondary spongiosa

4) The ring of LaCroix

 

The ossification groove of Ranvier contains chondrocytes that provide latitudinal growth of the growth plate. The groove of Ranvier may be thought of as âthe growth ring of the growth plate.â

 

 

The chondroepiphysis is the cartilage precursor of the epiphysis. The apophysis is a nonarticular growth region.

 

 

The ring of LaCroix is a peripheral structure that reinforces the growth plate. The secondary spongiosa is mature lamellar bone after it has been remodeled.

Correct Answer: The groove of Ranvier

 

 

1926. (911) Q5-1180:

Which area of the growth plate has the lowest oxygen tension:

 

1) The reserve zone

3) The hypertrophic zone

2) The proliferative zone

5) The perichondral ring

4) The zone of calcification

 

The lowest pressure of oxygen is in the hypertrophic zone because it is poorly vascularized. This is one of the features that triggers cell death and calcificaton.

Correct Answer: The hypertrophic zone

 

 

1927. (912) Q5-1181:

How many major centers of ossification comprise the normal lumbar vertebral body:

 

1) One

3) Three

2) Two

5) Five

4) Four

 

The normal lumbar and thoracic vertebrae are comprised of two posterior and one main anterior ossification center. These join at the neurocentral synchondrosis and the midline vertebal ossification center.

Correct Answer: Three

 

1928. (913) Q5-1182:

The somite refers to which of the following features of embryologic development:

 

1) A group of cells that produce the limb bud

3) A group of cells that differentiate to form the cranium

2) A structure that gives rise to the spinal cord

5) A set of homeobox genes

4) A group of cells that are repeated to form the axial skeleton

 

The somite is a group of tissues that are bilaterally symmetric and repeated to comprise the axial skeleton. Each somite contains a dermatome, myotome, and sclerotome.

Correct Answer: A group of cells that are repeated to form the axial skeleton

 

 

1929. (914) Q5-1183:

Homeobox genes are best characterized by which of the following descriptions:

 

1) Homeobox genes regulate fracture healing.

3) Homeobox genes are a series of genes that regulate limb genesis.

2) Homeobox genes are involved in immune system regulation.

5) Homeobox genes regulate vascular invasion of the physis.

4) Homeobox genes regulate spinal development.

 

Homeobox genes code for transcription factors involved in regulation of limb genesis in the embryonic period. Homeobox genes have a highly conserved 60-nucleotide sequence.

Correct Answer: Homeobox genes are a series of genes that regulate limb genesis.

 

 

1930. (915) Q5-1184:

Which of the following regions of the physis has the highest degree of proteoglycan aggregation in the matrix:

 

1) Reserve zone

3) Hypertrophic zone

2) Proliferative zone

5) Metaphysis

4) Zone of degeneration

 

The proteoglycan aggregation is greatest in the reserve zone and in the epiphysis. Proteoglycan concentration progressively decreases in the direction of the metaphysis. This phenomenon also enables calcification to occur.

Correct Answer: Reserve zone

 

 

1931. (4040) Q5-1186:

Which of the following categories of slipped capital femoral epiphysis (SCFE) is associated with the highest rate of bilaterality:

 

1) Idiopathic SCFE

3) Radiation therapy

2) Renal failure

5) Growth hormone deficiency

4) Hypothyroidism

 

Renal failure is associated with the highest risk (90%) of bilaterality. By contrast, idiopathic SCFE has a 20% risk of bilaterality initially and a further 10% to 20% risk until maturity.

Correct Answer: Renal failure

 

An African-American mother brings her 18-month-old baby in for examination because he has bowing of the legs. The childâs past medical history is normal including height and weight. The child is still breast-fed. Radiographs reveal mild widening and irregularity of the physes. Which of the following conditions is likely to be found on laboratory examination:

 

1) Elevated phosphorus

3) Normal 1,25 dihydroxyvitamin D

2) Decreased alkaline phosphatase

5) Elevated parathyroid hormone

4) Elevated creatinine

 

This patient has nutritional rickets because of a combination of decreased vitamin D intake and increased need for sunlight due to dark skin. The laboratory findings should include a lowânormal calcium phosphate, elevated alkaline phosphatase and parathryoid hormone, and low vitamin D levels and precursors. Vitamin D supplementation without bracing is adequate treatment.

 

Correct Answer: Elevated parathyroid hormone

 

 

1933. (917) Q5-1188:

The dose of acetaminophen for a child should be no more than:

 

1) 1.5 mg/kg

3) 10-15 mg/kg

2) 5 mg/kg

5) 50 mg/kg

4) 20-30 mg/kg

 

The dose of acetaminophen should be 10-15 mg/kg. The maximum daily dose should be no more than 75 mg/kg. Larger doses of acetaminophen pose a risk of hepatotoxicity over a long period of time.

Correct Answer: 10-15 mg/kg

 

 

1934. (918) Q5-1189:

Opioids must be used with caution in infants younger than 3 months of age because of which of the following properties:

 

1) Increased clearance

3) Increased hepatic activity

2) Increased protein binding

5) Decreased body surface area to mass

  1. Decreased clearance and protein binding

     

    Opioids must be used with caution in infants younger than 3 months of age because of decreased clearance and decreased protein binding. This results in greater bioavailability.

    Correct Answer: Decreased clearance and protein binding

     

     

    1935. (919) Q5-1190:

    The potency ratio of hydromorphone to morphine sulfate is:

     

    1) 1-2:1

    3) 7-10:1

    2) 5:1

  2. 50:1

4) 20-25:1

 

The potency ratio of commonly-used drugs is one of the factors that determines the appropriate dose of a drug. The potency ratio of hydromorphone to morphine sulfate is 7-10:1.

Correct Answer: 7-10:1

 

The youngest age at which a number scale to quantitate pain (i.e., rating it as 1-10) can be appropriately used is:

 

  1. 2 years

3) 5 years

2) 3 years

5) 9 years

4) 7 years

 

Pain measurement is becoming increasingly important for both children and adults. A number scale to quantitate pain can be appropriately used for most children over the age of 7 years. For younger children, the âfaces scaleâ or a finger span scale (stretching the fingers as far apart as possible) is more accurate.

 

Correct Answer: 7 years

 

 

1937. (921) Q5-1192:

Which of the following findings on a neuromuscular examination is not characteristic of spasticity:

 

1) Hyperreflexia

3) Velocity-dependent resistance

2) Clonus

5) Muscle contracture

4) Writhing movements

 

Spasticity is a hallmark of upper motor lesions. Spasticity is characterized by hyperreflexia, clonus, and velocity-dependent resistance. After early childhood, muscle contracture also develops. Writhing movements are characteristic of athetosis, not spasticity.

 

Correct Answer: Writhing movements

 

 

1938. (922) Q5-1193:

Which of the following describes the mechanism of action of Botulinum toxin type A:

 

1) Botulinum toxin type A blocks the release of acetylcholine.

3) Botulinum toxin type A interrupts transmission of impulses along axons.

2) Botulinum toxin type A prevents binding of acetylcholine to receptors.

5) Botulinum toxin type A depolarizes sarcolemma of muscles.

4) Botulinum toxin type A softens fibrous tissue causing contracture.

 

Botulinum toxin type A acts by blocking the release of acetylcholine at the nerve terminal. Over time, new terminals sprout and the effect is lost. By contrast, alcohol and phenol denature proteins and disrupt nerve conduction.

Correct Answer: Botulinum toxin type A blocks the release of acetylcholine.

 

 

1939. (923) Q5-1194:

Which of the following describes the mechanism of action of baclofen:

 

1) Baclofen mimics the effect of gamma-aminobutyric acid (GABA).

3) Baclofen binds to actylcholine receptors.

2) Baclofen blocks the effect of GABA.

5) Baclofen inhibits serotonin reuptake.

4) Baclofen potentiates the effect of actylcholine.

 

Baclofen mimics the effect of gamma-aminobutyric acid, an inhibitory neurotransmitter in both the peripheral and central nervous system. Baclofen reduces peripheral spasticity. A major side effect of baclofen is sedation. The ratio of sedation to spasticity reduction may be enhanced by intrathecal administration.

 

Correct Answer: Baclofen mimics the effect of gamma-aminobutyric acid (GABA).

 

 

 

1) The vertical migration of the center of the femoral head with respect to the lateral margin of the acetabulum.

3) The ratio of the uncovered portion of the femoral head to its total width.

2) The distance between the medial wall of the acetabulum and the femoral head.

5) The angle between two lines through the center of the femoral head â one vertical and one through the lateral edge of the acetabulum.

4) The difference in subluxation between neutral and abduction films.

 

The migration index of Reimers is used to quantitate hip subluxation in cerebral palsy. It is defined as the ratio of the uncovered portion of the femoral head (lateral to a vertical line through the outer edge of the acetabulum) to the total width of the head.

Correct Answer: The ratio of the uncovered portion of the femoral head to its total width.

 

 

1941. (925) Q5-1196:

An appropriate indication for preoperative magnetic resonance imaging in adolescent idiopathic scoliosis is:

 

1) Any curve requiring surgery

3) An abnormal neurologic examination

2) Any curve more than 90°

5) Any progression of 12° in 6 months

4) Any progressive deformity in a male

 

The only absolute indication for preoperative magnetic resonance imaging in idiopathic adolescent scoliosis is an abnormal neurologic exam or abnormal curve pattern.

Correct Answer: An abnormal neurologic examination

 

 

1942. (926) Q5-1197:

The mean Cobb measurement for idiopathic scoliosis curves with a 7° angle of trunk rotation (ATR) is:

 

1) 10°

3) 20°

2) 15°

5) 30°

4) 25°

 

Although the angle of trunk rotation (ATR) does not convert directly to a Cobb angle, there are population-based figures for mean curve at each ATR. The mean Cobb angle for curves having a 7° ATR is 20°.

Correct Answer: 20°

 

 

1943. (927) Q5-1198:

Using a 5° angle trunk rotation (ATR) as a positive screening threshold for detection of curves with a Cobb angle over 20° is characterized by:

 

1) A high degree of sensitivity and specificity

3) A high degree of specificity but low sensitivity

2) A high degree of sensitivity but low specificity

5) No predictable relationship to Cobb angle

4) A low degree of sensitivity and specificity

 

Use of a 5° ATR threshold for detection of curves with a Cobb angle over 20° has a sensitivity of 98%, but a specificity of only 64%.

Correct Answer: A high degree of sensitivity but low specificity

 

 

 

1) 20° to 50°

3) 25° to 45°

2) 20° to 40°

5) 30° to 60°

4) 25° to 60°

 

 

The normal values for kyphosis in the general population (2 standard deviations above and below the mean) are 20° to 50°. Correct Answer: 20° to 50°

 

1945. (929) Q5-1200:

Which of the following is the most common cause of death in children with pelvic fractures:

 

1) Intrapelvic exsanguination

3) Pulmonary embolism

2) Head injury

5) Spinal cord injury

4) Multisystem organ failure

 

Head injury is the cause of virtually all mortality in children with pelvic fractures. The rarity of life-threatening intrapelvic hemorrhage is in marked contrast to adults. Pulmonary embolism and multisystem organ failure are rare in children.

Correct Answer: Head injury

 

 

1946. (930) Q5-1201:

An 8-year-old pedestrian sustains a pelvic fracture when struck by a car. The injury consists of a posterior iliac fracture in which the iliac wing is rotated internally, although it is not vertically displaced. An anterior fracture of both ipsilateral pubic rami is present with 2 cm of overlap. Recommended treatment includes:

 

1) Open reduction and plate fixation of the rami

3) Percutaneous sacral screw fixation

2) Open reduction and plate fixation of the iliac wing

5) Bedrest

4) External fixation

 

A childâs pelvis can absorb a significant amount of energy without serious displacement. There is rapid healing and good remodeling potential. Bedrest is the appropriate treatment for this patient because of the low risk of complications.

Correct Answer: Bedrest

 

 

1947. (931) Q5-1202:

Which of the following factors should prompt a workup for an underlying disorder in a patient with slipped capital femoral epiphysis:

 

1) A bilateral slip

3) Weight under the 50th percentile for age

2) Weight over the 50th percentile for age

5) A slip of grade II or higher

4) A positive family history

 

A positive age-weight test is defined as a patient who is under the 50th percentile of weight for age or older than 16 years of age at presentation with slipped capital femoral epiphysis. These patients should have a workup for underlying renal or endocrine disease. A positive family history is found in 5% to 10% of patients with slipped capital femoral epiphysis.

 

Correct Answer: Weight under the 50th percentile for age

 

slip:

 

1) Idiopathic SCFE

3) Radiation therapy

2) Renal failure

5) Growth hormone deficiency

4) Hypothyroidism

 

A grade III slip refers to more than 50% displacement of the epiphysis. Renal failure is associated with the highest risk (43%) of progressing to a grade III slip. Fortunately, these slips are usually stable.

Correct Answer: Renal failure

 

 

1949. (1089) Q5-1449:

Which of the following is the central element of Klippel-Feil syndrome:

 

1) A low posterior hairline

3) Cervical stenosis

2) Hearing impairment

5) Cervical scoliosis

4) Congenital cervical fusion

 

The essential feature of the definition of Klippel-Feil syndrome is congenital cervical fusion. Features other than congenital cervical fusion are sometimes seen but are not essential features of the syndrome. Features other than those listed above include scoliosis in different regions of the spine, congenital heart defects, a short neck, Sprengel deformity, and renal anomalies.

 

Correct Answer: Congenital cervical fusion

 

 

1950. (1090) Q5-1450:

The most common cause of neurologic deficit in patients with Down syndrome is:

 

1) Basilar invagination

3) Atlantoaxial instability

2) Atlanto-occipital instability

5) Cervical kyphosis

4) Subaxial instability

 

Basilar invagination is extremely rare in this syndrome, and problematic cervical kyphosis is equally uncommon. Atlanto-occipital translation and subaxial instability occur but rarely cause symptoms. Atlantoaxial instability is the most common of the cervical abnormalities listed to cause neurologic compromise in Down syndrome. Nevertheless, surgery is only indicated if the atlantoaxial interval exceeds 6 mm, or causes neurologic deficits.Correct Answer: Atlantoaxial instability

 

 

1951. (1091) Q5-1451:

The timing of maximal progression of idiopathic scoliosis is most closely correlated with:

 

1) Chronological age

3) Peak height velocity

2) Skeletal age

5) Tanner stage

4) Risser sign

 

The cause of scoliosis or its progression is still not known. However, the timing of the progression is most closely correlated with the peak velocity of growth. The other factors are more weakly correlated: a chronological or skeletal age of 10 to 12 years, Risser sign of 0, or Tanner sign of 0 to 1.Correct Answer: Peak height velocity

 

Which of the following events typically occurs first in adolescent girls:

 

1) Risser sign turning to 1

3) Menarche

2) Peak height velocity

5) Tanner sign of 3

4) Skeletal maturity

 

Typically, the peak height velocity is the first of these events to occur in adolescent girls. Skeletal maturity is the last event to occur.Correct Answer: Peak height velocity

 

 

1953. (1093) Q5-1453:

A 15-year-old with idiopathic scoliosis develops a temperature of 39.5° C 8 days after posterior spinal fusion with instrumentation. Blood and urine cultures show no growth, and chest X-ray is normal. Your next step should be which of the following:

 

1) Begin intravenous cephalosporin treatment

3) Aspirate the wound for culture

2) Begin intravenous vancomycin treatment

5) Debride the wound and remove the bone graft

4) Debride the wound and remove the instrumentation

 

There is a significant likelihood of a deep wound infection, although other possibilities exist, including superficial wound infection, atelectasis, and drug-related fever. Therefore, confirmation of the diagnosis is the next step. The wound should be aspirated for culture next. If infection is confirmed, the wound should be opened and debrided.Correct Answer: Aspirate the wound for culture

 

 

1954. (1094) Q5-1454:

Nine days after surgery, a 16-year-old boy with idiopathic scoliosis has a temperature of 39.5° C. Wound aspiration reveals gram-positive cocci in clusters. Your next step in management is:

 

1) Begin intravenous cephalosporin and monitor the response

3) Begin hyperbaric oxygen and intravenous antibiotics

2) Begin intravenous vancomycin and monitor the response

5) Open and debride the wound, and remove the instrumentation

4) Open and debride the wound, leaving the instrumentation in place

 

Open debridement is the treatment of choice if an early deep wound infection is confirmed after spinal fusion. The instrumentation is left in place to stabilize the wound and promote fusion. The wound is closed as long as it can be cleaned up adequately during surgery, and muscle has a healthy appearance. If this cannot be achieved or if several debridements fail, the wound may be left open.Correct Answer: Open and debride the wound, leaving the instrumentation in place

 

 

1955. (1095) Q5-1455:

A thoracic curve of more than 50° due to an idiopathic adolescent scoliosis curve has an increased risk of causing all except which of the following consequences in adulthood, when compared to the general population:

 

1) Restrictive lung disease

3) Back pain

2) Obstructive lung disease

5) Increase in curvature

4) Less positive body image

 

Idiopathic adolescent scoliosis is likely to progress in adulthood if it exceeds 50°. The rate of progression in adults is slower than during adolescence; about 1° per year. Nevertheless, it is likely to have some effect on pulmonary function later during adulthood. The effect is one of restrictive rather than obstructive lung disease. There is an increase in the risk of back pain, although it is rarely disabling. Patients with idiopathic scoliosis as a group have a more negative body image, although it is not a serious life impairment. In all of these parameters, there are individual exceptions.Correct Answer: Obstructive lung disease

 

A 13-year-old girl with idiopathic adolescent scoliosis has a 32° right thoracolumbar curve. Her Risser sign is 1. Her curve measured 29° 4 months ago. You recommend:

 

1) Observing until the curve reaches 34°

3) Physical therapy to control the curve

2) Ordering a magnetic resonance image of the spine

5) A thoracolumbosacral orthosis

4) Electrical stimulation to the paraspinal muscles

 

Idiopathic scoliosis in skeletally immature patients should be braced if it is greater than 30° and significant growth remains (estimated by a Risser sign of 0, 1, or 2). Studies have shown that patients with idiopathic scoliosis without atypical findings do not need magnetic resonance imaging. Physical therapy and electrical stimulation have been shown not to have any effect on the progression of idiopathic scoliosis.Correct Answer: A thoracolumbosacral orthosis

 

 

1957. (1097) Q5-1457:

Which of the following factors are not related to the success of brace treatment for idiopathic scoliosis?

 

1) Curve size at start of treatment

3) Number of hours worn

2) Curve correction in brace

5) Positive family history of scoliosis

4) Gender

 

A lower chance of curve control with brace treatment has been shown with curves greater than 40°, correction of less than 50% in brace, brace worn fewer than 16 hours per day, or male gender. Positive family history has not been shown to be related to curve progression or chance of control.Correct Answer: Positive family history of scoliosis

 

 

1958. (1098) Q5-1458:

A 12-year-old patient has a rigid, 135° scoliosis. Gradually increasing halo-gravity traction is planned in correcting the curve. The maximum weight that should be used is:

 

1) 15% of body weight

3) 25% of body weight

2) 20% of body weight

5) 35% to 45% of body weight

4) 30% of body weight

 

Halo-gravity traction is a safe and effective means of gradually correcting large curves. It allows balanced force application and continuous neurological monitoring. Cranial and peripheral nerves should be monitored. The weight can be increased gradually up to 35% to 45% of body weight, as documented in published series.Correct Answer: 35% to 45% of body weight

 

 

1959. (1099) Q5-1459:

In evaluating infants for Blount disease, which diagnostic parameter allows correct classification of the greatest number of children:

 

1) Langenskjold rating

3) Tibial metaphyseal-diaphyseal (M-D) angle

2) Tibiofemoral angle

5) Femoral intercondylar distance divided by height

4) Femoral M-D angle divided by tibial M-D angle

 

The tibial metaphyseal-diaphyseal (M-D) angle, no matter what threshold is set, has a significant number of false-positives or false-negatives. Using the ratio of femoral to tibial M-D angle improves diagnostic accuracy significantly. The Langenskjold rating, tibiofemoral angle, and intercondylar distances, although conceptually important, have low diagnostic accuracy for an individual patient.Correct Answer: Femoral M-D angle divided by tibial M-D angle

 

Which of the following statements best characterizes a Dega osteotomy as a distinct osteotomy from a Salter osteotomy:

 

1) The Dega osteotomy is stabilized by threaded pins.

3) The Salter osteotomy hinges upon the triradiate cartilage.

2) The Dega osteotomy lengthens the ipsilateral limb.

5) The Dega osteotomy should only be done after triradiate cartilage closure.

4) The Dega osteotomy hinges upon the medial pelvic cortex.

 

The Dega osteotomy, originally described in Poland in 1964, is an incomplete transiliac osteotomy. The medial pelvic cortex is largely preserved, and the osteotomy hinges upon this point. Both osteotomies are best performed in young children, before triradiate cartilage closure. The Salter osteotomy is stabilized by pins, so the Dega needs no internal fixation. The Salter osteotomy lengthens the limb because it is a complete osteotomy.Correct Answer: The Dega osteotomy hinges upon the medial pelvic cortex.

 

 

1961. (1101) Q5-1461:

After the bone age of 15 years, boys will achieve the most growth from which of the following regions:

 

1) Distal femoral epiphysis

3) Distal tibial epiphysis

2) Proximal tibial epiphysis

5) Spine

4) Ilium and pelvis

 

After the age of 15 in boys (13 in girls), most of the growth (4.5 cm on average) occurs in the spine.Correct Answer: Spine

 

 

1962. (1102) Q5-1462:

The most accurate way to monitor the motor tracts during spinal surgery is to stimulate which of the following regions:

 

1) Tibial nerve

3) Motor cortex

2) Peroneal nerve

5) Proximal spinal cord

4) Gastrocnemius

 

The most accurate way to monitor the motor tracts of the spinal cord is to stimulate the motor cortex. Stimulation at the level of the spinal cord conducts mainly antidromically through sensory pathways. Stimulation of the tibial and peroneal nerve is performed for monitoring sensory pathways only; these are an important indicator of spinal cord integrity but do not monitor the motor pathways per se. Stimulation of the gastrocnemius does not have any value for monitoring.Correct Answer: Motor cortex

 

 

1963. (1103) Q5-1463:

Which of the following most definitively makes the diagnosis of Sever disease:

 

1) Sclerosis of the calcaneal apophysis on x-ray

3) Tenderness over the Achilles tendon

2) Irregularity of the calcaneal apophysis

5) Limitation of subtalar motion

4) Tenderness over the calcaneal apophysis

 

There are no radiographic findings diagnostic of Sever disease. Sclerosis and irregularity are normal findings, although they are often mistakenly called evidence of disease. The diagnosis of Sever disease is made by tenderness over the calcaneal apophysis.Correct Answer: Tenderness over the calcaneal apophysis

 

A 12-year-old boy comes to your office 2 weeks after a distal radius physeal fracture, which has been splinted in the emergency department. The epiphysis is displaced dorsally by 50%, and the articular surface has a dorsal tilt of 17°. You recommend:

 

1) Closed manipulation with sedation

3) Open reduction, internal fixation

2) Closed manipulation under anesthesia with relaxation

5) Percutaneous pin fixation in current position

4) Cast application and observation

 

Distal radial physeal fractures are common injuries. Reduction should be done gently and not repeated multiple times. Fractures presenting late like this one are difficult to manipulate atraumatically, but have good remodeling potential. Therefore, a cast should be applied to limit any further displacement, but no manipulation or operation is recommended.Correct Answer: Cast application and observation

 

 

1965. (1105) Q5-1465:

Which of the following lowers the chance of a good result from stretching of muscular torticollis in infants:

 

1) Palpable mass in the sternocleidomastoid muscle

3) Rotation to neutral

2) Age at presentation less than 1 month

5) High birth weight

4) Absence of plagiocephaly

 

Stretching produces good results in more than 90% of infants. Presence of a palpable mass in the sternomastoid predicts a lower success rate than absence of such a mass. The other factors are either positive or neutral.Correct Answer: Palpable mass in the sternocleidomastoid muscle

 

 

1966. (1106) Q5-1466:

A 14-year-old boy suffers a hip dislocation in a motor vehicle accident. It is reduced by closed means. The risk of avascular necrosis is:

 

  1. Less than 5%

    3) 20%

  2. 10%

    5) 50%

    4) 40%

     

    Avascular necrosis is a risk of traumatic hip dislocation. The risk is closest to 10%.Correct Answer: 10%

     

     

    1967. (1107) Q5-1467:

    Which of the following statements is true of the King classification of idiopathic scoliosis:

     

    1. It takes into account sagittal alignment of curves.

  3. It classifies all possible curve patterns.

    1. It defines structural and nonstructural curves.

    5) It has more curve types than the Lenke system.

    4) It has a lower interobserver reliability than the Lenke system.

     

    The Lenke system is more comprehensive than the King system, for the latter considers the entire range or thoracolumbar and lumbar curve possibilities, as well as the sagittal plane. The Lenke system also appears to have a higher interobserver reliability.Correct Answer: It has a lower interobserver reliability than the Lenke system.

     

    The most common form of chondrodysplasia punctata is much more common in girls than in boys. Which of the following explains this:

     

    1) Autosomal dominant inheritance

    3) X-linked recessive inheritance

    2) X-linked dominant inheritance

    5) The imprinting phenomenon

    4) Autosomal recessive inheritance

     

    X-linked dominant inheritance would explain the preponderance of girls with this condition, because they have twice as great a chance of having an affected x-chromosome. One affected x-chromosome is sufficient to convey the disease in a dominant condition. The imprinting phenomenon refers to a condition that varies depending upon whether the mother or the father passed it on (as in Angelman versus Prader-Willi syndromes).Correct Answer: X-linked dominant inheritance

     

     

    1969. (1109) Q5-1469:

    A 9-year-old boy injures his elbow in a fall from a 12-foot height. Radiographs reveal a posterior dislocation of the elbow with a fracture of the medial epicondyle which is displaced. The ossification center is 5 mm in diameter. You recommend:

     

    1) Open reduction internal fixation with a smooth pin

    3) Open reduction and fixation with a suture

    2) Open reduction internal fixation with a small screw

    5) Closed reduction of the dislocation and application of a splint

    4) Open reduction with excision of the fragment and suture of the medial collateral ligament

     

    A recent long-term study has shown that open reduction of displaced medial epicondyle fractures does not yield superior results to closed treatment. This applies even when the elbow is initially dislocated. Excision of the epicondyle with suture of the ligament yields the highest incidence of late problems, such as flexion contractures and degenerative changes.Correct Answer: Closed reduction of the dislocation and application of a splint

     

     

    1970. (1110) Q5-1470:

    The most accurate and practical means of determining pregnancy status in adolescent females undergoing surgery is:

     

    1) Self report

    3) Serum HCG

    2) Morning urine human chorionic gonadotropin (HCG)

    5) Serum estrogen levels

    4) Serum alpha fetoprotein

     

    Anesthesia and surgery can be detrimental to a fetus, especially during the first trimester. Patient self-report is not accurate in many cases. Immunospot testing of the first morning urine for human chorionic gonadotropin is 99.4% sensitive and specific, rapid, and inexpensive. Serum HCG testing is less rapid and more expensive; therefore, not appropriate for surgical screening. The other tests are not used to ascertain pregnancy status.Correct Answer: Morning urine human chorionic gonadotropin (HCG)

     

     

    1971. (1111) Q5-1471:

    Which of the following is considered a critical element in surgically correcting posttraumatic elbow flexion contractures in adolescents:

     

    1) Lengthening of the biceps muscle

    3) Perioperative indomethacin

    2) Lengthening of the triceps muscle

    5) Postoperative continuous passive motion and physical therapy

    4) Pre- or postoperative radiation

     

    Bae and Waters have shown that adolescents with significant posttraumatic elbow flexion contractures can gain an average of 54Â

    ° of motion with surgical release. They believe postoperative physical therapy and continuous passive motion are considered critical to success of surgical release. Lengthening of the biceps or triceps is not recommended. Measures to prevent postoperative heterotopic ossification did not influence the outcome.Correct Answer: Postoperative continuous passive motion and physical therapy

     

    Which of the following statements is true concerning atlanto-occipital dislocations in children:

     

    1) The dens-basion distance is greater than 10 mm.

    3) The injury is not survivable.

    2) The power ratio is greater than 1.0.

    5) No single plain radiographic finding is always diagnostic.

    4) Spinal cord injury always accompanies this dislocation.

     

    Pediatric atlanto-occipital dislocation is increasing in frequency due to improved emergency care. Although the injuries are sometimes fatal and are often accompanied by severe spinal cord injury, this is not always the case. Although the dens-basion distance should be less than 10 mm and the power ratio should be less than 1 in normals, these are not always abnormal in children with such injuries. Clinical and radiographic correlation, with computer tomography or magnetic resonance imaging if needed, are called for to maximize diagnosis.Correct Answer: No single plain radiographic finding is always diagnostic.

     

     

    1973. (1113) Q5-1473:

    Equinovarus positioning of the foot is normal during which stage of embryonic life:

     

    1) No stage

    3) Thirteenth to fifteenth week

    2) Eighth to tenth week

    5) Twentieth to twenty-third week

    4) Seventeenth to nineteenth week

     

    As the foot matures, it passes through a normal stage when it resembles a clubfoot in the eighth to tenth week. After this, the foot normally corrects itself.Correct Answer: Eighth to tenth week

     

     

    1974. (1114) Q5-1474:

    Which of the following is a similarity between congenital pseudarthrosis of the clavicle and congenital pseudarthrosis of the tibia:

     

    1) Both are common in neurofibromatosis

    3) Both have a low rate of union after treatment with autograft unless it is vascularized

    2) Both are common in cleidocranial dysplasia

    5) Both may present with tapered, atrophic bone ends at the pseudarthrosis

    4) Both are more common on the left side

     

    Congenital pseudarthrosis of the clavicle and tibia may present in infants with a gap between two tapered, atrophic bone ends. However, they are dissimilar in other respects. Pseudarthrosis of the clavicle is seen almost exclusively on the right side, while that of the tibia is seen on either side. Pseudarthrosis of the clavicle may be seen in cleidocranial dysostosis, although tibial pseudarthrosis is not. Pseudarthrosis of the clavicle has a high rate of union with simple bone graft, while that of the tibia does not. One-half of patients with pseudarthrosis of the tibia have neurofibromatosis, while this is almost never seen in congenital pseudarthrosis of the clavicle.Correct Answer: Both may present with tapered, atrophic bone ends at the pseudarthrosis

     

     

    1975. (1115) Q5-1475:

    A 13-year-old boy has a left slipped capital femoral epiphysis which has displaced 75%. He is unable to bear weight on the limb. The other hip has no clinical or radiographic abnormalities. Your preferred treatment is which of the following:

     

    1) Gentle reduction of the slip and fixation with a cannulated screw

    3) Cuneiform osteotomy

    2) Hip spica cast

    5) Free vascularized fibular grafting

    1. Subtrochanteric osteotomy

       

      This is an unstable slip. It has a much higher chance of avascular necrosis than a stable slip. Since the degree of the slip will increase the shear forces across the healing physis and decrease the function of the hip, some method of improving this is justified. Gentle reduction of the epiphysis, without forceful internal rotation, may increase the risk of avascular necrosis.

      Avascular necrosis is a significant risk in many series of cuneiform (metaphyseal; Fish) osteotomies. Subtrochanteric osteotomy is not justified in patients with an acute slip until it is healed. It has a high rate of chondrolysis. Free vascularized bone graft may be an option if avascular necrosis develops, but is not indicated at this time. A hip spica cast is also often followed by chondrolysis and delayed epiphyseodesis.Correct Answer: Gentle reduction of the slip and fixation with a cannulated screw

       

      Of all slipped capital femoral epiphyses, which percentage is unstable:

       

      1) 5%

      3) 25%

      2) 15%

    2. 50%

  4. 35%

     

    Unstable slipped capital femoral epiphysis places the patient at a high risk of avascular necrosis (up to 47%). Fortunately, it comprises only about 5% of all slips.Correct Answer: 5%

     

     

    1977. (1117) Q5-1477:

    The normal value for the hallux valgus angle is:

     

    1) 0° to 5°

    3) 10° to 20°

    2) 5° to 10°

    5) 30° to 40°

    4) 20° to 30°

     

    The angle between the first metatarsal and its proximal phalanx is normally one of mild (10° to 20°) valgus. It is not normal for it to be too straight. An increase in this angle beyond this value is often noted by the patient as a bunion.Correct Answer: 10° to 20°

     

     

    1978. (1118) Q5-1478:

    A 13-year-old girl is seen in clinic for bunion. She is asymptomatic but has a hallux valgus angle of 29°, an intermetatarsal angle of 15°, and a medial prominence over the first metatarsal head. The family asks whether anything can be done to prevent future problems with the foot. You recommend:

     

    1) Osteotomy of the first metatarsal base

    3) Double osteotomy of the first metatarsal

    2) Hemiepiphyseodesis of the medial physis of the first metatarsal

    5) Shoe modifications if symptoms develop

    4) Mitchell osteotomy

     

    Bunions may often be treated conservatively, and it is impossible to predict which ones will later develop symptoms. Surgical reconstruction of bunions in adolescents has a higher rate of recurrence than in adults in many reported series. For all of these reasons, nonoperative treatment is preferred for asymptomatic patients.Correct Answer: Shoe modifications if symptoms develop

     

     

    1979. (1119) Q5-1479:

    Which of the following tendons is not usually contracted in a patient with untreated vertical talus:

     

    1) Anterior tibialis

    3) Peroneus brevis

    2) Posterior tibialis

    5) Achilles

    4) Extensor digitorum longus

     

    A vertical talus is a fixed dorsolateral dislocation of the talonavicular joint. The forefoot is in calcaneus and the hindfoot is in equinus. Therefore, all of the tendons listed except for the posterior tibialis are contracted.Correct Answer: Posterior tibialis

     

    A 9-year-old boy is seen because of pain medially, in the arch of the foot. His ankle dorsiflexion is limited to 10° with the knee extended. Radiograph shows an accessory navicular, which corresponds to the point of his tenderness. You recommend:

     

    1) Excision of the accessory navicular

    3) Evans procedure (lateral column lengthening)

    2) Excision of the accessory navicular with advancement of the posterior tibialis tendon

    5) Activity restriction, stretching, arch support

    4) Tendoachilles lengthening

     

    Accessory navicular is seen in 10% to 14% of normal children. Sometimes, it becomes symptomatic in juveniles or adolescents, but this usually resolves by skeletal maturity. Conservative treatment, such as activity restriction, arch support, and stretching the Achilles if tight, will usually alleviate symptoms.Correct Answer: Activity restriction, stretching, arch support

     

     

    1981. (1121) Q5-1481:

    Which of the following disorders is due to a defect in anterior horn cells:

     

    1) Charcot-Marie-Tooth

    3) Friedreichâs ataxia

    2) Duchenne dystrophy

    5) Rett syndrome

    4) Spinal muscular atrophy

     

    Charcot-Marie-Tooth disease is due to a defect in peripheral nerves; Duchenne muscular dystrophy is due to a defect in dystrophin, affecting the muscle cell membrane; Friedreich ataxia is a degeneration of the spinocerebellar tracts. Rett syndrome is due to a defect in MECP-2 protein, affecting the brain. Only spinal muscular atrophy is due to a defect in anterior horn cells.Correct Answer: Spinal muscular atrophy

     

     

    1982. (4041) Q5-1482:

    Becker muscular dystrophy is due to a defect in the gene for which of the following:

     

    1) Dystrophin

    3) Myelin

    2) Sarcoglycan

    5) Fibroblast growth factor receptor

    4) Sulfate transport

     

    Becker muscular dystrophy has a defect in the same gene as Duchenne muscular dystrophy, namely the protein dystrophin. The mutation in Becker dystrophy results in a truncated protein that retains some function, whereas the mutation in Duchenne dystrophy is different and results in an unstable protein, which is degraded quickly.Correct Answer: Dystrophin

     

     

    1983. (1122) Q5-1483:

    Which of the following muscles is first affected in facioscapulohumeral dystrophy:

     

    1) Orbicularis oris

    3) Deltoid

    2) Serratus anterior

    5) Infraspinatus

    4) Supraspinatus

     

    In facioscapulohumeral dystrophy, the facial muscles are affected first, commonly presenting with an inability to whistle. The serratus muscles and scapular stabilizers are affected next. The deltoid, supraspinatus, and infraspinatus are typically not affected in this disease.Correct Answer: Orbicularis oris

     

    Emery-Dreifuss muscular dystrophy is unique among the dystrophies because of the development of which deformity:

     

    1) Neck extension contracture

    3) Knee flexion

    2) Hip abduction

    5) Scoliosis

    4) Equinus contracture

     

    Emery-Dreifuss muscular dystrophy is an x-linked disorder of emerin, which is a cell-membrane protein. Symptoms and signs develop within the first decade in most cases. A unique deformity, neck extension contracture, develops, in addition to elbow flexion contractures and peroneal wasting. The other deformities listed above are common in many dystrophies.Correct Answer: Neck extension contracture

     

     

    1985. (1124) Q5-1485:

    Electrodiagnostic testing in myopathies typically shows all of the following except:

     

    1) Low amplitude electromyogram (EMG) potentials

    3) Normal nerve conduction velocity

    2) Polyphasic EMG potentials

    5) Fibrillation potentials

    4) Decreased duration of EMG response

     

    Electrodiagnostic testing in myopathy typically shows low amplitude, polyphasic EMG potentials with a decreased duration of response. Nerve conduction velocity is normal, in contrast to findings in neuropathies. Fibrillations are not typically seen; these are more characteristic of neuropathy.Correct Answer: Fibrillation potentials

     

     

    1986. (1125) Q5-1486:

    In which region is direct anatomical extension from the metaphysis of a long bone to the adjacent joint not anatomically possible in the child:

     

    1) Shoulder

    3) Hip

    2) Elbow

    5) Ankle

    4) Knee

     

    The metaphysis of the proximal humerus lies partially within the shoulder joint; similarly, that of the proximal radius lies within the elbow. The metaphysis of the proximal femur lies within the hip joint and that of the distal lateral tibia within the ankle joint. There is no intra-articular metaphysis about the knee, however.Correct Answer: Knee

     

     

    1987. (1126) Q5-1487:

    Which of the following skeletal dysplasias is not commonly associated with non-orthopedic complications:

     

    1) McKusick metaphyseal chondrodysplasia

    3) Morquio syndrome

    2) Hurler syndrome

    5) Multiple epiphyseal dysplasia

    4) Chondroectodermal dysplasia

     

    McKusick dysplasia is commonly associated with immune and gastrointestinal disorders. Hurler syndrome is associated with progressive mental retardation, hepatosplenomegaly, and cataracts. Morquio syndrome is associated with cardiorespiratory difficulties. Chondroectodermal dysplasia, or Ellis van Creveld syndrome, is associated with congenital heart disease. Multiple epiphyseal dysplasia, however, is not associated with systemic non-orthopaedic complications.Correct Answer: Multiple epiphyseal dysplasia

     

    Which of the following by itself is not an indication for surgery in a child with acute hematogenous osteomyelitis:

     

    1) Fever higher than 38.5°

    3) Presence of a sequestrum

    2) Subperiosteal abscess

    5) Adjacent septic arthritis

    4) Intramedullary abscess

     

    Surgery is indicated in situations in which antibiotics alone will not be curative, including subperiosteal abscess, sequestrum, intramedullary abscess, and adjacent septic arthritis. Fever alone is not an indication for surgery.Correct Answer: Fever higher than 38.5°

     

     

    1989. (1128) Q5-1489:

    Which of the following is not usually seen in chronic recurrent multifocal osteomyelitis:

     

    1) Positive cultures for Staphylococcus epidermidis

    3) Involvement of the spine, long bones, and feet

    2) Gradual onset of symptoms

    5) Improvement with nonsteroidal anti-inflammatory agents

    4) Negative cultures

     

    Chronic recurrent multifocal osteomyelitis is believed to be an idiopathic noninfectious inflammatory disease. It has gradual onset of symptoms, and sites most commonly involved are the spine, long bones, and feet. Nonsteroidal anti-inflammatory drugs typically improve symptoms. Cultures are negative.Correct Answer: Positive cultures for Staphylococcus epidermidis

     

     

    1990. (1129) Q5-1490:

    A 5-year-old child is bitten by a tick. Which of the following has been shown to aid in management:

     

    1) Prompt tick removal

    3) Immediate treatment with amoxicillin

    2) Immediate treatment with doxycycline

    5) Steroid administration

    4) Prompt ELISA testing

     

    Prompt tick removal is recommended because Lyme disease is more likely if the tick is attached for more than 24 hours. Immediate antibiotic administration is not recommended because the incidence of Lyme disease is low after any single tick bite and treatment is equally effective once the disease is diagnosed. Doxycycline is not recommended in children younger than 8 years old because of dental discoloration. Immediate testing for antibodies (ELISA) is not useful because antibodies do not rise for at least several weeks. Steroids are not recommended in this setting.Correct Answer: Prompt tick removal

     

     

    1991. (1130) Q5-1491:

    Which of the following measures has not been shown to decrease rates of injury in healthy children participating in recreational sports:

     

    1) Knee braces during basketball and football

    3) Helmets for bicyclists

    2) Ankle braces in basketball

    5) Break-away bases for baseball

    4) Mouth guards for basketball

     

    Knee braces have been shown not to reduce injury rates for children for children with sound knees. All other measures have been shown to reduce injury rates.Correct Answer: Knee braces during basketball and football

     

    Which recreational activity causes the most musculoskeletal injuries in children ages 5 to 14 in the United States:

     

    1) Bicycles

    3) Soccer

    2) Football

    5) Gymnastics

    4) Trampolines

     

    Bicycles are the leading cause of musculoskeletal injury in American children, with 415,000 injuries per year, followed by basketball, football, and roller sports.Correct Answer: Bicycles

     

     

    1993. (1173) Q5-1538:

    The effects of pediatric orthopedic conditions in later adulthood commonly determine treatment choices for children. At what threshold does limb length discrepancy increase the energy cost of walking in older adults:

     

    1) 2 cm

    3) 4 cm

    2) 3 cm

    5) No such effect has been proven at any discrepancy.

    4) 5 cm

     

    A limb length discrepancy of as little as 2 cm has shown to increase perceived exertion as well as oxygen consumption in older adults.Correct Answer: 2 cm

     

     

    1994. (1174) Q5-1539:

    Anterior elbow release in children with cerebral palsy is likely to result in which of the following outcomes:

     

    1) Decreased flexion posture during use

    3) Increased use during bimanual activity

    2) Decreased flexion contracture

    5) Increased grip strength

    4) Increased strength of elbow flexion

     

    Anterior elbow release consists of lengthening of the lacertus fibrosus and the brachialis fascia. It may or may not include lengthening of the biceps tendon itself. Anterior elbow release effectively decreases the excessive flexion posture of the elbow during use, which one author has termed the âflexion posture angle.â It does not result in decreased (or increased) strength of elbow flexion if the biceps tendon is preserved. Unfortunately, increased use during bimanual activity and increased grip strength are usually not observed.Correct Answer: Decreased flexion posture during use

     

     

    1995. (1175) Q5-1540:

    Which of the following procedures is most likely to correct idiopathic toe walking with a single treatment:

     

    1) Stretching program

    3) Stretching cast

    2) Ankle foot orthosis

    5) Percutaneous tendoachilles lengthening

    4) Botulinum toxin injection

     

    Percutaneous tendoachilles lengthening is most likely to resolve idiopathic toe walking in a single treatment. The other methods have a higher likelihood of persistent toe walking.Correct Answer: Percutaneous tendoachilles lengthening

     

     

     

    1) Cbfa1 induces cells to differentiate into osteoblasts.

    3) Cbfa1 inhibits endochondral ossification.

    2) Cbfa1 induces cells to differentiate into osteoclasts.

    5) Cbfa1 causes premature cell death.

    4) Cbfa1 impairs sulfation of proteoglycans.

     

    Cbfa1 is a transcription factor that causes cells to differentiate into osteoblasts. An abnormality in its gene causes cleidocranial dysplasia.Correct Answer: Cbfa1 induces cells to differentiate into osteoblasts.

     

     

    1997. (1177) Q5-1542:

    Which of the following is found less often in children with lumbosacral agenesis as compared to controls:

     

    1) Cervical spine anomalies

    3) Hip dislocation

    2) Maternal diabetes

    5) Genu recurvatum

    4) Spina bifida

     

    Patients with lumbosacral agenesis often have knee flexion contractures as compared with controls. All of the other features listed are common in patients with lumbosacral agenesis.Correct Answer: Genu recurvatum

     

     

    1998. (1178) Q5-1543:

    Which of the following figures most closely approximates the prevalence of defects in the L5 pars interarticularis in a newborn:

     

    1. Less than 1%

      3) 5%

    2. 3%

  5. 15%

4) 10%

 

Pars interarticularis defects are not found in newborns, whereas the incidence is 5% in patients who are in the first grade. It remains close to this figure throughout later life.Correct Answer: Less than 1%

 

 

1999. (1179) Q5-1544:

Which of the following is the most common cause of low back pain in young athletes:

 

  1. Thoracolumbar Scheuermann apophysitis

3) Slipped vertebral apophysis

2) Herniated nucleus pulposus

5) Avulsion of the spinous apophysis

4) Spondylolysis

 

Spondylolysis is the most common cause of back pain in young athletes, accounting for approximately 50% of cases. The other causes are significantly less common.Correct Answer: Spondylolysis

 

without flattening. He has had pain in the groin for the past 6 months. Recommended treatment includes:

 

1) Adductor tenotomy

3) Proximal femoral resection

2) Adductor tenotomy and femoral osteotomy

5) Total hip arthroplasty

4) Hip arthrodesis

 

Pain in the spastic patient with subluxation without head deformity is most appropriately treated by reduction. Adductor tenotomy alone is not recommended when the hip subluxation exceeds 50% or the age is greater than 5 to 6 years. Femoral osteotomy should be added. Salvage procedures such as proximal femoral resection, arthrodesis, or arthroplasty are not indicated.Correct Answer: Adductor tenotomy and femoral osteotomy

 

 

2001. (1181) Q5-1546:

The Ponseti method of clubfoot cast treatment starts with which of the following steps:

 

1) Pronation of the forefoot

3) Dorsiflexion of the ankle

2) Dorsiflexion of the first ray

5) Internal rotation of the foot

4) External rotation of the foot

 

Dorsiflexion of the first ray is the first step in the Ponseti method of cast treatment. Dorsiflexion decreases the cavus component.

 

 

Pronation of the forefoot is the opposite of dorsiflexion and produces the opposite desired motion. Dorsiflexion of the ankle should be performed after the Achilles tenotomy.

 

 

External rotation of the foot is performed later. Internal rotation of the clubfoot is never done.

Correct Answer: Dorsiflexion of the first ray

 

 

2002. (1182) Q5-1547:

When correcting a clubfoot by the Ponseti method, the lateral mold on the foot must be placed against which of the following structures:

 

1) The head of the talus

3) The cuboid

2) The calcaneus

5) The tibia

4) The fifth metatarsal

 

Ponseti describes rotating the clubfoot against the head of the talus to allow the lateral side of the foot (calcaneus and cuboid) to rotate laterally.Correct Answer: The head of the talus

 

 

2003. (1183) Q5-1548:

Which of the following molecules is defective in osteogenesis imperfecta:

 

1) Type I collagen

3) Calcium transport channels

2) Type II collagen

5) Renal tubular reabsorption of phosphate

4) Vitamin D 1,25-dihydroxylase

 

Osteogenesis imperfecta (OI) is a disorder of type I collagen. Numerous different mutations in the genes for this molecule have been described, accounting for the variable clinical phenotypes. Type II collagen is important for articular cartilage, but it is not implicated in the pathogenesis of OI. The other factors are not abnormal in OI.Correct Answer: Type I collagen

 

 

 

1) Fibroblast growth factor gene

3) Platelet-derived growth factor gene

2) Homeobox genes

5) Sulfate transport gene

4) Leptin gene

 

Proximal-to-distal development of the limbs in utero is determined by the homeobox genes. These genes are located on a number of different chromosomes (2, 7, 12, and 17), and they guide the proximal-to-distal organization of limb and digital development.Correct Answer: Homeobox genes

 

 

2005. (1185) Q5-1550:

Meryon sign refers to which of the following physical phenomena in patients with muscular dystrophies:

 

1) Wide-based gait

3) Tendency to âslip throughâ when patient is suspended under the axillae

2) Use of upper extremities in coming to stand

5) Inability to fully flex the neck

4) Absence of facial expression

 

Meryon sign is the weakness of shoulder adduction when a child is lifted or suspended under the axillae. The examiner will feel that the child is slipping through his or her hands. It is due to weakness of the shoulder girdle muscles. Meryon sign is present in limb-girdle dystrophies and fascioscapulohumeral dystrophy.Correct Answer: Tendency to âslip throughâ when patient is suspended under the axillae

 

 

2006. (1186) Q5-1551:

Spinal muscular atrophy is best characterized as which of the following:

 

1) An upper motor neuropathy

3) A sensory neuropathy

2) A lower motor neuropathy

5) A generalized myopathy

4) A peripheral neuropathy

 

Spinal muscular atrophy is a degeneration of the anterior horn cells of the spinal cord. It results in a nonprogressive lower motor neuron disease with preservation of sensation and intelligence. There is no spasticity or hyperreflexia.Correct Answer: A lower motor neuropathy

 

 

2007. (1187) Q5-1552:

Which of the following is the gene that is abnormal in spinal muscular atrophy:

 

1) Survival motor neuron genes 1 and 2

3) Gene for frataxin

2) Gene for dystrophin

5) Gene for emerin

4) Gene for peripheral myelin protein

 

Spinal muscular atrophy is a disorder of survival motor neuron genes 1 and 2; its product is not yet known.

 

 

Dystrophin is abnormal in Duchenne and Becker muscular dystrophy. Frataxin is abnormal in Friedreich ataxia.

 

 

Peripheral myelin protein is defective in Charcot-Marie-Tooth disease. Emerin is abnormal in Emery-Dreifuss syndrome.

Correct Answer: Survival motor neuron genes 1 and 2

 

 

 

1) Heel cord contractures

3) Limited cervical flexion

2) Elbow contractures

5) Cardiomyopathy

4) Scoliosis

 

Emery-Dreifuss syndrome is due to an encoding error in emerin, which may be a stabilizer of the nuclear membrane. Emery-Dreifuss syndrome is characterized by a triad of contractures of the heel cord, elbow, and cervical spine. Cardiomyopathy may cause heart block or other arrhytmias. Scoliosis is not commonly found in this condition.Correct Answer: Scoliosis

 

 

2009. (1189) Q5-1554:

Which of the following bones of the foot is normally ossified at birth:

 

1) Cuboid

3) Third cuneiform

2) First cuneiform

5) First metatarsal

4) Navicular

 

The metatarsals are ossified at birth, along with the talus and the calcaneus.

 

 

The cuboid ossifies at 1 month, followed by the third, second, and first cuneiforms. The navicular does not ossify until age 2 or 3 years.

These facts are useful when interpreting radiographs for congenital foot deformities such as clubfoot. The location of the navicular must often be inferred from the position of the first metatarsal.Correct Answer: First metatarsal

 

 

 

2010. (1190) Q5-1555:

Chronic recurrent multifocal osteomyelitis is caused by which of the following:

 

1) Staphylococcus epidermidis

3) Spirochetal infection

2) Corynebacterium

5) No organism has been isolated

4) Viral infection

 

Chronic recurrent multifocal osteomyelitis presents as chronic joint pain at multiple locations and at different times. No organism has been isolated from patients presenting with chronic recurrent multifocal osteomyelitis. There is no role for surgery or antibiotics because symptoms generally resolve over a period of time.Correct Answer: No organism has been isolated

 

 

2011. (1191) Q5-1556:

A 5-year-old boy has midfoot pain and limps at the end of long walks. Radiographs show sclerosis and fragmentation of the navicular on the involved side. Recommended treatment is:

 

1) Core decompression

3) Electrical stimulation

2) Activity modification

5) Talonaviculocuneiform fusion

4) Bone grafting of the ossific nucleus of the navicular

 

Kohlerâs disease, or avascular necrosis of the tarsal navicular, occurs spontaneously. It is more common in boys than girls, and it frequently presents before the age of 8 years. Conservative treatment consisting of counseling and activity modification is usually sufficient, with reossification ensuing. Occasionally cast immobilization seems helpful in allaying symptoms.Correct Answer: Activity modification

 

Radiographs show increased density and flattening of the metatarsal head. Recommended treatment is:

 

1) Core decompression

3) Bone graft of the epiphysis of the second metatarsal

2) Decreased activity and use of orthotics

5) Osteotomy of the second metatarsal neck

4) Fusion of the metatarsophalangeal joint in a functional position

 

Freibergâs disorder, or osteochondrosis of the second metatarsal head, is most common in teenage girls, especially dancers. The length of this metatarsal may be a factor in the pathogenesis. Treatment must be conservative in most cases, although a mild degree of symptoms may persist if epiphyseal flattening does not remodel.Correct Answer: Decreased activity and use of orthotics

 

 

2013. (1193) Q5-1558:

Which of the following complications is not a recognized risk of the Salter osteotomy:

 

1) Recurrent subluxation

3) Pin infection

2) Pin migration

5) Migration of the bone graft

4) Avascular necrosis

 

Avascular necrosis is not a risk of the osteotomy, but rather of an open or closed reduction that may sometimes accompany it. All of the other complications are accepted as possible, although rare, sequelae of the procedure.Correct Answer: Avascular necrosis

 

 

2014. (1194) Q5-1559:

The result of treatment of developmental dysplasia of the hip with Salter osteotomy is is worse with which of the following:

 

1) A higher degree of the dislocation before treatment (Tonnis grade)

3) Higher preoperative acetabular index

2) Bilaterality

5) Performing open reduction at a separate surgery than the osteotomy

  1. Increased age at surgery

     

    Salter osteotomy is effective in treating developmental dysplasia of the hip in young children. The result is worse with higher degrees of dislocation as assessed by the Tonnis system. It is better if the open reduction (if needed) is performed as a separate step than the osteotomy. The other factors have not been shown to be predictive.Correct Answer: A higher degree of the dislocation before treatment (Tonnis grade)

     

     

    2015. (1195) Q5-1560:

    A 4-year-old girl with developmental hip dysplasia is advised to have a Salter innominate osteotomy. When the family asks about the long-term survivorship of the reconstruction, the surgeon tells them that good 30-year follow-up results are likely in at least what percentage of patients:

     

    1) 30%

    3) 70%

    2) 50%

  2. 98%

4) 85%

 

Thirty-year survivorship analysis shows good to excellent results in at least 85% of patients.Correct Answer: 85%

 

 

 

  1. Leptin increases bone formation.

3) Leptin decreases bone formation.

2) Leptin increases muscle mass.

5) Leptin increases longitudinal growth.

4) Leptin decreases muscle mass.

 

Leptin is a polypeptide secreted by adipocytes and acts upon the hypothalamus as a powerful inhibitor of bone mass.Correct Answer: Leptin decreases bone formation.

 

 

2017. (1197) Q5-1562:

Which of the following organisms is the most common cause of obturator internus muscle abscess in children:

 

1) Streptococcus A

3) Salmonella

2) Staphylococcus aureus

5) Pseudomonas aeruginosa

4) Escherichia coli

 

Abscess of the obturator internus muscle may mimic septic arthritis. It is best diagnosed by magnetic resonance image.

<1>Staphylococcus aureus is the most common causative organism, accounting for 75% of cases of obturator internus muscle abscess. Antibiotic treatment should be tried first and is successful in most cases.Correct Answer: Staphylococcus aureus

 

 

2018. (1198) Q5-1564:

Which of the following studies is likely to help in distinguishing osteomyelitis from infarct in a patient with sickle cell anemia:

 

1) Magnetic resonance imaging

3) Bone and bone marrow scans

2) Computed tomography

5) Plain radiographs

4) Ultrasound

 

The combination of bone and bone marrow scan is the only imaging method that is useful in distinguishing osteomyelitis from infarct in patients with sickle cell anemia. The bone marrow scan is normal, but the bone scan shows increased uptake in a patient with osteomyelitis.Correct Answer: Bone and bone marrow scans

 

 

2019. (1199) Q5-1565:

A 10-year-old boy with diplegic cerebral palsy walks with his knees turned in significantly. He has the appearance of severe valgus when walking. When examined in a supine position, there is no excessive valgus of the knees. His popliteal angle is 45°. An Ely test is negative. His hip internal rotation in the prone position is 80°, while his external rotation is 15°. The surgeon wishes to improve the patientâs knee position during gait. The intervention most likely to accomplish this is:

 

1) Botulinum toxin injection to the hamstrings

3) Hamstring lengthening, medially and laterally

2) Hamstring lengthening, medially

5) Femoral derotation osteotomy

4) Rectus transfer into the biceps

 

The findings highlighted here are those of severe anteversion. Anteversion causes the appearance of valgus of the knees, and it does not resolve spontaneously in cerebral palsy. The procedure most likely to make a lasting improvement in the patient is derotational osteotomy of the femur.Correct Answer: Femoral derotation osteotomy

 

 

 

1) Medial hamstring lengthening

3) Rectus transfer to the hamstrings

2) Medial and lateral hamstring lengthening

5) Adductor lengthening

4) Tendoachilles lengthening

 

Lengthening of the medial and lateral hamstrings is more likely to overlengthen the posterior knee checkrein. Therefore, it should only be performed in selected cases with severe spasticity and no cospasticity of the rectus femoris. Lengthening of only the medial hamstrings carries less risk. The other procedures listed do not carry this risk.Correct Answer: Medial and lateral hamstring lengthening

 

 

2021. (1201) Q5-1567:

A 7-year-old boy has short stature, unilateral coxa vara, and lack of ossification in the medial pubic rami. The most likely diagnosis is:

 

1) Achondroplasia

3) Osteogenesis imperfecta

2) Diastrophic dysplasia

5) Multiple epiphyseal dysplasia

4) Cleidocranial dysplasia

 

Cleidocranial dysplasia produces the above findings, in addition to partial or complete lack of formation of the clavicles and persistent widening of the cranial fontanelles. None of these findings are present in the other conditions.Correct Answer: Cleidocranial dysplasia

 

 

2022. (1362) Q5-1732:

An 11-year-old girl injures her knee while playing lacrosse and develops a hemarthrosis. The most likely diagnosis is:

 

1) Anterior cruciate ligament tear

3) Physeal injury of the distal femur

2) Patellar dislocation

5) Medial meniscus tear

4) Tibial spine avulsion

 

The most common cause of hemarthrosis of the knee in skeletally immature patients is a patellar dislocation. Anterior cruciate ligament injuries and tibial spine avulsions are less common. Medial meniscus tears are extremely rare in this population.Correct Answer: Patellar dislocation

 

 

2023. (1363) Q5-1733:

In order to see the articular cartilage of the knee in a child who has knee trauma, a magnetic resonance image must include:

 

1) T1-weighted images

3) Gradient echo sequences

2) T2-weighted images

5) Flexion and extension images

4) Gadolinium contrast

 

Articular cartilage may be injured during trauma to the knee and manifest as either osteochondral fractures or osteochondritis dissecans. The articular cartilage is best visualized using gradient echo sequences. Examples of this technique include fast low-angle shot (FLASH) imaging, fast imaging with steady precession (FISP), and short tau inversion recovery (STIR).Correct Answer: Gradient echo sequences

 

thoracic pedicle on the concave side is:

 

1) Less than 1 mm

3) 3 mm

2) 2 mm

5) 5 mm

4) 4 mm

 

The distance between the apical thoracic pedicle and the thecal sac is less than 1 mm on the concave side.Correct Answer: Less than 1 mm

 

 

2025. (1365) Q5-1735:

Which region (vertebral body) of the spine is the closest to the aorta:

 

1) T4

3) T1

2) T9

5) T12

4) T6

 

The aorta is on the left side of the vertebra in the upper and midthoracic spine. The aorta moves to an anterior location in the lower thoracic spine. The distance from the aorta to the vertebral body is 6 mm to 7 mm in skeletally mature patients with idiopathic scoliosis between the fourth and ninth vertebral bodies. The distance becomes less than 5 mm in the thoracolumbar junction and lumbar spine. The aortic arch does not extend to the first thoracic vertebral.Correct Answer: T12

 

 

2026. (1366) Q5-1736:

The width of the pedicles in a patient with idiopathic scoliosis in the surgical range is narrowest in the:

 

1) Thoracic spine on the convex side

3) Sacral spine

2) Lumbar spine on the convex side

5) Lumbar spine on the concave side

4) Thoracic spine on the concave side

 

The width of the pedicles is less in the thoracic spine than in the lumbar or sacral spine, and less on the concavity than on the convexity. The mean width of the thoracic pedicles on the concave side at the apex in skeletally mature patients is only 3 mm.Correct Answer: Thoracic spine on the concave side

 

 

2027. (1367) Q5-1737:

The angle of the pedicle with the midsagittal plane at T11 is closest to:

 

1) 0°

3) 15°

2) 5°

5) 25°

4) 20°

 

The angle of the pedicles is greatest in the upper thoracic and lumbar spines (approximately 15°). The angle decreases to approximately 7° at the thoracolumbar junction.Correct Answer: 5°

 

 

 

1) Radial head subluxation or dislocation

3) Decreased slope of the distal radial articular surface

2) Limitation of rotation

5) Decreased length of the ulna

4) Subluxation of the carpus toward the ulna

 

Multiple hereditary exostosis is characterized by differential growth of the two bones. There is often decreased length of the ulna. This results in increased inclination of the distal radial articular surface, radial head subluxation, and subluxation of the carpus toward the ulna. The exostoses themselves may block rotation.Correct Answer: Decreased slope of the distal radial articular surface

 

 

2029. (1368) Q5-1739:

Which of the following factors has proven to increase the risk of degenerative disease of the wrist in patients with multiple hereditary osteochondromas:

 

1) Ulnar negative variance of 5 mm

3) Distal radial articular slope larger than 25°

2) Ulnar negative variance of 10 mm

5) None of the above

4) Subluxation of the radial head

 

The above findings are all more common in the forearms and wrists of patients with multiple hereditary exostoses than in the general population. However, there is no parameter shown to increase the risk of degenerative wrist disease.Correct Answer: None of the above

 

 

2030. (1369) Q5-1740:

Which adverse outcome is most common in adults undergoing periacetabular osteotomy for hip dysplasia:

 

1) Sciatic nerve palsy

3) Nonunion of the osteotomy

2) Femoral nerve palsy

5) Heterotopic ossification

4) Persistent or worsening joint pain

 

Heterotopic ossification occurs in fewer than 5% of patients undergoing various types of periacetabular osteotomy. Femoral nerve palsy and sciatic nerve palsy occur in only 1% to 2% of patients. Nonunion is rare in this region because of abundant cancellous bone contact. Persistent or worsening joint pain is the most frequent adverse outcome and is least common in those patients with preoperative degenerative changes.Correct Answer: Persistent or worsening joint pain

 

 

2031. (1370) Q5-1741:

An 18-year-old woman complains of pain in her groin area. An anteroposterior radiograph reveals a dysplastic hip with a center-edge angle of 0°. The femoral head is spherical and centers better on abduction. There is a cyst in the superior acetabulum.

The joint space is narrowed superiorly by 1 mm. Recommended treatment includes:

 

1) Salter innominate osteotomy

3) Pemberton osteotomy

2) Total hip arthroplasty

5) Periacetabular osteotomy

4) Hip arthrodesis

 

This patient has a high likelihood of progressive hip deterioration so surgery is warranted. The periacetabular osteotomy is preferred because the procedure can correct this high degree of hip dysplasia. The Salter osteotomy, which hinges the hemipelvis on the symphysis pubis, cannot reliably correct more than 10° of dysplasia in this age group. The Pemberton osteotomy relies on plasticity of the immature skeleton to âfoldâ the pelvis on an open triradiate cartilage, therefore, it is not indicated in an 18-year-old woman who is skeletally mature. Hip arthrodesis is not indicated when there are other, better options. The same can be said for total hip arthroplasty due to the finite duration of fixation in a young person.Correct Answer: Periacetabular osteotomy

 

A high school basketball player dies suddenly during a game. Which of the following is the least likely cause of death:

 

1) Hypertrophic cardiomyopathy

3) Aortic dissection from Marfan syndrome

2) Prolonged Q-T interval

5) Substance abuse

4) Coronary artery anomalies

 

Hypertrophic cardiomyopathy, coronary artery anomalies, and electrical abnormalities are the most common causes of sudden death in athletes. Substance abuse is also common. Marfan syndrome, which can be heralded by skeletal features of arachnodactyly, should also be considered when screening athletes although it is not as likely as the other choices to cause death.Correct Answer: Aortic dissection from Marfan syndrome

 

 

2033. (1372) Q5-1743:

A 15-year-old girl has anterior hip pain and she tells you that she hears periodic snapping or clicking. Bringing the hip from the flexed-abducted position to the extended position reproduces the pain. Radiographs are normal. The diagnosis is most likely:

 

1) Trochanteric bursitis

3) Snapping psoas tendon

2) Acetabular dysplasia

5) Femoral hernia

4) Torn acetabular labrum

 

Snapping of the psoas tendon is more common in girls than boys. A snapped psoas tendon is characterized by anterior hip pain that can be reproduced by moving the hip from a figure 4 position to an extended position. The discomfort from trochanteric bursitis is located laterally. The symptoms of an abnormality in the labrum or the acetabulum are not associated with snapping.Correct Answer: Snapping psoas tendon

 

 

2034. (1373) Q5-1744:

A 15-year-old girl has a snapping psoas tendon. Abducting and adducting her hip reproduces symptoms of anterior hip pain. Her pain has not been relieved by rest and stretching. The next step of treatment is:

 

1) Excising the bursa

3) Intramuscular lengthening of the tendon

2) Releasing the hip from the lesser trochanter

5) Excising the pectineal eminence

4) Transferring the psoas tendon to the neck of the femur

 

The initial treatment of snapping psoas tendon is rest, antiinflammatory agents, and stretching. If symptoms remain significant, the next measure to offer the patient is intramuscular lengthening of the tendon.Correct Answer: Intramuscular lengthening of the tendon

 

 

2035. (1374) Q5-1745:

A 4-year-old girl is newly diagnosed with developmental dislocations of the hips. The femoral heads are fully dislocated and located 4 cm above the acetabulum. No pseudoacetabulum is seen. Recommended treatment includes:

 

1) No treatment

3) Open reduction through a medial approach

2) Traction and closed reduction

5) Open reduction through an anterolateral approach with femoral and iliac osteotomies

4) Closed reduction and Salter osteotomy

 

At the age of 4, femoral shortening is indicated to remove the pressure on the reduced femoral head. Realignment of the bony dysplasia is achieved by femoral derotation, iliac redirection, and possible creation of varus in the proximal femur. Open reduction through a medial approach is an option during the first 2 years, but after that the anterolateral approach is preferred in order to create a stable capsulorrhaphy.Correct Answer: Open reduction through an anterolateral approach with femoral and iliac osteotomies

 

A 3-year-old boy falls from a swing and injures his elbow. Radiographs show posteromedial displacement of the radius and ulna. A small fleck of bone is viewed on the radiograph that does not match the contralateral elbow. The most likely diagnosis is:

 

1) Elbow dislocation

3) Lateral condyle fracture with elbow dislocation

2) Salter I disruption of the distal humerus

5) Radial head fracture dislocation

4) Supracondylar fracture of the humerus

 

Isolated elbow dislocations are rare in very young children because the epiphysis is mostly cartilaginous. Supracondylar fractures usually occur at an older age and have an obvious fracture line. A Salter I injury does not have an osseous component. A radial head fracture-dislocation would not explain the disruption of the ulna-humerus articulation. A lateral condyle fracture-dislocation is the most likely diagnosis in this patient.Correct Answer: Lateral condyle fracture with elbow dislocation

 

 

2037. (1376) Q5-1747:

A 6-month-old baby is brought in for consultation because of bowing of the tibia and fibula. The apex of the bow is medial and posterior. The angulation measures 40° on the anteroposterior film and 35° on the lateral film. One of the babyâs legs is 1.5 cm shorter than the other one. Recommended treatment includes:

 

1) Manipulation and cast application

3) Osteotomy and intramedullary rod fixation

2) Osteoclasis

5) Observation

4) Osteotomy and lengthening

 

Posteromedial bowing of the tibia is not likely to go on to fracture. In most cases, the bowing resolves with growth. The length inequality remains proportionate throughout growth. No treatment is indicated at this time. Length equalization by standard means is indicated near maturity.Correct Answer: Observation

 

 

2038. (1377) Q5-1748:

Which of the following bone lesions is shown to improve radiographically and clinically by use of systemic medication:

 

1) Aneurysmal bone cyst

3) Multiple exostoses

2) Unicameral bone cyst

5) Fibrous cortical defect

4) Fibrous dysplasia

 

In patients with polyostotic fibrous dysplasia, there is a decrease in pain and fracture rate and an improvement in radiodensity through the use of bisphosphonates. None of the other lesions mentioned respond radiographically to systemic medication.Correct Answer: Fibrous dysplasia

 

 

2039. (1378) Q5-1749:

Polyostotic fibrous dysplasia is caused by a mutation in which of the following genes:

 

1) GNAS 1

3) Fibroblast growth factor receptor protein

2) EXT 1

5) COL1A1

4) CFBA1

 

Fibrous dysplasia is due to a postzygotic mutation in the GNAS1 gene. By contrast, EXT 1 mutations can cause multiple exostoses, and fibroblast growth factor receptor protein mutations cause achondroplasia, among other disorders. CFBA1 mutations are responsible for cleidocranial dysplasia. COL1A1 mutations are responsible for osteogenesis imperfecta.Correct Answer: GNAS 1

 

An 18-year-old man has a lesion of the distal femur. The femoral cortex is expanded and thinned but has no periosteal reaction. Internally, there are septations and multiple fluid-fluid levels. The most likely diagnosis is:

 

1) Unicameral bone cyst

3) Parosteal osteosarcoma

2) Aneurysmal bone cyst

5) Fibrous cortical defect

4) Classic osteogenic sarcoma

 

The presence of all of these findings is most consistent with aneurysmal bone cyst. The fluid-fluid levels represent settling of red blood cells. Unicameral bone cysts do not contain these findings. Parosteal osteosarcomas are surface lesions. Classic osteosarcomas usually produce periosteal reaction and not much expansion of the bone. Fibrous cortical defects produce none of the above.Correct Answer: Aneurysmal bone cyst

 

 

2041. (1380) Q5-1751:

A 17-year-old girl is newly diagnosed with an expansile lesion of the distal tibia that is causing pain. Radiographs and biopsy are consistent with an aneurysmal bone cyst. Recommended treatment includes:

 

1) Intralesional steroid injection

3) Curettage and bone graft

2) Intralesional bone marrow injection

5) Local radiation

4) Wide resection distal tibial osteoarticular allograft

 

Aneurysmal bone cyst is a progressively expansile lesion that does not involute with time. Treatment of aneurysmal bone cysts is curettage and bone graft. Wide resection and structural allografts have a high complication rate and are not indicated for initial treatment. Injection of marrow or steroids is effective for unicameral cysts, but not for aneurysmal cysts. Local radiation is indicated only as an adjunct for lesions that are not surgically resectable.Correct Answer: Curettage and bone graft

 

 

2042. (1381) Q5-1752:

Platelet calmodulin levels correlate with which of the following phenomena:

 

1) Progressive adolescent idiopathic scoliosis

3) Progressive slipped capital epiphysis

2) Progressive Scheuermann kyphosis

5) Progressive clubfoot

4) Bilateral Legg-Perthes disease

 

Platelets contain actin and myosin and, therefore, have some features similar to skeletal muscle. Calmodulin is a calcium-binding receptor protein that regulates intracellular calcium. Platelet calmodulin levels are increased in progressive adolescent idiopathic scoliosis.Correct Answer: Progressive adolescent idiopathic scoliosis

 

 

2043. (1382) Q5-1753:

The proximal humerus is the most common location of unicameral bone cysts. Which of the following is the second most common location:

 

1) Distal radius

3) Proximal femur

2) Iliac wing

5) Proximal tibia

4) Distal femur

 

The proximal femur is the second most common location for unicameral bone cysts. All other locations are distinctly less common.Correct Answer: Proximal femur

 

An 8-year old patient presents with pain in a unicameral bone cyst of the proximal femur. Which of the following treatments is most likely to prevent fracture:

 

1) Intralesional steroid injection

3) Intralesional injection of demineralized bone matrix

2) Intralesional bone marrow injection

5) Curettage, bone graft, and internal fixation

4) Curettage and bone graft

 

Injection of any substance has a risk of recurrence, even if the injection is repeated. Bone graft has a risk as well, but internal fixation decreases the risk of fracture.Correct Answer: Curettage, bone graft, and internal fixation

 

 

2045. (1384) Q5-1755:

All of the following are common in McCune-Albright syndrome except:

 

1) Precocious puberty

3) Café-au-lait spots

2) Polyostotic fibrous dysplasia

5) Multiple osteocartilaginous exostoses

4) Bowing of long bones

 

McCune-Albright syndrome consists of polyostotic fibrous dysplasia, precocious puberty, and café-au-lait spots. Long bone deformity is almost universal. Multiple osteocartilaginous exostoses are not part of this syndrome.Correct Answer: Multiple osteocartilaginous exostoses

 

 

2046. (1385) Q5-1756:

Which of the following is most likely to minimize bowing of the femora over the long term in a young patient with polyostotic fibrous dysplasia:

 

1) Bisphosphonates treatment

3) Application of allografts to lesions

2) Curettage and autograft packing of lesions

5) Injection of demineralized bone graft

4) Internal metal fixation

 

Bowing of the femora is a major problem for patients with polyostotic fibrous dysplasia. Bisphosphonates can decrease bone pain and increase bone density somewhat, but they do not prevent bowing. Allograft, autograft, and demineralized matrix are rapidly resorbed in most cases. Internal fixation with metal provides the best long-term protection.Correct Answer: Internal metal fixation

 

 

2047. (1386) Q5-1757:

Which of the following factors is not associated with increased risk for progressive slippage after internal fixation of slipped capital femoral epihysis:

 

1) Acute component

3) Cushing disease

2) Hyperthyroidism

5) Male gender

4) Persistent pain after fixation

 

All of the factors listed, except gender, have been implicated in cases with increased progression of slipped capital femoral epiphysis even after pinning. Patients with these factors need close follow-up with a prolonged period of nonweightbearing or redirectional osteotomies.Correct Answer: Male gender

 

Femoral anteversion is defined as the angle in the transverse plane between the femoral neck and which distal anatomical reference point:

 

1) The lesser trochanter

3) The posterior femoral condylar plane

2) The femoral shaft

5) The tibial plateau

4) The distal femoral condylar plane

 

Anteversion of the femur is a measure of its rotation in the transverse plane. It is defined as the angle between the plane containing the femoral head/neck and that containing the femoral condyles. Anteversion declines from approximately 25° in infancy to approximately 15° at maturity.Correct Answer: The distal femoral condylar plane

 

 

2049. (1388) Q5-1759:

The phenomenon of spinal cord injury without radiographic abnormality in children may be due to any one of the following except:

 

1) Increased longitudinal stretch of the skeletal elements compared to the spinal cord

3) Apophyseal injury

2) Increased physiologic translation of the cervical vertebrae

5) Lack of neural myelination in children

4) Transverse ligament injury of the atlas

 

In most cases, spinal cord injury without radiographic abnormality, or SCIWORA, is due to the greater elasticity or translation of the skeletal elements. Apophyseal or transverse ligament injuries are other explanations. Myelination of the cord is complete after birth so this is not an explanation.Correct Answer: Lack of neural myelination in children

 

 

2050. (1389) Q5-1760:

A 6-year-old girl is seriously injured in an automobile accident. She remains unconscious and intubated 6 days after the injury due to head and pulmonary injuries. She is expected to survive. A firm cervical collar was placed on her neck at the time of rescue and remains in place. Plain radiographs show no cervical abnormalities. At this time, recommended treatment includes:

 

1) Removing the collar

3) Passive flexion and extension radiographs by the physician to clear the spine of injury

2) Leaving the collar in place until she awakens

5) Magnetic resonance imaging to clear the spine of injury

4) Ultrasound of the cervical spine to clear it of injury

 

Spinal cord injury without radiographic abnormality (SCIWORA) may occur in children. Motor vehicle accidents and head injury are two risk factors for this. The neck collar cannot be left on indefinitely or it may cause pressure sores. Therefore, it is most prudent to evaluate the cervical spine with magentic resonance imaging if the patientâs neck cannot be cleared by physical exam. Passive range of the neck is risky, and ultrasound is not used for this purpose.Correct Answer: Magnetic resonance imaging to clear the spine of injury

 

 

2051. (1390) Q5-1761:

Which of the following properties is the same for both botulinum toxin types A and B:

 

1) Dose

3) Antigenicity

2) Duration of action

5) Molecular structure

4) Mechanism of action

 

There are seven serologic subtypes of botulinum toxin, and only two are approved for clinical use. The duration of action of type A is slightly longer and it is less antigenic. Both type A and B have different structures. The dose of type A is lower in absolute units. Both have the same site of action â they inhibit the release of acetylcholine from the neuromuscular junction, although they act on different proteins inside the cell.Correct Answer: Mechanism of action

 

Stickler syndrome is caused by a mutation in the gene for which of the following:

 

1) Collagen

3) Glycoprotein

2) Fibrillin

5) Fibroblast growth factor

4) Osteopontin

 

Stickler syndrome is characterized by premature osteoarthritis in multiple joints, visual complications, auditory defects, and cleft palate. Stickler syndrome is caused by a defect in the genes for type 2 or type 11 collagen.Correct Answer: Collagen

 

 

2053. (1549) Q5-1941:

Stickler syndrome is characterized by all of the following features except:

 

1) Visual disorders

3) Degenerative joint disease

2) Hearing problems

5) Irregular vertebral endplates

4) Significant short stature

 

Stickler syndrome, also known as hereditary arthro-ophthalmopathy, is characterized by visual problems, progressive hearing loss, and malar or mandibular hypoplasia. In terms of orthopedic complications, most patients develop premature degenerative joint disease and disk abnormalities, including narrow and irregular endplates. Significant short stature is not common in patients with Stickler syndrome.Correct Answer: Significant short stature

 

 

2054. (1550) Q5-1942:

The majority of pediatric pedestrian-vehicle injuries occur in which time period:

 

1) 12 AM to 5 AM

3) 10 AM to 3 PM

2) 5 AM to 10 AM

5) 8 PM to 12 PM

4) 3 PM to 8 PM

 

The majority (54%) of pediatric pedestrian-vehicle accidents occur from 3 pm to 8 pm. This time period may coincide with walks home from school, after-school play, and high traffic volume.Correct Answer: 3 PM to 8 PM

 

 

2055. (1551) Q5-1943:

Which of the following demographic factors places a child at increased risk of pedestrian vs vehicle trauma:

 

1) Female gender

3) 5 to 9 years old

2) Younger than 5 years old

5) 15 to 19 years old

4) 10 to 14 years old

 

Male gender and age 5 to 9 years old are factors that place children at increased risk of being struck by a vehicle.Correct Answer: 5 to 9 years old

 

Before any intervention, which of the following statements is true regarding the walking gait of a diplegic patient who has an equinus gait:

 

1) Ankle plantarflexion increases during single-limb stance.

3) Ankle plantarflexion does not change during single-limb stance.

2) Ankle plantarflexion decreases steadily during single-limb stance.

5) Knee flexion initially increases during stance-phase loading.

4) Ankle varus increases during stance.

 

In patients with spastic diplegia, ankle plantarflexion increases during single-limb stance, contrary to the normal pattern.Correct Answer: Ankle plantarflexion increases during single-limb stance.

 

 

2057. (1553) Q5-1945:

Which of the following complications is the most common in anterior iliac crest graft harvesting:

 

1) Hematoma

3) Sensory disturbance

2) Muscle herniation

5) Fracture

4) Pain

 

Transient or permanent sensory disturbance is the most common complication (13% of patients) of anterior iliac crest graft harvesting.Correct Answer: Sensory disturbance

 

 

2058. (1554) Q5-1946:

Which of the following is the most common site for primary epiphyseal osteomyelitis:

 

1) Proximal femur

3) Proximal tibia

2) Distal femur

5) Proximal humerus

4) Distal tibia

 

Osteomyelitis of the epiphysis is less common than osteomyelitis of the metaphysis. Osteomyelitis of the epiphysis may occur from spread across transphyseal vessels or primary hematogenous seeding. The most common location is the distal femoral epiphysis, and it most commonly affects infants and young children.Correct Answer: Distal femur

 

 

2059. (1555) Q5-1947:

A normal (negative) result is used to rule out septic arthritis of the hip on which of the following tests:

 

1) Plain radiographs

3) Culture of joint fluid

2) Ultrasound

5) None of the above

4) C-reactive protein

 

False-negative results may occur with each of the tests. Culture results may be negative in up to a quarter of patients. Ultrasound may be false-negative in up to 20% of patients with septic arthritis of the hip. C-reactive protein may be normal early in the course of disease. The diagnosis is based upon a constellation of findings, including clinical examination.Correct Answer: None of the above

 

Which of the following structures is the primary stabilizer of the atlantoaxial segment against anterior atlantal translation:

 

1) Apical ligament

3) Anterior atlantodental ligament

2) Alar ligament

5) Ligamentum nuchae

4) Transverse ligament

 

The transverse ligament is the primary stabilizer of the atlantoaxial segment against anterior atlantal translation. The transverse ligament runs between the lateral masses of C 1 and behind the odontoid process. The apical ligament is attached to the tip of the odontoid and the occiput, but not C 1. The paired alar ligaments run obliquely and are secondary restraints, as is the anterior atlantodental ligament. The ligamentum nuchae is a strong condensation of fibers extending from the external occipital protuberance to the tips of the spinous processes C 2-C 7.Correct Answer: Transverse ligament

 

 

2061. (1557) Q5-1949:

The radiographic line delimiting the foramen magnum that is used in determining basilar invagination is the:

 

1) McGregor line

3) Chamberlain line

2) McRae line

5) Swischuk line

4) Ranawat line

 

The McRae line is from the anterior to the posterior lip of the foramen magnum. Protrusion of the odontoid above this line indicates basilar invagination. The McGregor and Ranawat lines are also used to evaluate basilar invagination. The Swischuk line is from the posterior cortex of C 1 to C 3 lamina and is used in evaluating pseudosubluxation.Correct Answer: McRae line

 

 

2062. (2216) Q5-2643:

Which of the following factors determines the treatment for a child with tibial hemimelia:

 

1) The presence of a patella

3) The presence of a proximal tibial remnant

2) The length of the fibula

5) The status of the menisci

4) The status of the cruciate ligaments

 

When treating a child with tibial hemimelia, the surgeon must decide whether to attempt to reconstruct the knee or perform a disarticulation. The determining factor is whether there is a proximal tibial remnant.

Correct Answer: The presence of a proximal tibial remnant

 

 

2063. (2217) Q5-2644:

Which of the following is a contraindication to the use of vacuum assisted closure (VAC) (Kinetic Concepts Inc., San Antonio, Tex):

 

1) A deep surgical wound

3) A wound with necrotic muscle that has not been debrided

2) An acute soft tissue wound

5) A chronic wound that is slow to close

4) A fresh, split-thickness skin graft

 

Vacuum assisted closure (VAC) should be applied to infected or damaged tissue after debridement. All of the other possible answer choices are conditions for which VAC is appropriate.

Correct Answer: A wound with necrotic muscle that has not been debrided

 

The recommended interval for changing wound vacuum assisted closure (VAC) (Kinetic Concepts Inc., San Antonio, Tex) dressings is:

 

1) Twice daily

3) Every 2 days

2) Daily

5) Monthly

4) Weekly

 

The preferred interval for changing VAC dressings is every 2 days. Patients may have significant discomfort with initial dressing changes; however, the pain usually diminishes rapidly.Correct Answer: Every 2 days

 

 

 

2065. (2219) Q5-2646:

In children with Ewingâs sarcoma, the risk of local recurrence at the tumor site after treatment is greatest in which region:

 

1) Midshaft femur

3) Pelvis

2) Proximal fibula

5) Proximal humerus

4) Distal femur

 

The risk of local recurrence is greatest for Ewingâs sarcomas arising in the pelvis. The prognosis is poor. Many centers attempt resection of the pelvis if there is a good response to chemotherapy.

Correct Answer: Pelvis

 

 

2066. (2220) Q5-2647:

What is the histological difference between avascular necrosis of the femoral head in children versus adults:

 

1) Children have less creeping substitution than adults.

3) Large areas of fibrovascular tissue do not form in children.

2) Children do not demonstrate osseous collapse.

5) Children do not have any residual changes after 2 years.

4) There is no osteoclastic resorption in children.

 

Avascular necrosis models of the femoral head in immature animals show more osteoclastic resorption, fibroblastic response, and little creeping substitution when compared to models of mature animals. Osseous collapse is common, and there are often longterm residual changes in the shape of the femoral head.Correct Answer: Children have less creeping substitution than adults.

 

 

 

2067. (2221) Q5-2648:

Which of the following is a principle of the Ponseti technique for correction of a clubfoot:

 

1) Avoid use of casts

3) Perform a comprehensive release

2) Avoid use of braces

5) Minimize surgery to bones and joints

4) Osteotomies produce correction

 

The Ponseti technique, which was developed and tested by Ignacio Ponseti, MD, involves slow, gradual correction of a clubfoot using casts, a tenotomy if necessary to release the Achilles tendon, and maintenance of correction using braces (foot abduction orthoses) for several years. The technique avoids dissection of the growing bones and joints of a childâs foot because of the associated risks of stiffness and growth disturbance.Correct Answer: Minimize surgery to bones and joints

 

 

Which of the following pulse sequences is best for showing anatomy on magnetic resonance imaging (MRI):

 

1) T1-weighted images

3) Proton density images

2) T2-weighted images

5) Gradient echo sequence

4) Fat-suppressed T2-weighted images

 

T1-weighted images generally display the best anatomical detail. Fat is bright, and muscle is dark, giving excellent contrast. Cortical bone, tendons, and ligaments are low signal.Correct Answer: T1-weighted images

 

 

 

2069. (2223) Q5-2650:

Which of the following pulse sequences is best for imaging the pediatric growth plate:

 

1) T1-weighted images

3) Proton density images

2) T2-weighted images

5) Gradient echo sequence

4) Fat-suppressed T2-weighted images

 

The gradient echo sequence involves a short relaxation and excitation time. It shows both fat and water as intermediate signals. The gradient echo sequence is excellent for imaging physeal and articular cartilage. When imaging for a physeal bar, the physician ordering the magnetic resonance imaging should specify this sequence.Correct Answer: Gradient echo sequence

 

 

 

2070. (2224) Q5-2651:

Motion artifact in magnetic resonance imaging of the pediatric spine is caused by all of the following except:

 

1) Patient movement

3) Cerebrospinal fluid flow

2) Cardiac activity

5) A flexible titanium rod in the femur

4) Respiration

 

Motion artifact affects magnetic resonance imaging of the spine and can result from patient movement (common in children under 8 years old), cardiac activity, respiration, and cerebrospinal fluid flow. Presence of a titanium rod in a childâs femur, while causing a local signal void, does not affect spinal imaging.Correct Answer: A flexible titanium rod in the femur

 

 

 

2071. (2225) Q5-2652:

There are no internal moments in the lower extremity during which phase of gait:

 

1) Initial contact

3) Terminal stance

2) Midstance

5) Initial swing

4) Preswing

 

Preswing is the only phase of gait in which all muscle groups are silent in the ipsilateral lower extremity. In the next phase (initial swing), internal moments are generated at the hip and ankle to initiate swing.Correct Answer: Preswing